Ordinary Differential Equations - BAMAT-201
Ordinary Differential Equations - BAMAT-201
Ordinary Differential Equations - BAMAT-201
EQUATIONS
BAMAT-201
Reviewed by :
Assessed by:
Study Material Assessment Committee, as per the SVSU ordinance No. VI (2).
No part of this publication which is material protected by this copyright notice may be
reproduced or transmitted or utilized or stored in any form or by any means now known or
hereinafter invented, electronic, digital or mechanical, including photocopying, scanning,
recording or by any information storage or retrieval system, without prior permission from
the publisher.
Information contained in this book has been published by Laxmi Publications Pvt Ltd and has
been obtained by its authors from sources believed to be reliable and are correct to the best
of their knowledge. However, the publisher and its author shall in no event be liable for any
errors, omissions or damages arising out of use of this information and specially disclaim and
implied warranties or merchantability or fitness for any particular use.
Published by : Laxmi Publications Pvt Ltd., 113, Golden House, Daryaganj, New Delhi-110 002.
Tel: 43532500, E-mail: [email protected]
DEMC—
Typeset at: Printed at:
Edition: 2020
CONTENTS
UNIT I
1. DIFFERENTIAL EQUATIONS 1
Differential Equation ......................................................................................................................................................... 1
Formation of a Differential Equation Whose General Solution is Given ....................................................................... 4
Solution of a Differential Equation ................................................................................................................................ 13
Initial Value Problem ...................................................................................................................................................... 16
Solution of a Differential Equation by The Method of Separation of Variables ........................................................... 17
Homogeneous Differential Equations and their Solution .............................................................................................. 33
dy
Solution of Linear Differential Equation + Py = Q, where P and Q are functions of x or constants ................... 50
dx
Summary ......................................................................................................................................................................... 68
UNIT II
UNIT III
Self-Instructional Material i
Equations Solvable for x ............................................................................................................................................... 100
Clairaut’s Equation ........................................................................................................................................................ 102
d2 y dy
To find a Particular Integral of +P + Qy = 0 ............................................................................................. 138
dx 2 dx
Removal of the First Derivative (Ruduction to Normal Form) ................................................................................... 146
Transformation of the Equation by Changing the Independent Variable .................................................................... 153
Method of Variation of Parameters ............................................................................................................................... 160
UNIT IV
d2 y dy
Power Series Solution, when x = 0 is an Ordinary Point of the Equation 2
+ P(x ) + Q(x)y = 0 ................. 169
dx dx
Frobenius Method: Series Solution when x = 0 is a Regular Singular Ppoint of the Differential Equation
d2 y dy
2
+ P(x ) + + Q(x)y = 0 ................................................................................................................................... 177
dx dx
6. DIFFERENTIAL EQUATIONS
Introduction ................................................................................................................................................................... 195
Legendre’s Function of First kind Pn(x) ...................................................................................................................... 197
Legendre’s Function of Second kind Qn (x) ................................................................................................................ 197
Solution of Legendre’s Equation .................................................................................................................................. 198
Generating Function for Pn(x) ...................................................................................................................................... 198
ii Self-Instructional Material
Rodrigue’s Formula ....................................................................................................................................................... 202
Recurrence Relations .................................................................................................................................................... 206
Beltrami’s Result ........................................................................................................................................................... 208
Orthogonality of Legendre Polynomials ...................................................................................................................... 208
Laplace’s Integral of First Kind .................................................................................................................................... 210
Laplace’s Integral of Second Kind ............................................................................................................................... 210
Cristoffel’s Expansion Formula .................................................................................................................................... 211
Cristoffel’s Summation Formula .................................................................................................................................. 212
Expansion of a Function in a Series of Legendre Polynomials (Fourier-Legendre Series) ....................................... 213
Course I
Course The main objectives of this course are to introduce the students to
Objectives: the exciting world of Differential Equations and their applications.
Unit 1: Formation of differential equation, Degree, order and solution of
a D.E., Ordinary differential equations of first order: initial and
boundary conditions,Seperation of variables method, homogeneous
equations:equation reducible to Homogeneous Form, linear equations,
Equation reducible to homogeneous form
Unit 2: Exact differential Equation. Necessary and sufficient condition for exact
differential equation, First order higher degree equations solvable for
x, y, p. Singular solution and envelopes,Clairaut’s equation,Equation
Reducible to Clauriat,s form.
Unit 3: Linear differential equations with constant coefficients; Determinaton of
C.F. and the P.I., homogeneous linear differential equations, Determinaton
of C.F. and the P.I., linear differential equations of second order with
variable coefficients,
Unit 4: Series solutions of differential equations. Introduction Frobenious Method
Solution near an ordinary point and a regular singular point, Method of
differentiation, Bessel and Legendre equations. Solution of Legendre
equation, Defination of Legendre polynomials, Bessel and Legendre
functions.
Course Learning Outcomes: The course will enable the students to:
References:
1. Barnes, Belinda & Fulford, Glenn R. (2015). Mathematical Modelling with Case Studies,
Using Maple and MATLAB (3rd ed.). CRC Press, Taylor & Francis Group.
2. Edwards, C. Henry, Penney, David E., & Calvis, David T. (2015). Differential Equation and
Boundary Value Problems: Computing and Modeling (5th ed.). Pearson Education.
Ross, Shepley L. (2004). Differential Equations (3rd ed.). John Wiley & Sons. India
UNIT I Differential Equations
NOTES
1. DIFFERENTIAL EQUATIONS
STRUCTURE
Differential Equation
Formation of a Differential Equation Whose
General Solution is Given
Solution of a Differential Equation
Initial Value Problem
Solution of a Differential Equation by the Method of Separation of Variables
Homogeneous Differential Equations and their Solution
DIFFERENTIAL EQUATION
d2y dy dy
dy
2
3.
dx 2
4
dx
12y = x4 4. y = x
dx
+ a 1
dx
.
Self-Instructional Material 1
Ordinary Differential
Equations
F [
The highest order derivative occurring in this equation is and its order
FZ
is 3.
Order of given differential equation is 3.
NOTES
The degree of a differential equation is defined if it can be written as a
polynomial equation in the derivatives and for such a differential equation its degree
is given by the highest power of the highest order derivative appearing in it, provided
the derivatives are made free from radicals and fractions.
Consider the differential equation
F[ F[
y=2
FZ
+ 3
FZ . ...(1)
F[ = 3 F[
(1) y2
FZ FZ
[ F[ = 9 F[
FZ FZ
F[
y + 4 4y
F[ F[
9 18 = 0
2
FZ FZ FZ
F[
14 + 4y
F[
FZ y +9=0 2
FZ
The highest order derivative in this equation is
F[ and its highest power is 2.
FZ
Degree of given differential equation is 2.
FP [ FP [ F[
P0 + P + + Pn 1 + Pn y = Q,
FZ P 1
FZ P FZ
where P0, P1, ..., Pn 1, Pn, Q are functions of x or constants.
In particular, a linear differential equation of order one is of the form
P0
F[ + P y = Q.
FZ 1
dy d3 y y
The differential equations: cos x + y sin x = 1 and 3 + = x2 log x are linear
dx dx x
differential equations.
A differential equation which is not linear is called non-linear.
The degree of a linear differential equation is always one. But, the converse is
dy
not true. For example, the degree of y 7 sin x is one and it is not a linear
dx
differential equation.
2 Self-Instructional Material
SOLVED EXAMPLES Differential Equations
dy (1 y 2 )(1 x x 2 )
Solution. (i) The given differential equation is xy = .
dx 1 x2
dy
Order of the highest order derivative is 1.
dx
dy
Highest power of the highest order derivative is 1.
dx
Order and degree of the given differential equation are 1 each.
dy
The given differential equation is non-linear, because y and are multiplied
dx
together.
dy dy 3
(ii) The given differential equation is y =
dx
+ 1
dx . ...(1)
dy
Order of the highest order derivative is 1.
dx
dy dy 3
y dy 2
dy 3
(1) y
dx
= 1
dx
dx =1+
dx
dy
This is expressible as a polynomial in .
dx
dy
Highest power of the highest order derivative is 3.
dx
Order and degree of the given differential equation are 1 and 3 respectively.
dy
The given differential equation is non-linear because is multiplied by itself.
dx
EXERCISE A
Determine the order and degree, if defined, of the following differential equations. State also, if
these are linear or non-linear (Q No. 14):
F [ Z F[ = 0 F[ Z F [ = 0
1. (i) Z
FZ
FZ (ii)
FZ FZ
F[ [ F [ = 0 (iv)
F [ [ F[ + x = 0
FZ FZ FZ
(iii) 3
FZ
dy
2
d2 y d y dy
2
x
3 d y 3
2
dx
2. (i) 5 x
dx
dx 2
6 y log x (ii) y
dx 2 dx 3
2
dy dy
(iii) y2 2y y + 1 = 0 (iv) y = x +a 1
dx dx
Self-Instructional Material 3
Ordinary Differential
Equations dy
(i) 1
2 "# 3/ 2
d2 y
! dx
3. =5 (ii) 1 x2 dx + 1 y2 dy = 0
#$ dx2
4
NOTES ds d2 s dy
(iii) + 3s =0 (iv) y = px + a2 p2 b2 , where p =
dt dt2 dx
2
d4y d2 y dy
4. (i) sin ( y ) = 0 (ii) cos =0
dx 4
dx 2 dx
F %K F[ (K
(i) &K FZ )K (ii)
F [ F[
Z
FZ ' * FZ FZ
Answers
1. (i) Order = 2, degree = 2, non-linear (ii) Order = 2, degree = 1, non-linear
(iii) Order = 2, degree = 1, non-linear (iv) Order = 2, degree = 3, non-linear
2. (i) Order = 2, degree = 1, non-linear (ii) Order = 3, degree = 2, non-linear
(iii) Order = 2, degree = 2, non-linear (iv) Order = 1, degree = 2, non-linear
3. (i) Order = 2, degree = 2, non-linear (ii) Order = 1, degree = 1, non-linear
(iii) Order = 2, degree = 1, non-linear (iv) Order = 1, degree = 2, non-linear
4. (i) Order = 4, degree not defined, non-linear
(ii) Order = 2, degree not defined, non-linear
(iii) Order = 3, degree = 2, non-linear
(iv) Order = 2, degree = 1, non-linear
5. (i) 3 (ii) 5.
4 Self-Instructional Material
Differential Equations
Remark. The following are some of the important results of coordinate geometry which
are used in this section.
1. Equation of non-vertical line is y = mx + c, where m and c are arbitrary constants.
2. Equation of a non-vertical line passing through the origin is y = mx, where m is
NOTES
arbitrary constant.
3. Equation of the circle having centre (h, k) and radius r is (x h)2 + (y k)2 = r2.
4. Equation of circle in the general form is x2 + y2 + 2gx + 2fy + c = 0. Its centre and
radius are ( g, f) and g 2 f 2 c respectively.
5. Equation of a circle passing through the origin is x2 + y2 + 2gx + 2fy = 0, where g and
f are arbitrary constants.
6. Equation of a circle passing through the origin and having centre on the
x-axis is (x a)2 + y2 = a2, where a is arbitrary constant.
7. Equation of a circle passing through the origin and having centre on the
y-axis is x2 + (y a)2 = a2, where a is arbitrary constant.
8. Equation of a parabola with axis parallel to the x-axis is (y k)2 = 4a(x h), where a,
h and k are arbitrary constants.
9. Equation of a parabola with axis parallel to the y-axis is (x h)2 = 4a(y k), where a,
h and k are arbitrary constants.
10. Equation of an ellipse having centre at the origin and axes along the coordinate axes
x2 y2
is = 1, where a and b are arbitrary constants.
a2 b2
SOLVED EXAMPLES
Example 2. Form the differential equation of the following families of curves:
(i) y = mx, where m is an arbitrary constant.
(ii) (x a)2 + 2y2 = a2, where a is an arbitrary constant.
Solution. (i) We have y = mx. ...(1)
dy
Differentiating (1) w.r.t. x, we get =m
dx
F[ x.
Elimination of m. Putting the value of m in (1), we get y =
FZ
This is the required differential equation.
(ii) We have (x a)2 + 2y2 = a2 i.e., x2 2ax + 2y2 = 0 ...(1)
Differentiating (1) 2x 2a + 4yy = 0 a = x + 2yy
Self-Instructional Material 5
Ordinary Differential Elimination of a. Putting a = x + 2yy in (1), we get
Equations x2 2(x + 2yy)x + 2y2 = 0
x2 2x2 4xyy + 2y2 = 0 4xyy + x2 2y2 = 0
4xy
F[ + x2 2y2 = 0. This is the required differential equation.
NOTES FZ
Remark. The differential equation obtained for each system in the above example is of
order one. This is so, because each system contained only one arbitrary constant.
d2y dy
x +2 = 2A
dx 2 dx
Elimination of A and B. Putting the value of 2A in (2), we get
dy d2y dy
x
dx
+y= x 2 2
dx dx
x
x2
F[ +x
F[ y = 0. This is the required differential equation.
FZ FZ
(ii) We have y = Ae3x + Be5x. ...(1)
dy
Differentiating (1) w.r.t. x, we get = 3Ae3x + 5Be5x ...(2)
dx
d2y
Differentiating again = 9Ae3x + 25Be5x ...(3)
dx 2
Elimination of A and B.
d2y dy 1 d 2y 5 dy
(3) 5(2) 2 5 = 6Ae3x + = Ae3x
dx dx 6 dx 2 6 dx
d2y dy 1 d2y 3 dy
(3) 3(2) 3 = 10Be5x = Be5x
dx 2 dx 10 dx 2 10 dx
F [ F[ F [ F[
(1) y=
FZ FZ
+
FZ FZ
d2y dy d2y dy
30y = 5 + 25 +3 9
dx 2 dx dx 2 dx
6 Self-Instructional Material
d 2y dy Differential Equations
2 16 + 30y = 0
dx 2 dx
F [ 8
F[ + 15y = 0. This is the required differential equation.
FZ FZ NOTES
Alternative Method
We have y = Ae3x + Be5x. ...(1)
(1) y1 = 3Ae3x + 5Be5x ...(2)
(2) 3(1) y1 3y = 2Be5x ...(3)
(3) y2 3y1 = 10Be5x y2 3y1 = 5(y1 3y) y2 8y1 + 15y = 0
F [
8
F[ + 15y = 0. This is the required differential equation.
FZ FZ
Remark The differential equation obtained for each system in the above example is of
order two. This is so, because each system contained two arbitrary constants.
d y
7
F[ + 6y = 0. This is the required differential equation.
dx FZ
(ii) We have x2 + y2 + 2ax + 2by + c = 0. ...(1)
Differentiating (1) w.r.t. x, we get 2x + 2yy1 + 2a + 2by1 + 0 = 0.
x + yy1 + a + by1 = 0 ...(2)
Differentiating (2) w.r.t. x, we get 1 + (yy2 + y1y1) + 0 + by2 = 0
by2 = (1 + yy2 + y12) ...(3)
Differentiating (3) w.r.t. x, we get by3 = (0 + yy3 + y1y2 + 2y1y2)
by3 = (yy3 + 3y1y2) ...(4)
by2
[[ [
Elimination of a, b and c. Dividing (3) by (4), we get =
by3
[[ [ [
[ [[ [
= yy2y3 + 3y1y22 = y3 + yy2y3 + y12y3
[ [[ [ [
3y1y22 = y3 + y12y3 (1 + y21)y3 3y1y22 = 0
Self-Instructional Material 7
Ordinary Differential
F[ "# d y F[ d y = 0.
Equations
dx
! FZ #$
3
dx FZ
This is the required differential equation.
NOTES
Remark The differential equation obtained for each system in the above example is of
order three. This is so, because each system contained three arbitrary constants.
Example 5. Form the differential equation of all lines in a plane which are at a
constant distance p from the origin.
Solution. The distance of the lines of the Y
family from the origin is p. Let L be a line of this
L
family. Draw OK perpendicular to this line. Let OK
make angle with the x-axis.
The equation of the line L is K
x cos + y sin = p.
p
x cos + y sin p = 0 ...(1) a
Differentiating (1) w.r.t. x, we get X¢ O X
cos + y1 sin 0 = 0 ...(2)
Y¢
Solving (1) and (2), we get
cos UKP 1
= =
0 py1 R xy1 y
R[ R
EQU CPF UKP
Z[ [ Z[ [
We have cos2 + sin2 = 1.
R[
R
Z[ [ Z[
[
p2 y12 p2 ( xy1 – y)2
R [ R Z [ [ Z[[
R Z [ Z[[ R [
R Z F[ Z[
F[ R [
FZ FZ
This is the required differential equation.
Example 6. Form the differential equation of the system of circles touching the
x-axis at the origin. Y
Solution. The circles in the system will have their
centres on the y-axis. Let (0, a) be the centre of a circle
touching the x-axis at the origin. |a|
(0, a)
The radius this circle must be |a|*, otherwise the
circle will not touch the x-axis.
The equation of the system of circle is
X¢ X
(x 0)2 + (y a)2 = (|a|)2 Y¢
or x2 + y2 2ay = 0, where a is an arbitrary constant.
*Why this step If the centre of the circle is below the origin, then a is negative. For
such a circle, the radius of circle is a, which is equal to |a|.
8 Self-Instructional Material
Differentiating w.r.t. x, we get Differential Equations
2x + 2yy1 2ay1 = 0 ...(1)
Elimination of a.
x yy1 NOTES
(1) a=
y1
Putting the value of a in x2 + y2 2ay = 0, we get
x yy y 0 .
x2 y2 – 2 y
1
1
x2 y1 y2 y1 – 2 xy – 2 y2 y1 0 ( x2 – y2 ) y1 2 xy
Z [ F[ Z[. This is the required differential equation.
FZ
Remark. This differential equation also represent the system of circles passing through
the origin and having centre on the y-axis.
Example 7. (i) Form the differential equation of all parabolas with latus rectum
4a and whose axes are parallel to the x-axis.
(ii) Form the differential equation of all parabolas whose axes are parallel to the
y-axis.
Solution. (i) The equation of a parabola with latus rectum 4a and axis parallel
to the x-axis is
(y k)2 = 4a(x h), ...(1)
where h and k are arbitrary constants.
Differentiating (1) w.r.t. x, we get
2(y k)(y 0) = 4a(1 0) i.e., (y k)y = 2a ...(2)
Differentiating (2) w.r.t. x, we get
y 2
(y k)y + (y 0)y = 0 i.e., yk= ...(3)
y
Elimination of h and k.
y 2
. y 2 a C
F [ F[
(2) and (3)
y
FZ
FZ
This is the required differential equation.
(ii) The equation of a parabola whose axis is parallel to the y-axis is given by
Z J C
[ M , ...(1)
where h, k and a are arbitrary constants.
(1) 2( x – h)(1 – 0) 4 a( y – 0) x – h 2ay
1 – 0 2ay 0 2ay y 0
(Q a 0)
F [ This is the required differential equation.
FZ
Self-Instructional Material 9
Ordinary Differential Example 8. Form the differential equation representing the family of ellipses
Equations having foci on the x-axis and centre at the origin.
Solution. Let the equation of the family of ellipses be
NOTES x2 y2
= 1, ...(1)
a 2 b2
where a and b are parameters and a > b > 0.
Differentiating (1) w.r.t x, we get
2x 2 yy yy b2
2
2 = 0 2 ...(2)
a b x a
Differentiating (2) w.r.t. x, we get
Z = [[ [ [ ? [[
=0 i.e., xyy + xy2 yy = 0.
Z
F [
Z F[ [ F[
xy
FZ
FZ FZ = 0.
EXERCISE B
1. Form the differential equation of the family of curves given by:
(i) y = kx + k2 + k3 (ii) y + sin x = 0.
2. Form the differential equation of all straight lines passing through the origin.
3. Form the differential equation of the family of all non-vertical lines y = mx + c, in the
xy-plane.
4. (i) Form a differential equation of the family of curves y = a sin (bx + c) where a and c
being arbitrary constants.
(ii) Form a differential equation of the family of curves y = a sin (bx + c) where a, b and c
being arbitrary constants.
5. Obtain a differential equation that should be satisfied by the family of concentric circles
x2 + y2 = a2.
6. Form a differential equation of the family of circles given by x2 + y2 = 2ax.
7. Form the differential equation of the family of curves given by:
(i) y = Ae2x + Be2x (ii) y = ax + bx2
10 Self-Instructional Material
(ii) Form the differential equation of all circles in the second quadrant and touching the Differential Equations
coordinate axes.
11. (i) Form the differential equation of the family of circles of radius 2 units and having
centre on the x-axis.
(ii) Form the differential equation of the family of circles having centre on the NOTES
y-axis and radius 3 units.
12. (i) Form the differential equation of the family of circles (x a)2 + (y b)2 = r2 by eliminating
a and b.
(ii) Form the differential equation of the family of circles having radii 3.
13. Form the differential equation of all circles in the xy-plane.
14. (i) Form the differential equation of the family of parabolas having vertex at the origin
and axis along the positive y-axis.
(ii) Form the differential equation of the family of parabolas having vertex at the origin
and axis along the positive x-axis.
15. Form the differential equation of the family of ellipses having foci on the y-axis and centre
at the origin.
16. Form the differential equation of the family of hyperbolas having foci on the x-axis and
centre at the origin.
17. Show that the differential equation of which x2 y2 = c(x2 + y2)2 is a solution is
(x3 3xy2)dx = (y3 3x2y) dy.
Answers
dy dy dy
2 3
dy
1. (i) y = x
dx
dx dx (ii)
dx
= y cot x
dy d2 y
2. y= x 3. 0
dx dx2
d2 y d3 y d2 y dy
4. (i) b2 y 0 (ii) y 0
dx2 dx3 dx2 dx
dy dy
5. xy 0 6. 2 xy x2 – y 2 0
dx dx
F [ d2 y dy
7. (i) [ (ii) x2 2
2x 2y 0
FZ dx dx
F[ F [ F[
(iii) Z [ Z[ VCP Z (iv) T FT
FZ FT
d2 y dy d2 y dy
8. (i) 2
2 2y 0 (ii) x 2 xy x 2 2
dx dx dx2 dx
d2 y dy 2
dy d2 y dy
(iii) xy
dx 2
x
dx y
dx
0 (iv)
dx 2
4
dx
4y 0
dy dy
9. (i) 2 xy x 2 y2 0 (ii) 2 xy x 2 y2 0
dx dx
dy x y dy 2 2 dy "# x y dy
2 2
dx dx
! dx #$ dx
10. (i) ( x y) 2 1 (ii) ( x y)2 1
dy 2 2
(i) y 1 4
dy
dx
11. 2 (ii) ( x 2 9) 2
x 0
dx
d y 1 dy 0
2
2 2
3
d y 1 dy
2
2 2
3
dx dx dx dx
12. (i) r 2 (ii) 9 =0
2 2
Self-Instructional Material 11
Ordinary Differential 2 2
Equations
dy d3y dy d 2 y
13. 1 3 0
dx
dx 3 dx dx 2
dy dy
14. (i) x 2y = 0 (ii) y2 2xy 0
NOTES dx dx
2 2
d2y dy dy d2 y dy dy
15. xy x y =0 16. xy x y = 0.
dx 2 dx dx dx 2 dx dx
Hints
dy dy
2. y = mx m y x .
dx dx
4. (i) y = a sin (bx + c) y1 = ab cos (bx + c) y2 = ab2 sin (bx + c) = b2y
y2 + b2y = 0.
(ii) y = a sin (bx + c) y1 = ab cos (bx + c) y2 = ab2 sin (bx + c)
y3 = ab3 cos (bx + c)
y2 y
y2 = b2y and y3 = b2y1 3 yy3 = y1y2.
y y1
8. (i) y = ex(a cos x + b sin x)
y1 = ex (a cos x + b sin x) + ex( a sin x + b cos x)
y1 = y + ex( a sin x + b cos x)
y2 = y1 + ex ( a sin x + b cos x) + ex( a cos x b sin x)
y2 = y1 + (y1 y) + ( y) y2 2y1 + 2y = 0.
(ii) xy = Aex + Bex + x2 xy1 + y = Aex Bex + 2x
(xy2 + y1) + y1 = (Aex + Bex) + 2 xy2 + 2y1 = (xy x2) + 2.
(iii) y2 = a(b x2) 2yy1 = a(2x) yy1 = ax
yy1
yy2 + y1 . y1 = a yy2 + y12 = .
x
10. (i) Let the equation of the circle be
(x a)2 + (y a)2 = a2, where a is an arbitrary constant. Y
2(x a) + 2(y a)y1 = 0 a>0
x yy1
x a + yy1 ay1 = 0 a =
1 y1
Putting the value of a in the equation of circle, we get
2 2 2 a (a, a)
x yy1 x yy1 x yy1
x y a
1 y1 1 y1 1 y1
O X
(x + xy1 x yy1)2 + (y + yy1 x yy1)2 = (x + yy1)2
y12 (x y)2 + (y x)2 = (x + yy1)2.
11. (ii) Let the equation of family of circles be
(x 0)2 + (y a)2 = 9. ...(1)
x 2
x
2x + 2(y a)y = 0 y a = (1) x2 + = 9.
y y 2
12. (i) We have (x a)2 + (y b)2 = r2.
(1)
(1) 2(x a) + 2(y b)y1 = 0
(2)
(2) 2 + 2(y b)y2 + 2(y1)y1 = 0
(3)
[
(3) yb= and
[
[ [ [ [
(2) x a = (y b)y1 = [ [
Now put the values of x a and y b in (1).
12 Self-Instructional Material
13. The equation of a circle in xy-plane is Differential Equations
x2 + y2 + 2gx + 2fy + c = 0, where g, f, c are arbitrary constants.
(1)
(1) 2x + 2yy1 + 2g + 2fy1 + 0 = 0 x + yy1 + g + fy1 = 0
(2)
(2) 1 + yy2 + y12 + 0 + fy2 = 0 2
(y + f )y2 + y1 + 1 = 0 ...(3)
NOTES
(3) (y + f )y3 + y1y2 + 2y1y2 + 0 = 0 (y + f )y3 + 3y1y2 = 0 ...(4)
Multiply (3) by y3, (4) by y2 and subtract.
14. (i) Take x2 = 4ay as the equation of the family of parabolas.
x2 y2
15. Take 2
= 1, a > b > 0 as the equation of the family of ellipse.
b a2
x2 y2
16. Take 2
= 1 as the equation of the family of hyperbolas.
a b2
17. We have x2 y2 = c(x2 + y2)2. ...(1)
(1) 2x 2yy1 = c.2(x2 + y2)(2x + 2yy1)
x yy1 = 2c(x2 + y2)(x + yy1) ...(2)
Divide (2) by (1) and simplify.
SOLVED EXAMPLES
Example 9. Show that y = bex + ce2x is a solution of y2 3y1 + 2y = 0.
Solution. We have y = bex + ce2x.
y1 = bex + 2ce2x and y2 = bex + 4ce2x
y2 3y1 + 2y = (be + 4ce ) 3(bex + 2ce2x) + 2(bex + ce2x)
x 2x
Self-Instructional Material 13
Ordinary Differential B d2y dy
Equations Example 10. Show that y = Ax + is a solution of x2 2 + x y = 0.
x dx dx
B
Solution. We have y = Ax + .
x
NOTES B
dy
= A + B(1)x2 = A 2
dx x
d 2y 3
2B
and = 0 + B(1)(2) x =
dx 2 x3
d 2y dy 2B "# B B "# "#
x2
dx 2
+x
dx
y = x2
x 3 + x A 2 Ax
! $ x ! x $ ! $
2B B B
= + xA Ax =0
x x x
B d2y dy
y = Ax + is a solution of x2 +x y = 0.
x dx 2 dx
EXERCISE C
2
dy dy
1. Show that x2 + 4y = 0 is a solution of +x y = 0.
dx dx
xy
2. Show that y = 1 x2 is a solution of y = .
1 x2
1 d2 y
3. Show that y = + Ax + B is a solution of x 3 1.
2x dx2
4. Show that y = a cos x + b sin x is a solution of y + y = 0.
d2 y dy
5. Show that y = 3 cos(log x) + 4 sin(log x) is a solution of x2 2
+x + y = 0.
dx dx
6. Show that y = ae2x
+ bex
is a solution of y2 y1 2y = 0.
7. Show that y = e3x (A + Bx) is a solution of y2 6y1 + 9y = 0.
8. Show that y = c1eax cos bx + c2eax sin bx is a solution of y2 2ay1 + (a2 + b2)y = 0.
F [ F[
9. Show that y = cos (cos x) is a solution of EQV Z [ UKP Z = 0.
FZ FZ
10. Show that x + y = tan1 y is a solution of y2y + y2 + 1 = 0.
11. Show that y = x sin x is a solution of xy = y + x x2 y2 (x 0 and x > y or x < y).
14 Self-Instructional Material
12. Show that y cos y = x is a solution of (y sin y + cos y + x)y = y. Differential Equations
dy
13. Show that x2 = 2y2 log y is a solution of (x2 + y2) xy = 0.
dx
d2 y
14. Show that y = c1ex + c2ex is the general solution of y = 0.
dx2 NOTES
15. x
Show that y = e + 1 is a solution of y y = 0.
16. Show that y = x2 + 2x + C is a solution of y 2x 2 = 0.
17. Show that y = cos x + C is a solution of y + sin x = 0.
18. Show that y = Ax is a solution of xy = y, x 0.
d2 y
19. Show that y = aex + bex + x2 is a solution of y x 2 2 = 0.
dx2
d2 y
dy
20. Show that y = ex (a cos x + b sin x) is solution of 2 2 y = 0.
dx2 dx
21. Show that x2 y2 = c(x2 + y2)2 is a solution of (x3 3xy2)dx = (y3 3x2y)dy.
D
22. Can y = ax + be a solution of the following differential equation
C
F[ D
y= Z ?
FZ F[
FZ
If no, find the solution of the given differential equation. (CBSE 2018 SP)
Answer
22. Yes.
Hints
1 1 1
3. y= + Ax + B y1 = +A y2 = ( 2) x3 x3y2 = 1.
2x 2 x2 2
UKP
NQI Z EQU
NQI Z
5. y = 3 cos (log x) + 4 sin (log x) y1 =
Z Z
xy1 = 3 sin (log x) + 4 cos (log x)
EQU
NQI Z UKP
NQI Z [
xy2 + 1 y1 = .
Z Z Z
9. y = cos (cos x) y1 = sin (cos x) sin x = sin xsin (cos x)
y2 = cos x sin (cos x) + sin x cos (cos x) sin x
y
y2 = (cos x) 1 (sin 2 x) y .
sin x
1
10. x + y = tan1 y 1 + y= . y (1 + y2)(1 + y) = y
1 y2
1 + y2 + y2y = 0.
11. y = x sin x y = sin x + x cos x
y2
xy = x sin x + x2 cos x = y + x2 1 sin 2 x = y + x2 1 = y + x x2 y2 .
x2
12. y cos y = x y + (sin y)y = 1 ...(1)
Also, (y sin y + cos y + x) y = (y sin y + y x + x) y
= (y sin y + y) y = y(sin y + 1) y = y(1) = y.(By using (1))
2 y2
13. x2 = 2y2 log y 2x = 4yy log y + y x = (2y log y + y) y
y
xy = (2y2 log y + y2) y xy = (x2 + y2) y
Self-Instructional Material 15
Ordinary Differential
Equations INITIAL VALUE PROBLEM
SOLVED EXAMPLES
Z
Example 12. Show that y = 2 is a solution of the initial value problem:
Z
F[
F[
yx
FZ
= 2 Z
FZ
, y(1) = 1.
Z
Solution. We have y = 2 . ...(1)
Z
x=1 y=2 = 2 1 = 1 y(1) = 1
F[ = 0
Z Z
=
(1)
FZ
Z
Z
F[ = 2 Z x = 2 Z + Z
yx
FZ Z
Z
Z
Z
Z Z
Z Z Z Z
=
Z
Z
F[
Z
and
Z
FZ
Z
Z Z Z Z
Z
Z
=
F[
= Z
F[
FZ
yx
FZ
Z
y=2
Z
is a solution of the given initial value problem.
Example 13. Show that y = x sin 3x is a solution of the initial value problem:
F [
+ 9y 6 cos 3x = 0, y(0) = 0.
FZ
Solution. We have y = x sin 3x. ...(1)
x=0 y = 0 sin 3(0) = 0 y(0) = 0
(1)
F[ = x 3 cos 3x + 1 sin 3x = 3x cos 3x + sin 3x
FZ
16 Self-Instructional Material
F [ Differential Equations
= (3x (3 sin 3x) + 3.1. cos 3x) + 3 cos 3x
FZ
= 9x sin 3x + 6 cos 3x
F [ NOTES
+ 9y 6 cos 3x = 9x sin 3x + 6 cos 3x + 9(x sin 3x) 6 cos 3x = 0
FZ
y = x sin 3x is a solution of the given initial value problem.
EXERCISE D
1. Show that y = ex is a solution of the initial value problem:
F[
= y, y(0) = 1.
FZ
2. Show that y = sin x + cos x is a solution of the initial value problem:
F [ + y = 0, y(0) = 1, y(0) = 1.
FZ
3. Show that y = xex + ex is a solution of the initial value problem:
F [ F[
2 + y = 0, y(0) = 1, y(0) = 2.
FZ FZ
4. Show that 3x2y = 2x + y is a solution of the initial value problem:
x2 dy + (xy + y2) dx = 0, y(1) = 1.
5. Show that 2y = x(x + y) is a solution of the initial value problem:
x2
F[ = y2 + 2xy, y(1) = 1.
FZ
dy
Let us consider the differential equation = f(x) g(y), ...(1)
dx
where f(x) and g(y) are some functions of x and y respectively.
We know that
F[
dy = dx,
FZ
where dx and dy are respectively the differentials of the variables x and y.
(1) dy = f (x) g(y) dx
dy
= f(x) dx, provided g(y) 0 ...(2)
g ( y)
In equation (2), the expressions involving y are on one side and the expressions
involving x are on the other side.
Such a differential equation is said to be with variables separable.
Integrating equation (3), we get
I F[
I
[
= I f(x) dx + C, where C is an arbitrary constant.
Self-Instructional Material 17
Ordinary Differential
Equations
G\
Working Steps for Solving = f(x) g(y)
G[
Step I. Identify the functions f(x) and g(y).
NOTES Step II. Bring expressions involving x on one side and expressions involving y
on the other side. Always keep dx and dy in the numerators.
Step III. Integrate both sides and add arbitrary constant C only on one side.
This gives the required general solution.
Step IV. If some initial condition is given, then find the value of the arbitrary
constant C, so that the initial condition is satisfied. Put the value of
C in the general solution to get the required particular solution.
dy
Tpye I. Solution of = f(x)
dx
If g(y) = 1, then
F[ = f(x) g(y) reduces to F[ = f(x).
FZ FZ
SOLVED EXAMPLES
dy 1 cos x
(i) (ex + ex)dy (ex ex)dx = 0 (ii) .
dx 1 cos x
Solution. (i) We have (ex + ex) dy = (ex ex) dx.
e x ex
dy = dx (Variables are separate)
e x ex
Integrating, we get I I
1dy =
e x ex
e x ex
dx + C.
y = log|ex + ex| + C
Integrating, we get I I
1dy =
1 cos x
1 cos x
dx + C.
I
x
2 sin2
y= 2 dx + C
x
2 cos2
2
I
y= sec 2 x
2
1 dx + C y = 2 tan
Z x + C.
18 Self-Instructional Material
Example 15. Solve the differential equations: Differential Equations
F[ F[
(i) (1 + x2) x = 2 tan1 x (ii) cos x cos 2x = cos 3x.
FZ FZ
F[ NOTES
Solution. (i) We have (1 + x2)
x = 2 tan1 x.
FZ
F[ Z VCP Z
FZ Z
Z VCP Z
F[
Z
Z
dx (Variables are separate)
Z
dy =
dx
Z
VCP
Z
Z
Integrating, we get
I F[
I Z
Z
FZ I VCP
Z
Z
FZ + C.
VCP Z
y= log |1 + x2| + 2 +C
y= log (1 + x2) + (tan1 x)2 + C.
F[
(ii) We have cos x cos 2x = cos 3x.
FZ
F[ EQU Z EQU Z
FZ EQU Z
EQU Z EQU Z EQU Z
dy = dx (Variables are separate)
EQU Z
EQU Z
dy = EQU Z EQU Z
dx.
Self-Instructional Material 19
Ordinary Differential Integrating, we get
I I I
Equations
2 x2 x 2x 2 x
dy dx + C y= dx + C
x3 x2 x 1 ( x 1)( x 2 1)
I
1 3 1
NOTES
y= 2 2
x
2 dx + C
x1 x2 1
(By resolving into partial fractions)
1 3 1
y= log |x + 1| + log (x2 + 1) tan1 x + C
2 4 2
Also, y = 1 when x = 0 ...(2)
1 3 1
(1) 1= log |1| + log (1) (0) + C C=1
2 4 2
The required solution is
1 3 1
y= log |x + 1| + log (x2 + 1) tan1 x + 1.
2 4 2
EXERCISE E
Find the general solution of the following differential equations (Q. No. 13):
dy dy x
1. (i) x2 =2 (ii)
dx dx x2 1
dy dy 1 cos 4 x
(iii) = x2 + sin 3x (iv)
dx dx 1 cos 4 x
dy
2. (i) (x + 2) = x2 + 4x 9 (ii) 1 x6 dy = x2 dx
dx
dy F[
(iii) = log (x + 1) (iv)
dx FZ UKP Z EQU Z
F[ = x5 tan1 x3 F[
3. (i) (ii) = sin3 x cos2 x + x ex
FZ FZ
F[ F[ = sin 1 x
(iii) = tan 1 x (iv)
Z FZ FZ
4. Solve the following initial value problems:
dy
(i) x + 1 = 0, y(1) = 0 (ii) edy/dx = x + 1, y(0) = 5
dx
dy dy
(iii) x(x2 1) = 1, y(2) = 0 (iv) sin = k, y(0) = 1.
dx dx
Answers
2 1
1. (i) y + =C (ii) y = log (x2 + 1) + C
x 2
x3 cos 3 x 1
(iii) y = +C (iv) y = tan 2x x + C
3 3 2
x2 1
2. (i) y = + 2x 13 log | x + 2 | + C (ii) y = sin 1 x3 + C
2 3
20 Self-Instructional Material
1 tan x cot x Differential Equations
(iii) y = (x + 1) log (x + 1) x + C (iv) y = tan 1 +C
2 2
3. (i) 6y = (x6 + 1) tan1 x3 x3 + C (ii) y = cos5 x cos3 x + (x 1) ex + C
2 NOTES
(iii) y = (x + 1) tan1 x x + C (iv) y = x sin1 x + Z + C
4. (i) y + log |x| = 0 (ii) y = (x + 1) log (x + 1) x + 5
Z
(iii) y = log (iv) y = x sin1 k + 1.
Z
dy
TYPE II. Solution of = g(y)
dx
If f(x) = 1, then
F[ = f(x) g(y) reduces to F[ = g(y)
FZ FZ
SOLVED EXAMPLES
I I
(Variables are separate)
y
Integrating, we get dy dx + C.
1 y2
1
2 I 2y
1 y 2
dy = x + C
1
2
log (1 + y2) + x = C.
dy 1
(ii) We have .
dx log y
I I
log y dy = dx (Variables are separate)
Integrating, we get log y dy = dx + C.
I log y 1 dy = x + C (log y) y
I 1
y
y dy = x + C
y log y y = x + C.
Self-Instructional Material 21
I I
Ordinary Differential Integrating, we get
Equations
sec2 y dy = dx + C tan y = x + C.
dy 1 cos y
NOTES (ii) We have = 0.
dx 1 cos y
dy 1 cos y 1 cos y
dy = dx
dx 1 cos y 1 cos y
(Variables are separate)
Integrating, we get
I 1 cos y
1 cos y
dy I dx + C.
I I
y
2 sin 2
2 dy = x + C y
2 cos 2 y
sec 2
2
1 dy = x + C
2
y [
2 tan y=x+C x y + 2 tan = C.
2
EXERCISE F
Find the general solution of the following differential equations (Q. No. 13):
dy dy 1
1. (i) + y = 1, y 1 (ii) 2
dx dx y sin y
dy 1 y2 dy
(iii) (iv) = [
dx y3 dx
FZ
2. (i) y dy = (ii) (sin y cos y)dy = (sin y + cos y)dx
VCP [
(iii) (y5 tan 1 y3)dy = dx (iv) y log y dy = dx
F[ F[
3. (i) = sec y (ii) = sin2 y
FZ FZ
(iii)
F[ =
EQU [
(iv) (2y2 + y)
F[ = y3 + y2 + y + 1
FZ EQU [ FZ
4. Solve the following initial value problems:
dy dy
(i) + 2y2 = 0, y(1) = 1 (ii) + cos2 y = 0, y(0) = .
dx dx
Answers
y3
1. (i) log 1 y + x + C = 0 (ii) x = cos y + C
3
y2 1
(iii) x = log (y2 + 1) + C (iv) y = 2 sin (x + C)
2 2
1 2 [
2. (i) x = (y + 1) tan 1 y +C (ii) x + log sin y + cos y = C
2
(iii) 6x = (y6 + 1) tan 1 y3 y3 + C (iv) 4x = 2y2 log y y2 + C
22 Self-Instructional Material
3. (i) x = sin y + C (ii) x + cot y = C Differential Equations
(iii) x + cot y + y = C
(iv) x = log y + 1 + log (y2 + 1) tan1 y + C
NOTES
4. (i) y = (ii) x + tan y = 1.
Z
dy
TYPE III. Solution of = f(x) g(y)
dx
SOLVED EXAMPLES
Integrating, we get
I 1+ x
x
dx = I y2 1
y
dy + C.
I 1
x
1 dx =
I y
1
y
dy + C
y2
log |x| + x = log |y| + C
2
[
log |xy| + x = + C.
(ii) We have (y3 + 1)(ex + xex)dx xexy2 dy = 0.
(y3 + 1)ex (1 + x)dx = xexy2 dy
2
1+ x y
dx = 3 dy (Variables are separate)
x y 1
1
log |x| + x = log |y3 + 1| + C.
3
Example 20. Solve the differential equations:
(i) sec2 x tan y dx + sec2 y tan x dy = 0
(ii) ex tan y dx + (1 ex) sec2 y dy = 0.
Solution. (i) We have sec2 x tan y dx + sec2 y tan x dy = 0.
sec2 x tan y dx = sec2 y tan x dy
Self-Instructional Material 23
Ordinary Differential
UGE Z UGE [
Equations dx = dy (Variables are separate)
VCP Z VCP [
NOTES
Integrating, we get I sec2 x
tan x
dx = I sec2 y
tan y
dy + C.
Integrating, we get I ex
ex 1
dx = I UGE [
VCP [
dy + C.
1 x2 y
dx + dy = 0
x 1 y2
(Variables are separate)
Integrating, we get I 1 x2
x
dx + I y
1 y2
dy = C. ...(1)
1 x2 = z 1 + x2 = z2 2x dx = 2z dz
I 1 x2
x
dx = I 1 x2
x2
x dx =
I z
z2 1
. z dz
=
I z2
z2 1
dz = I ( z 2 1) 1
z2 1
dz
= I! 1
1
( z 1)( z 1)
"#
$
dz = z + I !
"#
$
\
\
dz
1 1 z1
=z+ (log |z 1| log |z + 1|) = z + log
2 2 z1
24 Self-Instructional Material
Differential Equations
1 1 x2 1 1 1 x2 1
= 1 x2 + log = 1 x2 + log
2 1 x2 1 2 1 x2 1
Also, 1 y2 = z 1 + y2 = z2 2y dy = 2z dz
I I I
NOTES
y \ F\
dy = = 1 dz = z = 1 y 2
1 y2 \
1 Z
(1) Z + 2 log + [ = C.
Z
dy = 3x + 4y.
(ii) We have log dx
dy dy
= e3x+4y = e3x e4y
dx dx
I I
e4y dy = e3x dx (Variables are separate)
Integrating, we get e4y dy = e3x dx + C.
Z
G [ = G + C.
Example 22. Show that the general solution of the differential equation
dy y 2 y 1
= 0 is given by (x + y + 1) = A(1 x y 2xy), where A is a parameter.
dx x 2 x 1
dy y 2 y 1 dy dx
Solution We have = 0, i.e., + = 0.
dx x 2 x 1 y2 y 1 2
x x1
(Variables are separate)
Integrating, we get I 2
y y1
dy
+ I 2
x x1
dx
= C. ...(1)
I 1
dy
3
2 2
+
I 1
dx
3
2 2
=C
y
2 2 x
2 2
1 [ 1 Z
tan1 + tan1 = C
3 /2 3 /2
2 [ Z
VCP VCP = C
3
Z 1 [ = 3
tan1 + tan C = C1, say
2
Z [
tan1 = C
3 (2 x 1 2 y 1)
= tan C1
Z [
1
3 (2 x 1)(2 y 1)
Self-Instructional Material 25
Ordinary Differential
2 3 ( x y 1) 3 ( x y 1) = tan C
Equations = tan C1 1
2 4 xy 2 x 2 y 1 x y 2 xy
x + y + 1 = tan C1 (1 x y 2xy)
NOTES 3
x + y + 1 = A(1 x y 2xy),
tan C1
where A = is a parameter.
3
Example 23. Solve the following differential equations:
F[ Z
NQI Z
(i) yex/y dx = (xex/y + y2)dy, y 0 (ii) .
FZ UKP [ [ EQU [
dx x x/y
Solution. (i) We have yex/y dx = (xex/y + y2)dy i.e., ex/y = e + y. ...(1)
dy y
x dx
Put z= x = zy and = z + y dz
y dy dy
dz dz dz
(1)
ez z y
dy
= zez + y ezy
dy
= y ez
dy
=1
Integrating, we get
I e z dz = I
ez = y + C
dy C .
ex/y = y + C.
F[ Z
NQI Z
(ii) We have =
FZ UKP [ [ EQU [
(sin y + y cos y) dy = 2x (log x + 1) dx
UKP [ [ EQU [F[ = Z NQI Z FZ Z FZ %
UKP [ F[ + [ EQU [ F[ = NQI Z Z FZ Z %
+ ++ + ++
Z Z
cos y + [ UKP [ UKP [ F[ = 2
NQI Z FZ + x2 + C
Z
Z
cos y + y sin y + cos y = x2 log x + x2 + C
Z
y sin y = x2 log x + + C.
Example 24. Solve the following initial value problems:
(i) y = y tan 2x, y(0) = 2 (ii) 2xy = 3y, y(1) = 4.
Solution. (i) We have y = y tan 2x.
dy dy
= y tan 2x = tan 2x dx
dx y
dy
y I I
= tan 2x dx + C log y =
log|sec 0|
log|sec 2x |
2
+ C.
Idx
x
dy 3
I 3
+ C log y = log x + C.
3
y
=
2 2
I I
cot y dy + cos x dx = C log |sin y| + sin x = C
I I
dx ey
ey dy = cos x dx + C e y = sin x + C.
Now y(0) = 0 implies e0 = sin 0 + C or 1 = 0 + C or C = 1
The required particular solution is ey = sin x + 1.
Self-Instructional Material 27
Ordinary Differential Integrating, we get
Equations
Z
tan1 y = x + + C. ...(1)
Also, y = 1 when x = 0
NOTES
(1) tan 1 (1) = 0 + 0 + C C = /4
Z
(1) tan 1 y = x + + .
(ii) We have (x y)(dx + dy) = dx dy.
dx dy d ( x y)
dx + dy = d(x + y) =
x y x y
Integrating, we get
I
d( x y)
Also, y = 1 when x = 0
I
d( x y)
x y
C x + y = log |x y| + C ...(1)
EXERCISE G
Find the general solution of the following differential equations (Q. No. 12):
dy 4y dy 1 y2
1. (i) = (ii) + =0
dx x( y 2) dx 1 x2
dy F[ [
(iii) = (ex + 1)y (iv) Z G
dx FZ
dy dy x 1
2. (i) x5 = y5 (ii)
dx dx 2 y
dy 1 y2
(iii) (iv) y log y dx x dy = 0.
dx 1 x2
Find the general solution of the following differential equations (Q. No. 38):
2 dy
3. (i) y(1 x ) x (1 y 2 ) (ii) (1 + x)y dx + (1 y)x dy = 0
dx
dy dy F[ .
(iii) y a = ay2 + x (iv) cosec x log y Z [
dx dx FZ
4. (i) (1 + x)(1 + y2)dx + (1 + y)(1 + x2)dy = 0
(ii) (x2 yx2) dy + (y2 + x2y2)dx = 0
dy dy
(iii) y x = a y2 (iv) x 1 y 2 dx + y 1 x 2 dy = 0
dx dx
dy
5. (i) = x + y + xy + 1 (ii) (1 ex) sec2 y dy 2ex tan y dx = 0
dx
dy
(iii) (1 + y2)(1 + log x)dx + x dy = 0 (iv) cos x cos y + sin x sin y = 0
dx
6. (i) (ex + 1)y dy = (y + 1)ex dx 2
(ii) tan y dx + sec y tan x dy = 0
dy
(iii) 1 x2 dy + 1 y 2 dx = 0 (iv) sin3 x = sin y
dx
28 Self-Instructional Material
dy Differential Equations
7. (i) e x y e x y (ii) (xy2 + 2x)dx + (x2y + 2y)dy = 0
dx
dy dy
(iii) = (cos2 x sin2 x)cos2 y (iv) xy = 1 + x + y + xy
dx dx
dy NOTES
8. (i) y(1 + ex)dy = (y + 1)ex dx (ii) = y2 tan 2x
dx
y
(iii) ex 1 y 2 dx + dy = 0 (iv) (1 + e2x)dy + (1 + y2)exdx = 0
x
Solve the following initial value problems (Q. No. 912):
9. (i) y = 2exy3, y(0) = 1/2 (ii) y = 4xy2, y(0) = 1
(iii) xyy = y + 2, y(2) = 0 (iv) y = y cot 2x, y(/4) = 2
10. (i) 2 2
x(1 + y )dx y(1 + x )dy = 0, y(0) = 1
(ii) (1 + y2)(1 + log x)dx + x dy = 0, y(1) = 1
(iii) (1 y2)(1 + log x)dx + 2xy dy = 0, y(1) = 0
(iv) sec2 y (1 + x2)dy + 2x tan y dx = 0, y(1) = /4
UKP Z F[ dy
11. (i) EQU Z [
(ii) (x + 1) = 2ey 1, y(0) = 0.
[ FZ dx
F[
(iii) (1 + e2x)dy + (1 + y2)ex dx = 0, y(0) = 1 (iv) Z
Z = 1, y(2) = 0
FZ
dy dy
12. (i) = y tan x, y(0) = 1 (ii) log = 3x + 4y, y(0) = 0
dx dx
(iii) xy
F[ = (x + 2)(y + 2), y(1) = 1 (iv) (x2 yx2)dy + (y2 + x2 y2) dx = 0, y(1) = 1
FZ
Answers
1. (i) y = log (x4y2) + C (ii)sin1 x + sin1 y = C
(iii) log |y| = ex + x + C (iv) 2e2y = x4 + C
2. (i) x4 + y4 = C (ii)x2 + y2 + 2x 4y + C = 0
(iii) tan1 y = tan1 x + C (iv) y = eCx
3. (i) (1 x2)(1 + y2) = C (ii)log |xy| + x y = C
NQI [
(iii) (a + x)(1 ay) = Cy (iv) + (2 x2) cos x + 2x sin x = C
[
1
4. (i) tan1 x + tan1 y + log (1 + x2)(1 + y2) = C
2
1 1
(ii) x log| y|– =C (iii) (x + a)(1 ay) = Cy
x y
(iv) 1 x2 + 1 y2 = C
1 2
5. (i) log |y + 1| = x +x+C (ii) tan y = C(1 ex)2
2
1
(iii) (1 + log x)2 + tan1 y = C (iv) sin y = C cos x
2
6. (i) y log |y + 1| = log (ex + 1) + C (ii) sin x tan y = C
1
(iii) (x + 1 x 2 )(y + 1 y2 ) = C (iv) cos y + cos3 x cos x = C
3
7. (i) ex
= +Cey ex (ii) (x2 + 2)(y2 + 2) = C
1
(iii) tan y = sin 2x + C (iv) y = x + log |x(1 + y)| + C
2
1 1
8. (i) y log |1 + y| = log C(1 + ex) (ii) = log |sec 2x| + C
y 2
(iii) (x 1)ex 1 y2 = C (iv) y + ex = C(1 yex)
Self-Instructional Material 29
Ordinary Differential 1
Equations 9. (i) y2(8 4ex) = 1 (ii) y =
2 x2 1
| x|
(iii) y = 2 log |y + 2| + log (iv) y2 = 4 sin 2x
8
NOTES 10. (i) y2 2x2 = 1 (ii) 2(1 + log x)2 + 4 tan1 y = 2 +
(iii) (1 + log x)2 = 2 log |1 y2| + 1 (iv) (1 + x2) tan y = 2
11. (i) (2 + sin x)(1 + y) = 4 (ii) (x + 1)(2 ey) = 1
1 4| x 2 1|
(iii) tan1 y + tan1 ex = /2 (iv) y = log
2 3x2
12. (i) y = sec x (ii) 4e3x + 3e4y = 7
(iii) y x + 2 = 2 log x(y + 2) (iv) x = x1 + y1 + log y 1.
dy
Solution of = f(ax + by + c) by the Method of Separation of Variables
dx
dy
Consider the differential equation = f (ax + by + c), ...(1)
dx
where f (ax + by + c) is some function of ax + by + c.
dz
dz dy dy a
Let z = ax + by + c. =a+b or = dx
dx dx dx b
dz
a
dx dz
(1) = f(z) = bf(z) + a
b dx
dz
= dx ...(2)
bf ( z ) a
In the differential equation (2), the variables x and z are separated.
Integrating (2), we get
I dz
bf ( z ) a
=
I 1dx + C.
I F\
DH
\ C
= x + C, where z = ax + by + c.
G\
Working Steps for Solving = f(ax + by + c)
G[
Step I. Identify the function f(ax + by + c).
Step II. Put z = ax + by + c and differentiate it w.r.t. x. Solve this to find the
dy
value of .
dx
dy
Step III. Put the values of and ax + by + c in the given differential equation.
dx
Separate the variables z and x and integrate both sides.
Step IV. Replace the value of z. This gives the general solution of the given
differential equation.
30 Self-Instructional Material
Differential Equations
SOLVED EXAMPLES
I
dz =
EQU \ I FZ + C.
z
I
EQU
\
dz = x + C
z
I UGE
\
dz = x + C
\
VCP Z [
z = x + C x + y tan =x+C
Z [
y = tan + C.
F[
(ii) We have cos2 (x 2y) = 1 2 .
FZ
F[ = EQU
Z [ = UKP
Z [
FZ
EQU
Z [
=
(1 cos (2x 4y))
F[ = (1 cos (2x 4y)) ...(1)
FZ
RHS of (1) is a function of 2x 4y.
F\ F[ F[ F\
z = 2x 4y
FZ
=24
FZ
FZ
=
FZ
F\
(1)
FZ
=
(1 cos z)
Self-Instructional Material 31
Ordinary Differential F\
F\
Equations 2 = 1 cos z = 1 + cos z
FZ FZ
F\ = dx (Variables are separate)
NOTES EQU \
Integrating, we get
I F\
EQU \
= I FZ + C.
I F\
EQU \
=x+C
I UGE
\
dz = x + C
\
VCP Z [
=x+C tan =x+C
tan (x 2y) = x + C.
F\ \ F\ \
= +1 =
FZ \ FZ \
\
dz = dx (Variables are separate)
\
\
Integrating, we get \ dz = FZ %
\
\
dz = x + C
\ dz = x + C
\ NQI \ = x + C 3z + 4 log 3z 1 = 9x + 9C
3(x + 2y) + 4 log 3(x + 2y) 1 = 9x + C1, where C1 = 9C
6(y x) + 4 log 3x + 6y 1 = C1.
F[ Z [ F[
Z [
(ii) We have = i.e., . ...(1)
FZ [ Z FZ
Z [
RHS of (1) is a function of 2x y.
32 Self-Instructional Material
F\ F[ F[ F\ Differential Equations
Let z = 2x y.=2 or =2
FZ FZ FZ FZ
F\ \ F\ \ \
(1) 2 = =2
FZ \ FZ \ \ NOTES
F\ \ \
= dz = dx (Variables are separate)
FZ \ \
\
Integrating, we get \
dz = FZ + C.
\
dz = x + C
z
log z = x + C
2z log z = 5x + 5C
2(2x y) log 2x y = 5x + C1, where C1 = 5C
x + 2y + log 2x y + C1 = 0.
EXERCISE H
Find the general solution of the following differential equations (Q. No. 14):
dy 2 dy
1. (i) = (ii) (x + y + 1) =1
dx x + 2y – 3 dx
dy dy
2. (i) = (3x + 2y + 1)2 (ii) (x + y)2 = k2
dx dx
F[ Z [ F[ Z [
3. (i) = (ii)
FZ Z [ FZ Z [
F[ = Z [ F[ Z [
4. (i) (ii) .
FZ Z [ FZ Z [
5. Solve the following initial value problems:
(i) (x + y + 1)2 dy = dx, y(1) = 0 (ii) cos (x + y)dy = dx, y(0) = 0.
Answers
1. (i) 2y = 4 log x + 2y + 1 + C (ii) y = log x + y + 2 + C
F[ H
Z [
F[ Z P (
[ Z = (
[ Z = (y/x), say
becomes
FZ I
Z [ FZ Z P )
[ Z )
[ Z
A homogeneous differential equation can also be expressed as
F[ = (y/x).
FZ
Illustration. Let
F[ Z [ ...(1)
FZ Z[ [
(1) is a homogeneous differential equation, because x3 2y3 and xy2 + 7y3 are
homogeneous functions of degree 3 each.
(1) can also be expressed as
F[
[ Z
.
FZ
[ Z
[ Z
Solution of Homogeneous Differential Equation
F[ H
Z [
Let ...(1)
FZ I
Z [
be a homogeneous differential equation.
f (x, y) and g(x, y) are homogeneous functions of same degree, say, n.
Let f (x, y) = xn F(y/x) and g(x, y) = xn G(y/x)
for some functions F(y/x) and G(y/x) of y/x.
H
Z [
P
Z (
[ Z (
[ Z = (y/x), say
P =
I
Z [ Z )
[ Z )
[ Z
34 Self-Instructional Material
(1)
F[ = (y/x) ...(2)
Differential Equations
FZ
F[ FX = v + x FX
Let y = vx. = v(1) + x
FZ FZ FZ
FX FX FZ NOTES
(2) v + x = (v) (Variables are separate)
X X Z
I I
FZ
FX FZ + C.
Integrating both sides, we get
X X Z
FX [
X
X
= log | x | + C, where v =
Z
.
This equation is solved and v is replaced by y/x.
G\ \
Working Steps for Solving
G[
=
[
Step I. Make sure that R.H.S. is either a function of y/x or the quotient of two
homogeneous functions of same degree.
dy dv
Step II. Put y = vx and differentiate it w.r.t. x to get =v+x .
dx dx
dy
Step III. Put the values of and y in the given differential equation. Separate
dx
the variables v and x and integrate both sides.
Step IV. Replace the value of v. This gives the general solution of the given
differential equation.
SOLVED EXAMPLES
x y
Example 29. Solve: y = .
x
dy xy
Solution. We have = . ...(1)
dx x
dy dv
This is a homogeneous differential equation. y = vx =v+x
dx dx
dv x vx
(1) v x 1 v
dx x
dv dx
x 1 dv (Variables are separate)
dx x
Self-Instructional Material 35
Ordinary Differential
Equations dv x 2 x (vx ) (vx )2
(1) v+x = = 1 + v + v2
dx x2
dv dv dx
NOTES x = 1 + v2 = (Variables are separate)
dx 1 v2 x
Integrating, we get I dv
1 v2
= I dx
x
+C
[
tan1 v = log |x| + C tan1 = log |x| + C.
Z
F[
Example 31. Solve: 2xy = x2 + y2.
FZ
F[ Z [
Solution. We have . ...(1)
FZ Z[
F[ FX
This is a homogeneous differential equation. y = vx =v+x
FZ FZ
(1) v+x
FX = Z
XZ = X
FZ Z
XZ X
FX X X
x = v=
FZ X X
X FZ
dv = Z (Variables are separate)
X
X FZ
dv + =0
X Z
log v2 1 + log x = log C
log x(v2 1) = log C
[
x(v2 1) = ± C x = C, where C = C
Z
y2 x2 = Cx.
Example 32. Solve: x dy y dx = x 2 y 2 dx .
dy dy y y 2
Solution. We have x
dx
y= x 2 y2 .
dx
=
x
+ 1
x ...(1)
dy dv
This is a homogeneous differential equation. y = vx =v+x
dx dx
dv dv dx
(1) v+x = v + 1 v2 =
dx 1v 2 x
(Variables are separate)
I dv
1v 2
= I dx
x
+ log C
36 Self-Instructional Material
Differential Equations
y y2 y x2 y2
log 1 2 = log (C|x|) = C|x|
x x x
Integrating, we get I dv
v(1 v)
= I dx
x
+ log C.
I
X X
dv = log |x| + log C
I I
dx x
dx
Integrating, we get cosec v dv = + log C.
x
v
log tan = log |x| + log C
2
y
log tan = log C|x|
2x
Self-Instructional Material 37
Ordinary Differential y y
Equations tan = C|x| tan = Cx
2x 2x
[
tan = C1x. (Putting C1 = C)
NOTES Z
dy y
Example 35. Solve: x = y x tan .
dx x
dy y
Solution. We have x = y x tan .
dx x
dy y y
= tan ...(1)
dx x x
dy dv
This is a homogeneous differential equation. y = vx =v+x
dx dx
dv
(1) v+x = v tan v
dx
dv dx
x = tan v cot v dv =
dx x
(Variables are separate)
C
log |sin v| = log |x| + log C log |sin v| = log
|x |
C y
|sin v| = |x| sin =C
|x | x
y y
x sin =C x sin =C
x x
[
x sin = C1. (Putting C1 = ± C)
Z
Example 36. Solve: (x3 3xy2)dx = (y3 3x2y)dy.
dy x 3 3xy 2
Solution. We have (x3 3xy2)dx = (y3 3x2y)dy i.e., = 3 . ...(1)
dx y 3x 2 y
dy dv
This is a homogeneous differential equation. y = vx =v+x
dx dx
dv x 3 3x (vx )2 x 3 3v2 x3 1 3v2
(1) v+x = 3 2 = 3 3 3 = 3
dx (vx ) 3x (vx ) v x 3vx v 3v
v3 3v dx
dv = (Variables are separate)
1 v4 x
Integrating, we get I v3 3v
1 v4
dv = I dx
x
+ log C.
I v3
1 v4
dv 3 I v
1 v4
dv = log |x| + log C
38 Self-Instructional Material
1
4 I X
X
3
dv 2 I 2v
1 – v4
dv = log C|x|
Differential Equations
1
4
log |1 v4|
3
2 I dt
1 t2
= log C|x|, where t = v2
NOTES
1 3 1 1 t
log |1 v4| log = log C|x|
4 2 2(1) 1 t
1 3 1 v2
log |1 v4| log = log C x
4 4 1 v2
(1 v2 )3 (1 v2 )4
log (1 v4 ) . = log (C|x|) 4 =
(1 v ) 2 3 2 2
(1 v ) %
Z
1 y
2
4
y 2
2
C4x4 (1 + v2)4 = (1 v2)2 C4x4 x 2
= 1
x 2
( x 2 y 2 )4 ( x 2 y2 ) 2
C4x4 . =
x 8 x4
C1(x2 + y2)4 = (x2 y2)2, where C1 = C4.
I
v cos v x
Integrating, we get
I v sin v cos v
dv = 2
dx
x
+ log C.
I
v cos v
cos v v sin v
dv = 2 log |x| + log C
v cos v
log |v cos v| = log Cx2
1 x
= Cx2 = Cx2
v cos v y cos ( y/ x)
y = 1 [ = C , where C 1
xy cos
x C xy cos
Z 1 1 =
C
.
Self-Instructional Material 39
Ordinary Differential Example 38. Check whether the following differential equation is homogeneous
Equations
F[ [
or not: x2 xy = 1 + cos , x 0?
FZ Z
NOTES Find the general solution of the differential equation using substitution y = vx.
Solution. We have
F[ [
x2 xy = 1 + cos , x 0.
FZ Z
[
F[ Z[ EQU Z
=
FZ Z
F[ [ [
EQU
= ...(1)
FZ Z Z Z
This is not a homogeneous differential equation, because RHS is not a function
of y/x.
F[ FX
Let y = vx =v+x
FZ FZ
FX
(1) v+x = v + (1 + cos v)
FZ Z
FX EQU X FX FZ
FZ = Z EQU X =
Z
(Variables are separate)
Integrating, we get
FX FZ
EQU X = Z + C.
EQU X Z
UKP X
dv =
+C
EQUGE X EQV X EQUGE X dv =
Z
+C
cot v + cosec v = +C
Z
[ [
cosec cot + = C.
Z Z Z
y y dx + x dy = 0 given that y = when x = 1.
Example 39. Solve: x sin 2
x
4
y
Solution. We have x sin y dx + x dy = 0.
x
2
y x sin
y
x i.e., dy = y sin y
2
dy
x
= 2 ...(1)
dx x dx x
40 Self-Instructional Material
dy dv Differential Equations
This is a homogeneous differential equation. y = vx =v+x
dx dx
dv vx vx
(1) v+x
dx
=
x
sin 2
x NOTES
dv dx
x = sin2 v cosec2 v dv =
dx x
(Variables are separate)
Integrating, we get I dx
cosec 2 v dv =
x
C I
cot v = log |x| + C
log |x| cot (y/x) = C ...(2)
Now, y = /4 when x = 1.
(2) 1
log |1| cot ( / 4) 1 = C 6 C = 0 1 = 1
(2) log |x| cot (y/x) = 1. This is the required solution.
Example 40. Solve: x2 dy + (xy + y2)dx = 0, given that y = 1 when x = 1.
dy y( x y )
Solution. We have x2 dy + y(x + y) dx = 0. = ...(1)
dx x2
dy dv
This is a homogeneous differential equation. y = vx =v+x
dx dx
dv vx ( x vx )
(1) v+x = = v(1 + v)
dx x2
dv
x = v v2 v = v(2 + v)
dx
dv dx
= (Variables are separate)
v( 2 v) x
Integrating, we get I dv
v( 2 v)
=
I dx
x
+ log C.
I
X
X
dv = log |x| + log C
1 1 C
log |v| log |2 + v| = log
2 2 |x |
1 v C v C
log = log log = 2 log
2 2v |x | 2v |x |
v C2 y/ x C2
= 2 =
2v x 2 y/ x x2
y C2
2
2x y x
x2y = k(2x + y), where k = ±C2
Now, y = 1 when x = 1. (1)2 (1) = k(2(1) + 1) i.e., k = 1/3
1
The required solution is x2y = (2x + y) or 3x2y = 2x + y.
3
Self-Instructional Material 41
Ordinary Differential
dy x(2y x)
Equations Example 41. Solve: , y(1) = 1.
dx x(2y x)
dy x(2 y x) dy 2 y x
Solution. We have i.e., . ...(1)
NOTES dx x(2 y x) dx 2 y x
dy dv
This is a homogeneous differential equation. y = vx =v+x
dx dx
dv 2vx x
(1) v+x = = 2v 1
dx 2vx x 2v 1
dv 2v 1 2v 1 2v2 v 2v2 v 1
x = v= =
dx 2v 1 2v 1 2v 1
2v 1 dx
dv = (Variables are separate)
2v2 v 1 x
Integrating, we get I 2v 1
2v2 v 1
dv =
I dx
x
C.
I 2v 1
2v2 v 1
dv = log |x| + C ...(1)
Now
I 2v 1
2
2v v 1
dv =
1
2 I (4v 1) 3
2v2 v 1
dv
=
1
2 I 4v 1
2
2v v 1
dv
3
2 I 2
dv
2v v 1
=
1
2
log |2v2 v + 1| +
3
4 I v
dv
1
2
7
4 16
X
1 3
= log|2v 2 v 1| + tan1
2 4
1 3 4v 1
= log |2v2 v + 1| + tan1
2 7 7
1 3 4v 1
(1) log|2v2 v 1| + tan1 = log |x| + C
2 7 7
1 2 y2 y 3 4y x
log 1 + tan 1 = log |x| + C
2 x2 x 7 7x
1 3 4y x
log |2y2 xy + x2| log |x| + tan1 = log |x| + C
2 7 7x
1 3 4y x
log |2y2 xy + x2| + tan1 =C ...(2)
2 7 7x
We have y = 1 when x = 1.
3 41
(2) log |2 1 + 1| + tan1 =C
2 7 7
42 Self-Instructional Material
1 3 3 Differential Equations
C= log 2 + tan1
2 7 7
Using (2), the required solution is
[ Z 1 3 3 NOTES
NQI^ [ Z[ Z ^ + VCP = log 2 + tan 1 .
Z 2 7 7
dx
Solution of Homogeneous Differential Equation = y(x/y)
dy
We have
FZ = (x/y). ...(1)
F[
FZ FX FX
Let x = vy. = v(1) + y =v+y
F[ F[ F[
FX FX F[
(1) v+y = (v) (Variables are separate)
F[
X X [
Integrating both sides, we get I FX
X X
F[
[ I
+ C.
I FX
\
X X
= log |y| + C, where v =
Z
[
.
Z
This equation is solved and v is replaced by
[
Solution. We have
FZ = Z + sin Z . ...(1)
F[ [ [
FZ
This is a homogeneous differential equation of the form = (x/y).
F[
Let x = vy
FZ =v+y
FX
F[ F[
FX
(1) v+y = v + sin v
F[
y
FX = sin v cosec v dv =
F[ (Variables are separate)
F[ [
Integrating, we get I EQUGE X dv =
X
I F[
[
+ log C.
Self-Instructional Material 43
Ordinary Differential
Equations Z
log VCP = log C |y|
[
Z Z
VCP = C y VCP = Cy
NOTES [ [
Z
VCP = C1y. (Putting C1 = ±C)
[
Example 43. Solve: 2yex/y dx + (y 2xex/y)dy = 0.
Solution. We have 2yex/y dx + (y 2xex/y)dy = 0.
FZ ZG Z [
[ FZ
Z [ G Z [
= = ...(1)
F[ [G Z [
F[ G Z [
FZ
This is a homogeneous differential equation of the form = (x/y).
F[
FZ FX
Let x = vy. =v+y
F[ F[
(1) v+y
FX =
XG X
F[ GX
FX XG X
y = v
F[ GX
FX
y = X 2evdv = dy
F[ G [
Integrating, we get
I GX FX =
[
I
dy + C.
Solution. We have (1 + e )dx + e
Z
dy = 0.
[
x/y x/y
Z
G Z [
FZ
[ ...(1)
F[ GZ [
FZ
This is a homogeneous differential equation of the form
Z [
F[
FZ FX
Let x = vy =v+y
F[ F[
(1) v+y
FX =
G
X
X
F[ G
X
y
FX =
X
G XG
X
X
X
G XG
X
X XG
X
F[ GX GX
FX G
X
X
y =
F[ G X
44 Self-Instructional Material
GX F[ Differential Equations
dv = (Variables are separate)
X G
X [
I GX
X G
log v +
X
dv =
ev
I F[
[
+ log C
FZ Z + y x sin Z = 0, y(/2) = 1.
Example 45. Solve: y
F[
sin
[ [
Solution. We have
FZ Z + y x sin Z = 0.
y
F[
sin
[ [
FZ =
Z UKP
Z [ [
FZ =
Z [ UKP
Z [
...(1)
F[ [ UKP
Z [ F[ UKP
Z [
0=0+C C=0
(2) log | y | = cos (x/y). This is the required solution.
EXERCISE I
Self-Instructional Material 45
Ordinary Differential dy y( x y ) dy
Equations 9. x2 = 10. x = y(log y log x + 1)
dx 2 dx
dy y y dy
11. = + tan 12. (x y) = x + 2y
dx x x dx
NOTES 13. x2y1 = x2 2y2 + xy 14. (x2 y2)dx + xy dy = 0
y
15. x2 dy + y(x + y)dx = 0 16. y dx + x log dy 2x dy = 0
x
[ [ [ [ Z [ [
17. Z[ NQI FZ [ Z F[ 18. EQU FZ UKP EQU F[
Z
NQI
Z Z Z [ Z Z
19. (y2 2xy)dx = (x2 2xy)dy 20. y2 dx + (x2 xy + y2)dy = 0
21. 2xy dx + (x2 + 2y2)dy = 0 22. (y2 x2)dy = 3xy dx
y dy y
23. x cos = y cos x 24. (x y)dy (x + y)dx = 0
x dx x
dy y
25. x y x sin = 0.
dx x
Solve the following initial value problems (Q. No. 2645):
2 2 dy dy
26. y x xy , y(1) 1 27. x(x2 + 3y2)dx + y(y2 + 3x2)dy = 0, y(1) = 1
dx dx
y/ x dy
28. (y4 2x3y)dx + (x4 2xy3)dy = 0, y(1) = 1 29. xe yx 0, y( e) 0
dx
30. (xey/x + y)dx = x dy, y(1) = 1 31. (x + y)dy + (x y)dx = 0, y(1) = 1
dy dy
32. 2xy + y2 2x2 = 0, y(1) = 2 33. 2x2 2xy + y2 = 0, y(e) = e
dx dx
34. 2yex/y dx + (y 2xex/y)dy = 0, y(0) = 1 35. (x2 y2)dx + 2xy dy = 0, y(1) = 1
dy
36. x2 = y2 + 2xy, y(1) = 1 37. dy y cosec y 0 , y(1) = 0
dx dx x x
y y
38. (x dy y dx)y sin = (y dx + x dy)x cos , y(3) =
x x
F[ [ F[ [ [
39. x y + x sin = 0, y(2) = . 40. x sin + x y sin = 0, y(1) =
FZ Z FZ Z Z
[ F[ [
41. (3xy + y2) dx + (x2 + xy)dy = 0, y(1) = 1 42. x cos = x + y cos , y(1) =
Z FZ Z
43. (x2 + xy)dy = (x2 + y2)dx, y(1) = 0 44. (x y)
F[ = x + 2y, y(1) = 0
FZ
45. (x2 + y2)dy xy dx = 0, y(0) = 1.
Answers
1. x2y(2x + y) = C 2. x2 + y2 = Cx3
y
3. log |x| = log (x y)2 + +C 4. x = C(x2 + y2)
x
x x3
5. log |x| = +C 6. = log |y| + C
y 3 y3
1 y
7. y log |x| + x + Cy = 0 8. log (x2 + y2) + tan1 =C
2 x
y
9. xy2 = C(y x)2 10. log = Cx
x
46 Self-Instructional Material
y Z [ Differential Equations
11. x = C sin 12. log |x2 + xy + y2| VCP =C
x Z
1 x 2y
13. log = log |x| + C 14. y2 = x2(C 2 log|x|)
2 2 x 2y
NOTES
y
15. x2y = C(y + 2x) 16. Cy = log 1
x
Z [ y
17. log y2 + NQI =C 18. y sin =C
[ Z x
1
19. x2y xy2 = C 20. y = Ce tan ( y / x)
21. 3x2y + 2y3 = C 22. y2(4x2 y2)3 = C
[ y 1
23. sin = log |x| + C 24. tan 1 log ( x2 y2 ) C
Z x 2
y C y y
25. 1 cos sin 26. log |y| = 1
x x x x
27. x4 + 6x2y2 + y4 = 8 28. x3 + y3 = 2xy
1 1
29. y = x log log |x| 30. log |x| =
e e y/ x
y
31. log (x2 + y2) + 2 tan1 = + log 2
x 2
2x
32. + log |x| = 1 33. y log ex = 2x
y
34. 2ex/y + log |y| = 2 35. x2 + y2 = 2x
[
36. 2y = x(x + y) 37. EQU NQI^ Z^
Z
[ [ [
38. 2xy cos = 3 39. Z EQUGE EQV = 2
Z Z Z
[
40. log x = cos 41. x2y2 + 2x3y = 3
Z
[ [
42. sin = log x + 43. + 2log x y log x = 0
Z Z
Z [
44. log x2 + y2 + xy + = tan1
Z
45. x2 = 2y2 log y.
dy a x b1y c1 a1 b
Solution of 1 , where 1 by Reducing it to a
dx a2 x b2 y c2 a2 b2
Homogeneous Equation
Consider the differential equation
dy a x b1 y c1
a1 b
= 1 1., where ...(1)
dx a2 x b2 y c2
a2 b2
We substitute x = X + h and y = Y + k, where h and k are constants to be
properly chosen.
dy dy dY dX dY dY
= × × =1× ×1=
dx dY dX dx dX dX
Self-Instructional Material 47
Ordinary Differential dY a (X h) b1 (Y k) c1
Equations (1) = 1
dX a2 (X h) b2 (Y k) c2
dY a X b1Y (a1h b1k c1)
= 1 ...(2)
dX a2 X b2 Y (a2 h b2 k c2 )
NOTES The constants h and k are chosen so that a1h + b1k + c1 = 0 and
a2h + b2k + c2 = 0.
dY a X b1 Y
(2) = 1 ...(3)
dX a2X b2 Y
This is a homogeneous differential equation and can be solved by putting
Y = VX.
dY dV
Y = VX =V+X
dX dX
dV a X b1VX a b V
(3) V+X = 1 = 1 1
dX a2 X b2 VX a2 b2 V
dV a b V a b V a2 V b2 V 2
X = 1 1 V = 1 1
dX a2 b2 V a2 b2 V
C D 8 dX
dV = ...(4)
C
D C 8 D 8 X
In the differential equation (4), the variables X and V are separated.
I C D 8
C
D C 8 D 8
dV = log |X| + C,
C D F[ C Z D [ E
Remark. If in the differential equation then it can be easily
C D FZ C Z D [ E
solved by putting z = a1x + b1y or z = a2x + b2y.
G\ D [ E \ F D E
Working Steps for Solving = , where
G[ D [ E \ F D E
Step I. Put x = X + h and y = Y + k. Given differential equation reduces to
dY a X b1Y ( a1h b1k c1 )
= 1 .
dX a2 X b2Y ( a2h b2 k c2 )
Step II. Solve a1h + b1k + c1 = 0 and a2h + b2k + c2 = 0 to get the values of h
dY a X b1Y
and k. The resultant equation = 1 is a homogeneous
dX a2 X b2Y
differential equation.
Step III. Put Y = VX. This gives a differential equation in X and V with variables
separated.
Step IV. Solve this differential equation and put V = Y/X, X = x h and
Y = y k to get the answer in original variables x and y.
48 Self-Instructional Material
SOLVED EXAMPLES Differential Equations
dy x 2y 5
Example 46. Solve: = .
dx 2x + y – 4
dy x 2y 5 NOTES
Solution. We have = . ...(1)
dx 2 x+y–4
a1 1 b1 2 a1 b
Here = and = =2 1
a2 2 b2 1 a2 b2
Let x = X + h and y = Y + k.
dy dy dY dX dY dY
= × × =1× ×1=
dx dY dX dx dX dX
dY (X h ) 2(Y k ) 5
(1) =
dX 2(X h ) (Y k ) 4
dY X 2Y (h 2k 5)
= ...(2)
dX 2 X Y (2h k 4)
Let h and k be such that h + 2k 5 = 0 and 2h + k 4 = 0.
h = 1, k = 2 (On simplification)
dY X 2Y
(2) = ...(3)
dX 2X Y
This is a homogeneous differential equation.
dY dV
Let Y = VX. =V+X
dX dX
dV X 2(VX) 1 2V
(3) V+X = =
dX 2 X VX 2V
dV 1 2V 1 2V 2 V V2 1 V2
X = V= =
dX 2V 2V 2V
2V
1 V 2 dV =
dX
X
I 2V
1 V2
dV = I dX
X
+ log C
(Variables are separate)
I 2 V
(1 V)(1 V)
dV = log |X| + log C
I !
8
8
"#
$
dV = log C|X|
1 3 log|1 V|
log |1 + V| + = log C|X|
2 2 1
1 V
log = log C2X2
(1 V)3
1 Y/X XY
= C2X2 = ±C2
(1 Y/X)3 (X Y)3
X + Y = C1(X Y)3, where C1 = ±C2
(x 1) + (y 2) = C1((x 1) (y 2))3
x + y 3 = C1(x y + 1)3.
Self-Instructional Material 49
Ordinary Differential EXERCISE J
Equations
Solve the following differential equations:
dy x 2y 3 dy y x1
1. = 2. =
NOTES dx 2x y 3 dx yx5
dy x y1 dy 2x y 1
3. = 4. = .
dx x y2 dx x 2y 3
Answers
y2 1
1. x + y = C(x y + 6)3 2. tan1 + log (x2 + y2 6x 4y + 13) = C
x3 2
3. y2 + 2xy x2 2x 4y = C 4. x2 y2 xy + x + 3y = C.
dy
Let + Py = Q ...(1)
dx
be a linear differential equation, where P and Q are functions of x or constants.
P dx
Multiplying both sides of (1) by e I
, we get
dy
e I P dx + e I P dx Py = Q e
I P dx
.
dx
F
e I P dx
dy
+
d I P dx
(e ). y = Q e I P dx '
FZ
2FZ 2
dx dx
d
(y e I P dx ) = Q e I P dx
dx
I ! d
dx
"#
( ye I P dx ) dx =
$ I Qe I P dx
dx + C
yG I2 FZ
= I QG I2 FZ
dx + C.
P dx
This is the general solution of linear differential equation (1). The function e I
y(I.F.) =
3
+( FZ
%
log f ( x )
Remark. In evaluating integrating factor (I.F.), the results e = f (x) is frequently
used.
50 Self-Instructional Material
Differential Equations
G\
Working Steps for Solving + Py = Q
G[
F[
Step I. If the coefficient of is not unity, it must be made unity by dividing NOTES
FZ
F[
the equation by the coefficient of .
FZ
Step II. Identify P and Q and make sure that these are functions of x or
constants.
Step III. Evaluate P dx.
Step IV. Find e P dx . This is the integrating factor (I.F.).
I
Step V. Put the value of I.F. in the general solution y(I.F.) = Q(I.F.) dx + C and
simplify it. This gives the general solution of the given differential
equation.
SOLVED EXAMPLES
dy
Example 47. Solve: + 2y = ex.
dx
dy
Solution. We have + 2y = ex. ...(1)
dx
This is a linear differential equation. Here P = 2 and Q = ex.
2 FZ = I 2 dx = 2x I.F. = e
I P dx
= e2x
ye2x =
G
ye2x = ex + C
Z
G Z FZ %
ye2x = I ex dx + C
y = ex + Ce2x.
Example 48. Solve: y 2y = cos 3x.
dy
Solution. We have 2y = cos 3x. ...(1)
dx
This is a linear differential equation. Here P = 2 and Q = cos 3x.
P dx
I
P dx = 2 dx = 2x I.F. = e I
= e2x
EQU Z . e G
Z
ye2x = 2x dx + C ye2x = cos 3x dx + C
G
Z
Let I= cos 3x dx
Self-Instructional Material 51
Ordinary Differential
e2 x sin 3x 2 Z EQU Z EQU Z
G
Z
Equations = +
3
G FZ
3
NOTES
=
e2 x sin 3x
3
2
e2x cos 3x
9
4
9 I e2x cos 3x dx
e 2x 4
I= (3 sin 3x 2 cos 3x) I
9 9
1 4 I = e 2x
e 2 x
9 9 (3 sin 3x 2 cos 3x) I=
13
(3 sin 3x 2 cos 3x)
(2) y = e2x
e 2 x "#
(3 sin 3 x 2 cos 3 x ) + Ce2x
! 13 #$
1
(3 sin 3x 2 cos 3x) + Ce2x.
y=
13
dy
Example 49. Solve: + ay = emx.
dx
dy
Solution. We have + ay = emx. ...(1)
dx
This is a linear differential equation. Here P = a and Q = emx.
P dx
P dx = a dx = ax I.F. = e = eax
I
e ( a m ) x G CZ G
C O Z
yeax = +C y= + C eax
am CO
(Provided a + m 0)
G OZ
y= + Ceax.
CO
This is the required solution of the given differential equation.
dy
If a + m = 0, then the given differential equation (1) becomes my = emx.
dx
Here P = m and Q = emx.
I 2 dx = I m dx = mx I.F. = e I P dx
= emx
y = emx
! I "#
FZ %
$ y = emx(x + C).
52 Self-Instructional Material
1 Differential Equations
This is a linear differential equation. Here P = and Q = 2x2.
I I
x
1 1
P dx = dx = log |x| = log (Assuming x > 0)
x x
1 NOTES
P dx
log
x 1
I.F. = e =e =
I
x
The solution of (1) is
y(I.F.) = I Q(I.F.) dx + C. y
1
x I 2 x2
1 dx C
x
y
x
2
Idy
x dx C
y
x
=2
x2
2
+C y = x3 + Cx.
I
P dx = sec2 x dx = tan x I.F. = e I
= etan x
The solution of (1) is y(I.F.) = Q(I.F.) dx + C.
I= I zez dz = zez I
= zez ez = (z 1) ez = (tan x 1) etan x
1.ez dz
I P dx = I 2( x 2)
x2 1
dx = I 3
1
x 1 x 1
dx
( x 1)3
= 3 log (x 1) log (x + 1) = log
x 1
(Assuming x 1, x + 1 > 0)
3
( x 1)
I P dx
log
x 1 ( x 1)3
I.F. = e =e =
I
x 1
Self-Instructional Material 53
Ordinary Differential
Equations y
( x 1)3
x 1
= I 2
x 1
.
( x 1)3
x 1
dx + C = 2 I x 2 2x 1
x 1
dx + C
NOTES
=2 I x 3 4 dx + C = 2 x
x 1 !2
2 "#
3 x 4 log ( x 1) + C
#$
Z
y = x2 6x + 8 log (x + 1) + C.
Z
dy
Example 53. Solve: sin x + 3y = cos x.
dx
dy dy
Solution. We have sin x + 3y = cos x. + 3y cosec x = cot x
dx dx
...(1)
I I
This is a linear differential equation. Here P = 3cosec x and Q = cot x.
x x
P dx = 3 cosec x dx = 3 log tan = log tan3
2 2
x
log tan 3 x
P dx 2
I.F. = e I
=e = tan3
I
2
The solution of (1) is y(I.F.) = Q(I.F.) dx + C.
I I
x
x x 1 tan 2
y tan3 = cot x tan3 dx + C = 2 . tan3 x dx + C
2 2 x 2
2 tan
2
=
1
2 I tan 2
x
2
dx + C
tan 4
x
2
=
1
2 I !tan
x
2
2
tan
x
2
sec
x
2
2 "
1 # dx + C
$
2
=
1
2 I! 2 tan
x
2
2
tan
x
2
sec
x"
2 #$
2
dx + C
2
=
1
2 I !
2 sec
x
2
1 tan
2 x
2
sec
x "
2 $
# dx + C
2 2
e 2 x dx = 1
Example 54. Solve: x
x dy
y
(x 0).
e 2 x
y dx = 1 dy e 2 x y
Solution. We have
x
x dy
i.e.,
dx
x
x
dy 1 2 x
e
dx x
y= ...(1)
x
54 Self-Instructional Material
1 e 2 x Differential Equations
This is a linear differential equation. Here P = and Q = .
I I
x x
P dx
1
dx = 2 x I.F. = e I P dx
=e
2 x
x
NOTES
The solution of (1) is y(I.F.) =
3
+( FZ % .
ye 2 x
I e 2 x
x
. e2 x
dx C = I x 1/2 dx C = 2 x + C
[G Z = 2 Z + C.
dy
Example 55. Solve: x + y x + xy cot x = 0 (x 0).
dx
x dy + y x + xy cot x = 0 i.e.,
dy 1
Solution. We have
dx dx
+
x
cot x y = 1.
...(1)
1
This is a linear differential equation. Here P = + cot x and Q = 1.
I I
x
P dx = 1
x
cot x dx = log |x| + log |sin x|
I
I.F. = e = elog (x sin x) = x sin x
The solution of (1) is y(I.F.) = Q(I.F.) dx C .
y(x sin x) =
Z UKP Z FZ %
= x( cos x)
I
EQU Z FZ % = x cos x + sin x + C
y = cot x +
+
% .
Z Z UKP Z
dy
Example 56. Solve: 3y cot x = sin 2x, y = 2 when x = .
dx 2
Solution. We have dy (3 cot x)y = sin 2x. ...(1)
dx
This is a linear differential equation. Here P = 3 cot x and Q = sin 2x.
I.F. = e I P dx
= e log (sin x)
3
= (sin x)3
The solution of (1) is
y(I.F.) =
3
+( FZ %
y(sin x)3 =
I UKP Z
UKP Z FZ %
=2 I
UKP Z EQU Z FZ % = 2
UKP Z
%
Self-Instructional Material 55
Ordinary Differential y = 2 sin2 x + C sin3 x ...(2)
Equations
Now, y = 2 when x = /2.
(2) 2 = 2 sin2 + C sin3 2 = 2 + C(1) C=4
2 2
NOTES (2) y = 2 sin2 x + 4 sin3 x.
F[
Example 57. Solve: tan x = 2x tan x + x2 y, y = 0 when x = .
FZ
Solution. We have
F[
tan x = 2x tan x + x2 y. ...(1)
FZ
F[ Z [
= 2x +
FZ VCP Z VCP Z
F[
+ (cot x) y = 2x + x2 cot x.
FZ
This is a linear differential equation. Here P = cot x, and Q = 2x + x2 cot x.
EXERCISE K
Solve the following differential equations (Q. No. 12):
dy dy
1. (i) + 2y = e4x (ii) 2y = e3x
dx dx
56 Self-Instructional Material
dy dy Differential Equations
(iii) + 2y = 6ex (iv) 4 + 8y = 5e3x
dx dx
dy dy
2. (i) +y=1 (ii) + y = ex
dx dx
dy dy NOTES
(iii) + y = e3x (iv) 4y = ex .
dx dx
Solve the following differential equations (Q. No. 315):
dy FZ
3. (i) +y=2x (ii) (y + 3x2) =x
dx F[
(iii) x dy + (y x3)dx = 0 (iv) x dy (y + 2x2)dx = 0
dy dy
4. (i) + 2y = sin x (ii) y = cos x
dx dx
dy dy
(iii) + 2y = xe4x (iv) + y = cos 2x
dx dx
dy 1 x log x dy 1 sin x
5. (i) +y= (ii) +y=
dx x dx 1 cos x
dy dy
(iii) + y = cos x sin x (iv) x + 2y = x cos x
dx dx
dy y dy
6. (i) = 2x2 (ii) 2x + y = 6x3
dx x dx
dy y dy y
(iii) x2 (iv) + = 3x2
dx x dx 2x
dy dy y
7. (i) sec x y = sin x (ii) + = ex
dx dx x
dy dy y sin x
(iii) x + 2y = x2 log x (iv) + = cos x +
dx dx x x
dy dy
8. (i) = y tan x 2 sin x (ii) + y sec x = tan x
dx dx
dy dy
(iii) y tan x = ex (iv) cos3 x + y cos x = sin x
dx dx
dy
9. (i) x y=x+1 (ii) (1 + x2)dy + 2xy dx = cot x dx
dx
dy dy
(iii) x + 2y = x2 (iv) (1 + x2) + y = tan1 x
dx dx
dy F[
10. (i) x log x + y = 2 log x (ii) Z [ G Z
dx FZ
dy 3 x2 sin 2 x dy
(iii) + y = (iv) (1 + x2) + y = etan 1 x
dx 1 x3 1 x3 dx
dy 4x 1 dy 4x 1
11. (i) + y= (ii) + 2 y=
dx x2 1 ( x 2 1)3 dx x 1 ( x 2 1) 2
dy [ 1 dy
(iii) + = (iv) + y tan x = 2x + x2 tan x
dx Z NQI Z x dx
dy dy
12. (i) x ay = x + 1 (ii) y tan x = 2 sin x
dx dx
dy dy
(iii) + 2y tan x = sin x (iv) + y cot x = 2 cos x
dx dx
Self-Instructional Material 57
Ordinary Differential dy 2 dy 2
Equations 13. (i) x log x +y= log x (ii) (x2 1) + 2xy =
dx x dx x2 1
dy dy
14. (i) (x2 + 1) + 2xy = x2 4 (ii) x log x + y = log x
dx dx
NOTES F[ F[ Z [ EQU Z
(iii) + y tan x = x2 cos2 x (iv) + =0
FZ FZ UKP Z
dy dy
15. (i) sin x + y cos x = cos x sin2 x (ii) + y cot x = x2 cot x + 2x
dx dx
dy
(iii) (1 + x2) 2xy = (x2 + 2)(x2 + 1)
dx
dy
(iv) (1 x2) + xy = ax.
dx
Solve the following initial value problems (Q.No. 1618):
dy dy
16. (i) x y = (x + 1)ex, y(1) = 0 (ii) y cot x = 4x cosec x, y(/2) = 0
dx dx
dy
(iii) x y = x cos x + sin x, y(/2) = 1
dx
dy
(iv) (x2 + 1) 2xy = ( x 4 2 x 2 1) cos x, y(0) 0
dx
dy dy
17. (i) 2 y tan x sin x , y( /3 ) 0 (ii) + y sec2 x = tan x sec2 x, y(0) = 1
dx dx
(iii)
F[ y = cos x, y(0) = 1 (iv) x
F[ + 2y = x2, y(2) = 1
FZ FZ
dy 1 dy
18. (i) (1 + x2) + 2xy = 2 , y(1) = 0 (ii) y cot x = 2x + x2 cot x, x 0, y(/2) = 0
dx 1 x dx
(iii) (1 + y + x2y)dx + (x + x3)dy = 0, y(1) = 0
(iv) cos x dy = sin x(cos x 2y)dx, y(/3) = 0.
Answers
e4 x
1. (i) y = + Ce2x (ii) y = e3x + Ce2x
6
5 3x
(iii) y = 2ex + Ce2x (iv) y = e + Ce2x
4
1 x
2. (i) y = 1 + Cex (ii) y = e + Cex
2
1 3x 1 x
(iii) y = e + Cex (iv) y = e + Ce4x
2 3
3. (i) y = 3 x + Cex (ii) y = 3x2 + Cx
Z %
(iii) y = (iv) y = 2x2 + Cx
Z
1 1
4. (i) y = (2 sin x cos x) + Ce 2x (ii) y = (sin x cos x) + Cex
5 2
1 4x 1 4x 1
(iii) y = xe e Ce2 x (iv) y = (2 sin 2x + cos 2x) + Cex
6 36 5
x
5. (i) y = log x + Cex (ii) y = tan + Cex
2
(iii) y = cos x + Cex (iv) yx2 = (x2 2) sin x + 2x cos x + C
58 Self-Instructional Material
6 3 C Differential Equations
6. (i) y = x3 + Cx (ii) y = x
7 x
x3 C 6 3 C
(iii) y = + (iv) y = x +
4 x 7 x
x–1 x C
NOTES
7. (i) y + 1 + sin x = C esin x (ii) y = e +
x x
1 2 1 2 C C
(iii) y = x log |x| x + (iv) y = sin x +
4 16 x 2 x
cos 2x xC
8. (i) y cos x = +C (ii) y = 1
2 sec x tan x
ex
(iii) y cos x = (sin x + cos x) + C (iv) y = tan x 1 + Cetan x
2
9. (i) y = x log x 1 + Cx (ii) (1 + x2)y = log |sin x| + C
x2 C
(iv) y = tan1 x 1 + C e tan x
1
(iii) y 2
4 x
x x2 4
14. (i) y(x2 + 1) = + 2 log |x + x2 4 | + C
2
1 C
(ii) y = log x (iii) y sec x = (x2 2) sin x + 2x cos x + C
2 log x
Z
(iv) (1 + sin x) y + =C
1
15. (i) y = sin2 x + C cosec x (ii) y = x2 + C cosec x
3
(iii) y = (1 + x2)(x + tan 1 x + C) (iv) y = a + C 1 x2
16. (i) y = xe1 ex (ii) y sin x = 2x2 2/2
(iii) y = sin x (iv) y = (x2 + 1) sin x
17. (i) y = cos x 2cos2 x (ii) y = tan x 1 + 2etan x
(iii) y = (sin x cos x) + ex (iv) 4y = x2
2
18. (i) (1 + x2)y = tan1 x /4 (ii) y = x2
4 sin x
(iii) xy + tan1 x = (iv) y = cos x 2 cos2 x.
4
Self-Instructional Material 59
Ordinary Differential
Equations
Hints
I
18. (ii) Here, I.F. = sin x.
I I I
NOTES
(2x + x2 cot x) sin x dx = 2x sin x dx + x2 cos x dx
= (sin x) x2
I (cos x) x2 dx
dy 1 y (1 x 2 ) 1 1
dx
x (1 x 2 )
=
x(1 x 2 )
x
y.
dy
Solution of + Py = Qyn, where P and Q are Functions of x or
dx
Constants, by Reducing it to a Linear Differential Equation
dy
Consider the differential equation + Py = Qyn, ...(1)
dx
where P and Q are functions of x or constants and n 0, 1.
Equation (1) is known as Bernoullis equation.
dy
Dividing (1) by yn, we get yn + Pyn+1 = Q. ...(2)
dx
Let z= yn+1.
dz dy dy dz
= (n + 1)yn or yn = ·
dx dx dx P dx
dz dz
(2) · + Pz = Q + P(1 n)z = Q(1 n). ...(3)
P dx dx
(3) is a linear differential equation with z as the dependent variable.
G\ n
Working Steps for Solving + Py = Qy
G[
dy
Step I. Divide the equation by yn and get yn + Pyn+1 = Q ...(1)
dx
dz dy dy 1 dz
Step II. Put z = yn+1. = ( n + 1)yn or yn = .
dx dx dx 1 n dx
dy
Putting the values of yn+1 and yn in (1), we get
dx
dz
+ P(1 n)z = Q(1 n). ...(2)
dx
Step III. (2) is a linear differential equation with dependent variable z.
60 Self-Instructional Material
dy 2 x Differential Equations
Example 58. Solve: + y= .
dx 3 y
dy 2 x
Solution. We have + y= . ...(1)
dx 3 y NOTES
This is a Bernoullis equation.
dy 2
Multiplying (1) by y , we get y
+ y3/2 = x. ...(2)
dx 3
dz 3 1/2 dy dy 2 dz
Let z = y3/2. = y or y =
dx 2 dx dx 3 dx
2 dz 2 dz 3
(2) + z=x +z= x ...(3)
3 dx 3 dx 2
(3) is a linear differential equation with z as the dependent variable.
3
Here P = 1 and Q= x.
I I
2
P dx
P dx = 1 dx = x and we have I.F. = e = ex
I
zex = I
3
3
2
x ex dx + C y3/2 ex =
3
2
3
ZG
!
Z
I GZ FZ "# + C
$
y3/2 ex = (x 1)ex + C y3/2 = (x 1) + Cex.
2 2
Example 59. Solve: y(x2y + ex)dx ex dy = 0.
Solution. We have y(x2y + ex)dx ex dy = 0
dy [
Z [ G Z dy x2 y2
= = +y
dx GZ dx ex
dy x y
2
dx
+ (1)y = e
x
2 ...(1)
x2
Here P = 1 and Q= .
ex
Self-Instructional Material 61
Ordinary Differential
Equations The solution of (3) is z(I.F.) = I Q(I.F.) dx + C.
zex = I –
x2
e x
e
x dx + C
I
NOTES
1 Z
ex = x2 dx + C ex = + C.
y [
EXERCISE L
Solve the following differential equations:
dy y dy y y2 dy x
1. + = y2 2. + = 3. + y=x y
dx x dx x x2 dx 1 x2
1
dy 2 y3 dy x2 dy
4. + y= 5. + xy = y2 e 2 sin x 6. + xy = xy5.
dx x x3 dx dx
Answers
1
1. + log x = C 2. 2x y = Cx2y
xy
1 1
3. 3 y x2 + 1 = C(1 x2)1/4 4. = +C
y2 x 4 3 x6
1 2
x 1
= 1 + C e2 x .
2
5. e 2 = y(cos x + C) 6.
y4
dy
Solution of f (y) + Pf(y) = Q, where P and Q are Functions of x or
dx
Constants and f(y) is Some Function of y, by Reducing it to a Linear
Differential Equation
dy
Consider the differential equation f (y) + Pf (y) = Q, ...(1)
dx
where P and Q are functions of x or constants and f(y) is some function of y.
dz dy
Let z = f(y). = f (y)
dx dx
dz
(1) + Pz = Q. ...(2)
dx
(2) is a linear differential equation with z as the dependent variable.
G\
Working Steps for Solving f ¢( y) + Pf (y) = Q
G[
dz dy
Step I Put z = f(y). = f (y)
dx dx
dy
Step II Put the values of f(y) and f (y) in the given differential equation
dx
dz
and get + Pz = Q.
dx
This is a linear differential equation with dependent variable z.
62 Self-Instructional Material
SOLVED EXAMPLES Differential Equations
dy 1 ey
Example 60. Solve: + = 2 .
dx x x
dy 1 ey NOTES
Solution. We have + = 2. ...(1)
dx x x
dy 1 1
Dividing (1) by ey, we get ey + ey = 2 .
dx x x
dy 1
1
ey
dx
+
x
ey = 2
x
...(2)
dy
(2) is a differential equation of the form f (y) + Pf(y) = Q, where f(y) = ey.
dx
dz dy
Let z = f(y) = ey. = ey
dx dx
dz
1 1
(2)
dx
+
x
z= 2.
x
...(3)
I I
x x2
1 1
P dx = dx = log x = log (Assuming x > 0)
x x
1
P dx
log 1
I.F. = e I
= e x =
I
x
The solution of (3) is z(I.F.) = Q(I.F.) dx + C.
ey
1
x
I –
1
x 2
.
1
x
dx C
e y
x
=
Z
+C
e y 1
= +C 2xey = 1 + 2Cx2.
x 2x 2
dy
Example 61. Solve: sin y = cos y(1 x cos y).
dx
dy
Solution. We have sin y = cos y(1 x cos y).
dx
dy
sin y cos y = x cos2 y ...(1)
dx
sin y dy cos y
Dividing (1) by cos2 y, we get = x.
cos2 y dx cos2 y
dy
sec y tan y + (1) sec y = x ...(2)
dx
dy
(2) is a differential equation of the form f (y) + Pf (y) = Q, where f(y) = sec y.
dx
dz dy
Let z = f (y) = sec y. = sec y tan y
dx dx
dz
(2) + (1)z = x ...(3)
dx
(3) is a linear differential equation with z as the dependent variable.
Here P = 1 and Q = x.
Self-Instructional Material 63
Ordinary Differential
I P dx = I 1 dx = x and we have I.F. = e P dx
= ex.
I
I
Equations
The solution of (3) is z(I.F.) = Q(I.F.) dx + C.
ex sec y = Z
!
G
Z
I
G
Z "#
#$
FZ + C
GZ
ex sec y = xex +C sec y = x + 1 + Cex.
Remark. Please note carefully the placing of cos y on the LHS in equation (1). The
placing of x cos2 y on the LHS of (1) will not reduce the given differential equation to the
desired form.
EXERCISE M
Solve the following differential equations:
dy dy 1
1. (1 x) 1 ex – y 2. + y = y3
dx dx x
dy x 2 y2 1 dy
3. = 4. 2 tan y + x sin2 y = x3 cos2 y
dx 2xy dx
dy 1 1 dy y y
5. + tan y = 2 tan y sin y 6. log y = 2 .
dx x x dx x x (log y )2
Answers
1. ey(1 + x) = ex + C 2. 2xy2 + Cx2y2 = 1
1 2
x
3. y2 + 1 = x2 + Cx 4. tan2 y = x2 2 + C e 2
3
5. 2x cosec y = 1 + 2Cx2 6. (log y)3 = + Cx3.
4x
Hints
dy x 2 y2 1
3. We have = .
dx 2xy
dy x 2 y2 1 dy y2 1 dy 1
2y = 2y =x+ + 2y + (y2 + 1) = x.
dx x dx x x dx x
dy
Solution of Linear differential equation + Px = Q, where P and Q are
dx
functions of y or constants
FZ
Let + Px = Q ...(1)
F[
be a linear differential equation, where P and Q are functions of y or constants.
Multiplying both sides of (1) by e P dy, we get
FZ
G I 2 F[ G I 2 F[ 2Z 3 G I 2 F[
.
F[
64 Self-Instructional Material
FZ F F Differential Equations
G I 2 F[
F[ F[
G I
2 F[ Z 3G I 2 F[ '
F[ 2F[ 2
F 2 F[ 3 G 2 F[
Z G
I I
F[ NOTES
F
F[
Z G
2 F[
F[
3 G 2 F[ F[ %
Z G 2 F[
3 G 2 F[ F[ %
G[
Working Steps for Solving + Px = Q
G\
Step I. Identify P and Q and make sure that these are functions of y or
constants.
Step II. Evaluate P dy.
Step III. Find eP dy. This is the integrating factor (I.F.).
Step IV. Put the value of I.F. in the general solution x(I.F.) = Q(I.F.) dy + C and
simplify it. This gives the general solution of the given differential
equation.
SOLVED EXAMPLES
FZ
(1) is a linear differential equation of the form + Px = Q.
F[
Here, P= and Q = y2.
[
I I
2 F[
[
dy = log y I.F. = e P dy = elog y = y
(Assuming y > 0)
xy = I [ [ F[ + C xy =
[
+ C.
dy
Example 63. Solve: (x + 3y2) = y, y > 0.
dx
dy
Solution. We have (x + 3y2) = y.
dx
Self-Instructional Material 65
Ordinary Differential
Equations dx dx x dx 1
y
dy
= x + 3y2
dy
=
y
+ 3y
dy
+
y
x = 3y ...(1)
FZ
NOTES This is a linear differential equation of the form + Px = Q.
F[
1
Here, P= and Q = 3y.
y
I P dy = I 1
y
dy = log y = log y1 = log
1
1
y
(y > 0 |y| = y)
I
y
x
1 =
y I 3y
1 dy + C
y
Z
[
= 3y + C.
FZ
Example 64. Solve: y2 Z = 0.
F[ [
FZ
Solution. We have y2 Z = 0.
F[ [
FZ FZ
y2
F[
Z
[
F[
[
Z
[
...(1)
FZ
(1) is a linear differential equation of the form + Px = Q.
F[
Here, P= and Q= .
[ [
I 2 F[ I [
F[
[
I.F. = e P dy = e1/y
xe1/y = I
[
G [ F[ + C ...(2)
Let I= I
[
G [ F[
Let z= y= and dy = dz
[ \ \
I= I \ G \
F\ I \G F\
\
I
\
\G G F\"# = ze + e = e (1 z) = e
! $ [
\ \ z z z 1/y
=
+ C x = 1 + + Ce .
xe 1/y = e 1/y
[
(2) 1/y
[
F[
Example 65. Solve: (1 + y2) + (x G VCP = 0.
[
)
FZ
[ F[
Solution. We have (1 + y2) + (x G VCP ) = 0.
FZ
66 Self-Instructional Material
Differential Equations
FZ FZ G VCP [
(1 + y2) Z G VCP [
=0 Z ...(1)
F[ F[ [ [
FZ
Equation (1) is a linear differential equation of the form + Px = Q. NOTES
F[
[
G
VCP
Here P= and Q = .
[ [
I 2 F[ I [
F[ VCP
[ I.F. = e P dy = G VCP
[
I
VCP [
[ G
[
x G VCP = G VCP F[ %
[
x G VCP
[
= I G
\
F\ + C, where z = tan1 y
\ VCP [
G G
%
xG VCP [
= +C 2x G VCP [
= G VCP [
+ 2C.
Example 66. Solve: y dx (x + 2y2)dy = 0, y > 0 given that y = 1 when x = 2.
Solution. We have y dx (x + 2y2)dy = 0.
dx dx x dx 1 x = 2y
y
dy
= x + 2y2
dy
=
y
+ 2y
dy
+
y ...(1)
FZ
(1) is a linear differential equation of the form + Px = Q.
F[
1
Here P= and Q = 2y.
y
I P dy = I
1
y
dy = log y = log y1 = log
1
y
(y > 0 y = y)
1
log 1
P dy y
I.F. = e =e =
I
x
1 =
y I 2y
1 dy + C
y
x
y
= 2y + C ...(2)
2
Now y = 1 when x = 2. (2) = 2(1) + C or C = 0
1
x
Using (2), the required solution is = 2y + 0 i.e., x = 2y2.
y
Self-Instructional Material 67
Ordinary Differential EXERCISE N
Equations
Solve the following differential equations:
F[
1. (i) y dx (x + 2y2)dy = 0 (ii) (x + 2y3) = y, y > 0
NOTES FZ
F[
(iii) (3y2 x)dy = y dx, y > 0 (iv) y2 + Z
[ FZ
=0
F[
2. (i) (1 + y2)dx = (tan1 y x)dy (ii) (1 + y2) + (2xy cot y)
FZ
F[ F[
(iii) (2x 10y3) +y=0 (iv) (x + y) = 1.
FZ FZ
VCP [
3. (i) (1 + y2)dx + (x G )dy = 0, y(0) = 0
(ii) (1 + y2)dx = (tan 1 y x)dy, y(1) = 0
(iii) (x sin y)dy + (tan y)dx = 0, y(0) = 0.
FZ
(iv) + x cot y = 2y + y2 cot y, (y 0), y(0) = .
F[
Answers
1. (i) x = 2y2
+ Cy (ii) x = y3 + Cy
(iii) xy = y3 + C (iv) x = 1 + y 1 + Ce1/y
VCP [
2. (i) x = tan1 y 1 + C G (ii) x(1 + y2) = log sin y + C
(iii) x = 2y3 + Cy 2 (iv) x + y 1 = Cey
3. (i) x GVCP [ = tan 1 y (ii) (x tan1 y + 1) GVCP [ = 2
(iii) 2x = sin y (iv) 4(x y2) sin y + 2 = 0.
SUMMARY
1. An equation involving independent and dependent variables and at least one derivative/
differential of these variables called a differential equation.
2. The order of a differential equation is the order of the derivative of the highest order,
occurring in the differential equation.
3. The degree of a differential equation is defined if it can be written as a polynomial equation
in the derivatives and for such a differential equation its degree is given by the highest
power of the highest order derivative appearing in it, provided the derivatives are made
free from radicals and fractions.
4. A differential equation is said to be linear, if the dependent variable and its derivatives
occur only in the first degree and are not multiplied together.
5. (i) A solution of a differential equation is a functional relation between the variables
involved which satisfies the given differential equation.
(ii) A solution of a differential equation is called the general solution (or complete
solution), if it contains as many arbitrary constants as the order of the differential
equation.
(iii) A solution obtained by giving particular values to arbitrary constants in the general
solution of a differential equation is called a particular solution of the differential
equation, under consideration.
68 Self-Instructional Material
6. (i) If
F[
FZ
= f (x), then dy = f (x) dx.
I I F[ H
Z FZ + C.
This represents the general solution of the given differential equation.
Differential Equations
(ii) If
F[
FZ
= g(y), then
F[
I
[
= f (x).
I I F[
I
[
H
Z % .
This represents the general solution of the given differential equation.
NOTES
(iii) If
dy
dx
= f(x) g(y), then
dy
g ( y)
= f(x) dx. I 1
g( y)
dy
dy
7. If = f(ax + by + c), then z = ax + by + c reduces the given differentiable equation to
dx
variable separable form.
dy y
8. If
dx
= f
x is a homogeneous equation, then y = vx reduces the given differential
F[ C Z D [ E C D
9. If CPF then put x = X + h and y = Y + k where h and k are
FZ C Z D [ E C D
constant such that a1h + b1k + c1 = 0, a2h + b2 k + c2 = 0. The substitution Y = VX
reduces the resultant equation to variable separable form.
dy
10. If + Py = Q is a linear differential equation, where P and Q are functions of x or
I
dx
constants, then [G I 2 FZ =
3G
I
2 FZ FZ %
dy
11. A differential equation of the form + Py = Qyn, where n 0, 1 and P and Q are functions
dx
of x or constants, is solved by putting z = yn+1. This substitution reduces the given
differential equation to a linear differential equation.
dy
12. A differential equation of the form f (y) + f(y)P = Q, where P and Q are functions of x
dx
or constants, is solved by putting z = f(y). This substitution reduces the given differential
equation to a linear differential equation.
FZ
13. If + Px = Q is a linear differential equation, where P and Q are functions of y or
F[
constants, then Z GI 2 F[
equation.
I
3 G 2 F[ F[ % is the general solution of the given differential
I
Self-Instructional Material 69
Ordinary Differential
Equations UNIT II
NOTES
2. EXACT DIFFERENTIAL
EQUATIONS
STRUCTURE
Introduction
Theorem
Equations Reducible to Exact Equations
INTRODUCTION
THEOREM
The necessary and sufficient condition for the differential equation Mdx + Ndy =
0 to be exact is
M N
.
y x
The condition is necessary
The equation Mdx + Ndy = 0 will be exact, if du = Mdx + Ndy
u u
But du = dx dy
x y
u u
Mdx + Ndy = dx dy
x y
Equating co-efficients of dx and dy, we get
u u
M= and N =
x y
70 Self-Instructional Material
Exact Differential
M 2 u N 2 u Equations
and
y yx x xy
2u 2u
But NOTES
yx xy
M N
y x
which is the necessary condition of exactness.
The condition is sufficient.
Let u= I
y constant
Mdx
u 2 u M
M and
x yx y
2u 2u M N
But and
yx xy y x
N 2 u u
x xy x y
u
Integrating both sides w.r.t. x treating y as constant, we have N = f ( y)
y
u u%& () ' M
u
,N=
u
f ( y)
"#
Mdx + Ndy =
x
dx
y
f ( y) dy
' * ! x y $
u dx u dy + f(y) dy = du + f(y) dy = d[u + f(y) dy]
=
x y
which shows that Mdx + Ndy is an exact differential and hence Mdx + Ndy = 0 is an
exact differential equation.
Note. Since Mdx + Ndy = d[u + f(y) dy]
Mdx + Ndy = 0 d[u + f(y) dy] = 0
I
Integrating, u + f(y) dy = c
I I
Hence the solution of Mdx + Ndy = 0 is
Mdx (terms of N not containing x) dy = c.
y constant
SOLVED EXAMPLES
Self-Instructional Material 71
Ordinary Differential
I I
Thus the given equation is exact and its solution is
Equations
Mdx (terms of N not containing x) dy = c
y constant
NOTES
i.e.,
I
y constant
(5x4 + 3x2y2 2xy3) dx + I 5y4dy = c
or x5 + x3y2 x2y3 y5 = c.
Example 2. Solve [cos x tan y + cos (x + y)] dx + [sin x sec2 y + cos (x + y)] dy = 0.
Sol. Here, M = cos x tan y + cos (x + y)
and N = sin x sec2 y + cos (x + y)
M N
= cos x sec2 y sin (x + y) =
y x
Thus the given equation is exact and its solution is
I
y constant
I I
Thus the given equation is exact and its solution is
I
y constant
EXERCISE A
Solve the following differential equations (1 to 22):
1. (1 + 4xy + 2y2)dx + (1 + 4xy + 2x2) dy = 0 2. (3x2 + 6xy2)dx + (6x2y + 4y3) dy = 0
3. y(y2 3x2)dy + x(x2 3y2) dx = 0, y(0) = 1 4. (2x3 xy2 2y + 3) dx (x2y + 2x)dy = 0
dy ax hy g y2 1 1 x
2
5. 0 6. 2
dx 2 dy 0
dx hx by f ( y x) x y ( x y)
72 Self-Instructional Material
2 Exact Differential
xdy ydx a ( xdy ydx )
7. xdy + ydx + 2 2
0 8. xdx ydy 2 2 Equations
x y x y
9. dx
y
1 x 2 y2
dx
x
1 x 2 y2
dy
10. 2x 1
2
x y dx 2
x y dy
NOTES
11. ( y cos x + 1) dx + sin x dy = 0
1
(ii) y 1 cos y dx (x log x )(cos y y sin y) dy 0
x
x
21. e (cos y dx sin y dy) 0, y(0) 0
1 sin x 9
22. cos x log(2 y 8) x dx y 4 dy 0, y (1)
2
23. Find the value of for which the differential equation ( xy 2 x 2 y)dx ( x y)x 2dy 0 ,
is exact. Hence solve it.
Answers
1. (x + y) (1 + 2xy) = c 2. x3 + 3x2y2+ y4 = c 3. x4 6x2y2 + y4 = 1
4. x4 x2y2 4xy + 6x = c 5. ax2 + 2hxy + by2 + 2gx + 2fy + c = 0
y2 y 1 x
6. log c 7. xy tan c
yx x y
x 1 xy
8. x 2 y2 2a 2 tan 1 c 9. log 2x c
2
10. 3x 2(x y)
2 3/2
c
y 1 xy
11. y sin x + x = c 12. (i) y(x + log x) + x cos y = c (ii) y cos y (x + log x) = c
13. x2y + xy x tan y + tan y = c 14. x + yex/y = c 15. xey + y2 = c
2
16. exy + y2 = c 17. e xy x 4 y3 c 18. sec x tan y ex = c
1 2x
19. cos x cos y + e log sec y c 20. y sin x2 x2y + x = c
2
21. ex cos y = 1 22. sin x log (2y 8) + log x = 0
1 2 2 3
23. = 3; x y x y c.
2
Self-Instructional Material 73
Ordinary Differential
Equations EQUATIONS REDUCIBLE TO EXACT EQUATIONS
Differential equations which are not exact can sometimes be made exact after
NOTES multiplying by a suitable factor (a function of x and/or y) called the integrating factor.
For example, consider the equation y dx x dy = 0 ...(1)
Here, M = y and N=x
M N , therefore the equation is not exact.
y x
1
(i) Multiplying the equation by 2 , it becomes
y
ydx xdy x
y2
= 0 or d
y
=0
which is exact.
1
(ii) Multiplying the equation by , it becomes
x2
ydx xdy y = 0
x2
=0 or d
x which is exact.
1
(iii) Multiplying the equation by , it becomes
xy
dx dy
= 0 or d(log x log y) = 0 which is exact.
x y
1 1 1
, and are integrating factors of (1).
y2 x 2 xy
If a differential equation has one integrating factor, it has an infinite number of
integrating factors.
xdy ydx
y "# ydx xdy
(v)
xy
d log
!
x $ (vi)
xy
= d [log (xy)]
xdx ydy 1
log ( x 2 y 2 )
"# xdy ydx 1 x y
(vii)
x 2 y2
d
2 ! $ (viii)
x 2 y2
d
2
log
x y
.
SOLVED EXAMPLES
3
Example 4. Solve ydx xdy + 3x2y2 e x dx = 0.
3 3 3
Sol. Since 3 x 2 e x = d( e x ), the term 3 x 2 y2 e x dx should not involve y2.
1
This suggests that may be an I.F.
y2
74 Self-Instructional Material
1 ydx xdy 3 Exact Differential
Multiplying throughout by 2
, we have 2
+ 3x2 e x dx = 0 Equations
y y
x d (e x3
or d
y ) = 0, which is exact.
NOTES
x 3
Integrating, we get e x c , which is the required solution.
y
Example 5. Solve xdy ydx = x x 2 y 2 dx.
xdy ydx
y
1
2
x2
Sol. The given equation is xdy ydx = x2
x dx or
y
2
dx
1
x
y
1
or d sin = dx, which is exact.
x
y
Integrating, we get sin1 =x+c or y = x sin (x + c), which is the required
x
solution.
a 2 (xdy ydx)
Example 6. Solve: xdx + ydy = .
x2 y2
y
1
Sol. The given equation is xdx + ydy a2d tan =0
x
x2 y2 y
Integrating, we get a 2 tan 1 c
2 2 x
y
or x2 + y2 2a2 tan1 = C, where C = 2c.
x
EXERCISE B
Solve the following differential equations:
1. xdy ydx = (x2 + y2) dx 2. xdy ydx = (x2 + y2) (dx + dy)
3. y(2xy + ex) dx = ex dy 4. (y log y 2xy) dx + (x + y) dy = 0
5. xdy ydx = xy2 dx 6. xdy = (x2y2 y) dx
2 dy dy
7. ( x y) x y xy 1 8. xdy ydx (4x 2 y2 )dy
dx dx
9. (y + y2 cos x) dx (x y3) dy = 0.
Answers
1 y 1 y
1. tan xc 2. tan x yc
x x
ex
3. x2 c 4. x log y x2 + y = c
y
x x2 1
5. c 6. xc
y 2 xy
1 1 1 y
7. log (xy) c 8. tan yc
x y 2 2x
x y2
9. sin x c.
y 2
Self-Instructional Material 75
Ordinary Differential Hints
Equations
xdy ydx y 1 ye x dx e x dy ex
1. 2 2
dx d tan 1 dx 3. I.F. 2
and 2
d
x y x y y y
NOTES 1 x xdy ydx d( xy )
4. I.F. and dy log ydx d ( x log y) 6. dx dx
y y x 2 y2 ( xy )2
xdy ydx dx dy d( xy ) d( x y )
7.
xy ( x y )2 xy ( x y )2
y
d
xdy – ydx ( xdy ydx )/x 2 x dy
8. dy dy .
4 x 2 y2 4 ( y/x )2 y
2
4
x
1 2 dx x 3 dy = 0, which is exact.
y x y y 2
EXERCISE C
Solve the following differential equations:
1. (xy 2y 2) dx (x2 3xy) dy = 0 2. x2y dx (x3 + y3) dy = 0
3. (3xy2 y3) dx (2x2y xy2) dy = 0 4. (x2 3xy + 2y2) dx + x(3x 2y)dy = 0.
Answers
3
x x
1. 2 log x 3 log y c 2. log y 3
c
y 3y
3. y 2
4. x log x 3xy y cx
2 2
3 log x 2 log y c
x
76 Self-Instructional Material
I.F. for an Equation of the Form f1(xy) ydx + f2(xy) xdy = 0. Exact Differential
Equations
1
If Mdx + Ndy = 0 is of the form f1(xy) ydx + f2(xy) xdy = 0, then is an
Mx – Ny
I.F. provided Mx Ny 0. NOTES
EXERCISE D
Solve the following differential equations:
1. (1 + xy) ydx + (1 xy)xdy = 0. 2. (x2 y2 + xy + 1) ydx + (x2y2 xy + 1)xdy = 0.
3. y(2xy + 1)dx + x(1 + 2xy x3y3)dy = 0. 4. (xy2 + 2x2y3)dx + (x2y x3y2) dy = 0.
5. (y xy2)dx (x + x2y) dy = 0.
6. (xy sin xy + cos xy) ydx + (xy sin xy cos xy) xdy = 0.
Answers
1 x x 1
1. log c 2. xy log c
xy y y xy
1 1 1
3. 2 2
3 3
log y c 4. 2 log x log y c
x y 3x y xy
x
5. log xy c 6. y cos xy = cx.
y
Self-Instructional Material 77
Ordinary Differential N M
Equations
x y
(ii) If = g(y), a function of y only, then eg(y)dy is an I.F.
M
NOTES
SOLVED EXAMPLES
1
3
Example 9. Solve: (xy2 ex ) dx x2y dy = 0.
1
3
Sol. Here, M= xy2 ex and N = x2y
M N
y x 2 xy ( 2 xy) 4
2
, which is a function of x only.
N x y x
I.F. = e
I
4
x
dx
e 4 log x
1
x4
y 1
, we have
2 1
x x e dx x
1 y x3
Multiplying throughout by dy = 0
x4 3 4 2
I y 1 2 1
y constant 3
4
I I
1
y2 1 3 3 y2 1 1
or e x dx c or e t dt = c, where t =
2x 2
3 x 4
2x 2
3 x3
1
y2 1 3 y2 3
or 2
et c or 2
2e x = C, where C = 6c.
2x 3 x
N M
x y 4 xy 2 2 3 xy 2 1 xy2 1 1
3
2
M xy y y( xy 1) y
which is a function of y only.
I.F. = e
I 1
y
dy
= elog y = y
Multiplying throughout by y, we have (xy4 + y2) dx + 2 (x2y3 + xy + y5) dy = 0
x2 y4 y6
or xy2 c.
2 3
78 Self-Instructional Material
EXERCISE E Exact Differential
Equations
Solve the following differential equations:
1. (x2 + y2 + x) dx + xydy = 0 2. (x2 + y2 +1) dx 2xydy = 0.
3. (x2 + y2 + 2x)dx + 2ydy = 0. 4. (y4 + 2y)dx + (xy3 + 2y4 4x) dy = 0. NOTES
y
3
x
2
1 2
5. y dx (x xy )dy 0. 6. (x sec2 y x2 cos y)dy = (tan y 3x4)dx.
3 2 4
7. (xyex/y + y2) dx x2ex/y dy = 0. 8. (3xy 2ay2)dx + (x2 2axy)dy = 0.
9. (x4ex 2mxy2 )dx + 2mx2ydy = 0. 10. y(2x2y + ex)dx = (ex + y3)dy.
11. ydx xdy + log x dx = 0. 12. (2x log x xy) dy + 2ydx = 0.
13. (3x2y4 + 2xy)dx + (2x3y3 x2) dy = 0. 14. y log y dx + (x log y) dy = 0.
Answers
2
y 1 x 2 2
1. 3x4 + 6x2y2 + 4x3 = c 2. x c 3. e (x y ) c
x x
2 2 tan y
4. y 2 x y c 5. 3x2y + x4y3 + x6 = c 6. x 3 sin y c
y x
2
x y
7. ex/y + log x = c 8. x2y(x ay) = c 9. e m c
x
2x 3 e x y2 1 2
10. c 11. 1 + y + log x = cx 12. 2 y log x y c
3 y 2 2
2 2 4 6
3 2 x 1 2 x y 2 y
13. x y c 14. x log y (log y) c 15. xy c.
y 2 2 3
Self-Instructional Material 79
Ordinary Differential 11 19
Equations 7 y 7
I.F. = x
11 19
Multiplying the given equation by x 7 y 7 , we have
NOTES
3
5 11 12 4 19 10 12
dy = 0
2x
y x 7 y
dx + 3 x y
7
7
7
7
7 x7 y
7
EXERCISE E
Solve the following differential equations:
1. (x2y + y4)dx + (2x3 + 4xy3)dy = 0 2. (y2 + 2x2y)dx + (2x3 xy)dy = 0
3. (2x2y 3y4)dx + (3x3 + 2xy3)dy =0 4. (y3 2x2y)dx + (2xy2 x3)dy = 0
5. (2ydx + 3xdy) + 2xy(3ydx + 4xdy) = 0 6. x(3ydx + 2xdy) + 8y4 (ydx + 3xdy) = 0
7. (2y2 4x2y) dx + (4xy + 3x3) dy = 0.
Answers
3/2
y
1. 7x11/2y11 + 11x7/2y1/4 = c (I.F. = x5/2y10) 2. 6 xy c (I.F. = x5/2y1/2)
x
3. 5x36/13 y24/13 12x10/13 y15/13 = c (I.F. = x49/13y28/13)
4. x2y4 y2x4 = c (I.F. = xy) 5. x2y3 (1 + 2xy) = c (I.F. = xy2)
2 4 20 5
11 y 11 x 11 y 11 c (I.F. = x13/11y26/11)
6. x3y2 + 4x2y6 = c (I.F. = xy) 7. 5x
80 Self-Instructional Material
UNIT III linear Differential
Equations of the
First Order
STRUCTURE
Definition
To Solve the Equation + Py = Q, where P and Q are Functions of x only
(Leibnitzs Equation)
Bernoullis Equation (Equations Reducible to the Linear Form)
Differential equations of the first order and higher degree
Equations Solvable For p
Equations Solvable For y
Equations Solvable For x
Clairauts Equation
DEFINITION
dy
The given equation is + Py = Q
dx
P dx
Multiplying throughout by e I
, we get
dy P dx P dx P dx
.e I
+ Py . e I
=Q. e I
...(1)
dx
Self-Instructional Material 81
Ordinary Differential dy d
d P dx P dx P dx
Equations Now, [y e I
]= .e I
+y. [e I
]
dx dx dx
dy P dx P dx d
= .e I
+y. e I
. [ P dx]
NOTES dx dx
' d
> C
e f ( x) e f ( x) .
d
; @"#$
! dx dx
f ( x)
dy P dx P dx dy P dx P dx
= .e I
+y. e I
.P= .e I
+ Py . e I
dx dx
d
From (1), [y . e P dx ] = Q . e I I P dx
dx
Integrating both sides w.r.t. x, we have
y . e P dx = Q . e P dx dx + c
I I
Note 1. The factor e I P dx , on multiplying by which the LHS of the differential equation
becomes the differential co-efficient of some function of x and y, is called an integrating factor of
the differential equation and is shortly written as I.F.
dy
Note 2. The solution of the linear equation + Py = Q, where P and Q are
dx
functions of x only, is
y(I.F.) = Q(I.F.) dx + c
Note 3. Sometimes a differential equation becomes linear if we take y as the independent
variable and x as dependent variable. In that case, the equation can be put in the form
dx
+ Px = Q, where P and Q are functions of y (and not of x) or constants.
dy
elog f ( x ) = f(x).
2
Thus, elog x = x2.
dy
Note 5. The co-efficient of , if not unity, must be made unity by dividing
dx
throughout by it.
SOLVED EXAMPLES
I.F. = e
I P dx
=e
I 2x
1 x2
dx
e log (1 x
2
)
= 1 + x2
Hence the solution is
y . (I.F.) = I Q . (I.F.) dx + c
or y(1 + x2) = I 4 x2
1 x2
. (1 + x2) dx + c
4 x3
or y(1 + x2) = + c.
3
dy
(ii) Given equation is (tan x) . y = 2 sin x
dx
dy
It is of the form + Py = Q
dx
Here P = tan x, Q = 2 sin x
I.F. = e P dx = e tan x dx = e( log cos x)
I I
y (I.F.) = I Q . (I.F.) dx + c
=
1
I sin 2x dx + c =
cos 2 x
2
+c
or y cos x = 2
cos 2x + c.
Self-Instructional Material 83
Ordinary Differential Here, P = cos x, Q = sin x cos x
Equations
P dx cos x dx
I.F. = e I
=e I
= e sin x
I
Hence the solution is
NOTES y . (I.F.) = Q . (I.F.) dx + c
=t.
et
1
1.
et
1 I
dt + c = tet et + c
I.F. = e
I P dx
=e
I 1
x log x
dx
=e
I 1/x
log x
dx
= elog log x = log x
Hence the solution is
y . (I.F.) = I Q. (I.F.) dx + c
or y log x =
2
I
log x dx + c
I
x
1
or y log x = 2 . log x dx + c
x
(log x) 2
' I[ f ( x ) n
f ( x ) dx
n 1
=2. +c [ f ( x )]
2 ,n 1
n 1
or y log x = (log x)2 + c.
dy
Example 3. Solve: x(x 1) (x 2)y = x3(2x 1).
dx
Sol. Given equation is
dy
x(x 1) (x 2)y = x3(2x 1)
dx
dy x2 x 2 (2 x 1)
or y=
dx x( x 1) x1
dy
It is of the form + Py = Q
dx
84 Self-Instructional Material
linear Differential
x2 x 2 (2 x 1) Equations of the
Here, P= ,Q=
x( x 1) x1 First Order
I.F. = e I P dx
=e
I
x2
x ( x 1)
dx
e
I
2
1
dx
x x 1 NOTES
2
= e [2 log x log (x 1)] = e [log x log ( x 1)]
x
1
x2
x2 1
log 2
log x1
x 1
x1 x 1
= e =e
x2
The solution is
y.
x1
x2
= I x 2 (2 x 1) x 1
x1
.
x2
dx + c = I (2x 1) dx + c = x2 x + c
2 x2 1 x2
Here P= , Q=
x (1 x 2 ) 1 x2
2x 2 1 1 1 1
Now P= = [Partial fractions]
x(1 x)(1 x) x 2(1 x) 2(1 x)
I Pdx = log x
1
2
1
log (1 x) log (1 + x)
2
= log [x (1 x)1/2 (1 + x)1/2]
P dx log ( x 1 x 2 ) 1 1
I.F. = e =e =
I
x 1 x2
The solution is
y.
1
x 1 x2
= I x2
1 x 2
.
1
x 1 x2
dx + c
= I (1 x )
x
2 3/ 2
dx + c =
1
2 I (1 x2)3/2 . ( 2x) dx + c
Self-Instructional Material 85
Ordinary Differential
Equations 1 (1 x 2 ) 1/ 2
= . +c
2 1
2
NOTES y 1
= +c y = x + cx 1 x2
2
x 1 x 1 x2
which is the required solution.
e 2 x dx = 1.
Example 5. Solve: x
y
x dy
dy e 2 x y dy 1 e 2 x
or = or + y
dx x x dx x x
dy
It is of the form + Py = Q
dx
1 e 2 x
Here P= , Q=
x x
I.F. = e
I 1
x
dx
=e I x 1/ 2 dx
= e
2 x
y. e
2 x
= I e 2
x
x
.e
2 x
dx + c
or y. e
2 x
= I 1
x
dx + c
or ye2 x
= 2 x +c or y = e 2 x
(2 x + c).
dx
Equations of the Form + Px = Q where P and Q are functions of y
dy
only.
Example 6. Solve the following:
tandy 1
y dy
(i) (1 + y2) + ( x e =0 ) (ii) (2x 10y3) + y = 0.
dx dx
Sol. (i) The given equation is
1 dy
(1 + y2) + ( x e tan y
) =0
dx
dx tan 1 y
or (1 + y2) +x e =0
dy
1
dx 1 e tan y
or + . x =
dy 1 y2 1 y2
86 Self-Instructional Material
dx linear Differential
It is of the form + Px = Q Equations of the
dy First Order
I.F. = e
I 1
1 y2
dy
=e
tan 1
y
NOTES
The solution is
I I
1
tan 1
y e tan y
tan 1
y
x. e = 2 .e dy + c = et . et dt + c where t = tan1 y
1 y
tan 1
y
= I e2t dt + c =
1
1
2
e2t + c
or x. e = 1
2 e 2 tan y
+ c.
dy
(ii) The given equation is (2x 10y3) +y=0
dx
dx dx 2
or y. + 2x 10y3 = 0 or + . x = 10y2
dy dy y
dx
It is of the form + Px = Q
dy
2
dy
e P dx
2
log y
I.F. = = e
y
= e2 log y = e y2
or xy2 = + c = 2y5 + c.
5
EXERCISE A
Solve the following differential equations:
dy y dy
1. x2 2. y sec x tan x
dx x dx
dy dy
3. y tan x sec x 4. (1 + x2) + 2xy = cos x
dx dx
dy x y 1 dy
5. = 6. (x + 1) ny = ex (x + 1)n + 1
dx x 1 dx
dy dy
7. cos2 x + y = tan x 8. (1 + x2) + y = tan1 x
dx dx
dy 2x 1
9. .y 2 given that y = 0 when x = 1
dx x 2 1 ( x 1)2
dy
10. + 2y tan x = sin x given that y = 0 when x =
dx 3
dy dy
11. x 2 y x 2 log x 12. y cos x sin 2 x
dx dx
dy dy
13. = x(x2 2y) 14. sin x + y cos x = 2 sin2 x cos x
dx dx
Self-Instructional Material 87
Ordinary Differential dy dy
Equations 15. (1 x2) + xy = ax 16. x(x 1) y = x2(x 1)2
dx dx
dy
17. y dx x dy + log x dx = 0 18. + 2y cot x = 3x2 cosec2 x
dx
NOTES dy dy
19. sin 2x = y + tan x 20. (x + 2y3) =y
dx dx
21. (1 + y2) dx = (tan1 y x) dy 22. ey dx + (1 + xey) dy = 0
dx
23. + 2x = 6ey 24. 2y + 4y = x2 x
dy
dy
25. y 2y = cos 3x 26. + y cot x = 2x + x2 cot x
dx
1 x log x
27. y + y = 28. xy y = (x 1) ex
x
Answers
1
1. xy = x4 + c 2. y(sec x + tan x) = sec x + tan x x + c
4
3. y = sin x + c cos x 4. y(1 + x2) = sin x + c
y
5. = log (x + 1) + c 6. y = (x + 1)n (ex + c)
x 1
7. y = tan x 1 + cetan x 8. y = tan1 x 1 + cetan x
9. y(x2 + 1) = tan1 x 10. y = cos x 2 cos2 x
4
x4 x4
11. x2y = log x c 12. y = 2(sin x 1) + cesin x
4 16
1 2 2 2
13. y = (x 1) + ce x 14. y sin x = sin3 x + c
2 3
1 x3
15. y = a + c 1 x 2 16. y = 1 c
x 3
17. y + 1 + log x = cx 18. y sin2 x = x3 + c
BERNOULLIS EQ
EQUUATION (EQUATIONS REDUCIBLE TO
(EQU
THE LINEAR FORM)
dy
To solve the equation + Py = Qyn, where P and Q are functions of x
dx
only
dy
The given equation is + Py = Qyn ...(i)
dx
88 Self-Instructional Material
Dividing both sides of (i) by yn, to make the RHS a function of x only. linear Differential
Equations of the
dy First Order
yn + Py1n = Q ...(ii)
dx
Put y1n = z, then NOTES
dy dz dy 1 dz
(1 n) . yn = or yn = .
dx dx dx 1 n dx
1 dz
(ii) becomes . + Pz = Q
1 n dx
dz
or + (1 n) . Pz = (1 n) Q.
dx
which is a linear equation in z and can be solved.
In the solution, putting z = y1n, we get the required solution.
SOLVED EXAMPLES
dy y y2
Example 7. Solve: 2 = + 2.
dx x x
dy y y2
Sol. The given equation is 2. = 2 .
dx x x
Dividing throughout by y2
dy 1 1
2y2 . y1 = 2 ...(i)
dx x x
dy dz
Put y1 = z, then y2 =
dx dx
(i) becomes
dz 1 1 dz 1 1
2 z= 2 or + z=
dx x x dx 2x 2x2
which is linear in z.
1 1
P= ,Q=
2x 2x2
I.F. = e
I 1
2x
dx
1
e2
log x
e log x
= x
The solution is z . x = I
2x2
1
x dx + c
or y1 x =
1
2 I x3/2 dx + c or
y
x
1
x
+c
or x = y(1 + c x ).
Example 8. Solve the following :
dy 1 ey dy x
(i) + 2 (ii) + y x y.
dx x x dx 1 x2
Self-Instructional Material 89
Ordinary Differential dy 1 ey
Equations Sol. (i) The given equation is + 2
dx x x
Dividing throughout by ey
NOTES dy dy 1 1
ey + ey + = 2 ...(i)
dx dx x x
dy dz
Put ey = z, then ey =
dx dx
(i) becomes
dz 1 1 dz 1 1
+z. = 2 or .z= 2
dx x x dx x x
1 1
which is linear in z. P= ,Q= 2
x x
I.F. = e
I
1
x
dx
e log x e
log
1
x
=
1
x
The solution is z.
1
x
=
x
1
2 I
.
1
x
dx + c
or
or
1
ey . =
x
1
x3 I
dx + c
2x = ey + 2cx2ey.
or ey .
1
=
1
x 2x2
+c
dy x
(ii) The given equation is + y=x y
dx 1 x2
Dividing throughout by y,
dy x
y1/2 . + y1/2 = x ...(i)
dx 1 x2
1 1/2 dy dz
Put y1/2 = z ; then y .
2 dx dx
(i) becomes
dz x dz x x
2. + .z=x or
+ .z
dx 1 x2 dx 2
2(1 x ) 2
x x
which is linear in z. P= ,Q
2(1 x 2 ) 2
I.F. = e
I x
2( 1 x 2 )
dx
e
1
4 I 2x
1 x2
dx
| Note
1
log ( 1 x 2 ) 2 1 / 4
= e 4 e log ( 1 x )
= (1 x2)1/4
or y . (1 x2)1/4 =
1
4 I
2x(1 x2)1/4 dx + c
1 (1 x 2 ) 3 / 4
or y . (1 x2)1/4 = . +c
4 3
4
1
or y = (1 x2) + c(1 x2)1/4.
3
90 Self-Instructional Material
Example 9. Solve: (x2y3 + xy) dy = dx. linear Differential
Equations of the
Sol. The given equation is (x2y3 + xy)dy = dx First Order
dx
or = x2y3 + xy
dy NOTES
dx dx
or xy = x2y3 Form + Px = Qxn
dy dy
Dividing throughout by x2
dx
x2 x1 y = y3 ...(i)
dy
dx dz
Put x1 = z, then x2 =
dy dy
(i) becomes
dz dz
zy = y3 or + y.z = y3
dy dy
which is linear in z. P = y, Q = y3
2
y dy y /2
I.F. = e =e
I
I
2 2
The solution is z . e 1/2 y = y3 . e 1/2 y dy + c
or
2
x1 . e 1/2 y = I 2
y 2 . y . e 1/2 y dy c
or
2
= I 2t et dt + c,
I.F. = e
I 1
x1
dx
e log ( x 1) = x + 1
or
The solution is z (x + 1) =
2
x1
cy . (x + 1) = 2x + c.
I. (x + 1) dx + c
Self-Instructional Material 91
Ordinary Differential (ii) The given equation is
Equations dy dy
= exy (ex ey) or = e2x . ey ex
dx dx
dy dy
or + ex = e2x . ey or ey . + ex . ey = e2x ...(i)
NOTES dx dx
dy dz
Put ey = z, then ey . =
dx dx
dz
(i) becomes + ex . z = e2x
dx
which is linear in z. P = ex, Q = e2x
e x dx x
I.F. = e ee
I
I
x x
The solution is z . ee = e2x . e e dx + c
or
x
ey . e e = I ex . ex . e e dx + c
x
= I t et dt + c,
= et (t 1) + c
where t = ex
x x x
or ey . e e = e e (ex 1) + c or ey = ex 1 + c e e .
dy
Example 11. Solve: + (2x tan1 y x3)(1 + y2) = 0.
dx
Sol. The given equation is
dy
+ (2x tan1 y x3)(1 + y2) = 0
dx
1 dy
or . + 2x tan1 y x3 = 0
1 y 2 dx
1 dy
or . + 2x tan1 y = x3 ...(i)
1 y 2 dx
1 dy dz
Put tan1 y = z, then 2
. =
1 y dx dx
dz
From (i), + 2xz = x3
dx
which is linear in z. P = 2x, Q = x3
2
2 x dx x
=e
I
I.F. = e I
2 2
x
The solution is z . ex = x3 . e dx + c
or
x
tan1 y . e =
2
1
I x
2x . x2 e dx + c
2
I
2
1
= 2
t et dt + c, where t = x2
1
= 2 et (t 1) + c
2 2
x
or tan1 y . e = 1
2
e x (x2 1) + c
2
or tan1 y = 1
2
(x2 1) + c e x .
Example 12. Solve the following differential equations:
dy y y
(i) (x3y2 + xy) dx = dy (ii) + log y 2 (log y)2
dx x x
92 Self-Instructional Material
Sol. (i) The given equation is (x3y2 + xy) dx = dy linear Differential
dy dy Equations of the
or = x3y2 + xy or xy = x3y2 First Order
dx dx
dy
Dividing both sides by y2, y2 xy1 = x3 ...(i)
dx NOTES
dy dz
Put y1 = z, then y2 =
dx dx
dz dz
(i) becomes xz = x3 or + xz = x3
dx dx
which is linear in z. P = x, Q = x3
x2
x dx
I.F. = e
I
e2
I I
2
x x2 x2
The solution is z. e2 = x3 . e2 dx + c = x2 . xe 2 dx + c
= I 2tet dt + c, where t =
x2
I
2
= 2tetdt + c = 2et (t 1) + c
x2 x2 x 1
2
y1 . e2 = 2e 2 2 + c
x2
or = + 2 + ce y1 . x2 2
dy y y
(ii) The given equation is + log y = 2 (log y)2
dx x x
Dividing both sides by y (log y)2, we get
1 dy 1 1 1
. + . 2 ...(i)
y(log y) 2 dx log y x x
1 1 dy dz
Put = (log y)1 = z, then (log y)2 . =
log y y dx dx
1 dy dz
or . =
y(log y) 2 dx dx
dz 1 1 dz 1 1
From (i), +z. = 2 or z= 2
dx x x dx x x
1 1
which is linear in z. P= ,Q= 2 .
x x
I.F. = e
I
1
x
dx
e log x e log x
1
= x1 =
1
x
The solution is z.
1
x
= I
1 1
. dx + c
x2 x
or z.
1
x
= I x3 dx + c =
x 2
2
+c
1 1 1 1 1
or . = +c or = + cx.
log y x 2 x2 log y 2x
Self-Instructional Material 93
Ordinary Differential Example 13. Show how to solve an equation of the form
Equations
dy
f (y)
+ Pf(y) = Q where P, Q are functions of x only.
dx
Sol. (a) The given equation is
NOTES
dy
f (y)
+ Pf(y) = Q ...(i)
dx
where P, Q are functions of x only.
dy dz
Put f(y) = z, then f (y) =
dx dx
dz
(i) becomes + Pz = Q
dx
which is linear in z and can be solved.
z(I.F.) = Q . (I.F.) dx + c
or f(y) . (I.F.) = Q . (I.F.) dx + c
Example 14. Solve the following differential equations:
dy dy
(i) (x + 1) + 1 = exy (ii) = y tan x y2 sec x.
dx dx
Sol. (i) The given equation is
dy ex dy ey ex
(x + 1) +1= y or ey + = ...(i)
dx e dx x1 x1
dy dz
Putting ey = z so that ey =
dx dx
dz z ex
(i) becomes + =
dx x1 x1
1 ex
which is linear in z with P= ,Q=
x1 x1
I.F. = e
I P dx
e
I 1
x1
dx
e log ( x 1) = x + 1
The solution is z (x + 1) = I ex
x1
. (x + 1) dx + c or ey (x + 1) = ex + c.
1 1 dy dz
Putting = z so that 2 =
y y dx dx
94 Self-Instructional Material
dz linear Differential
Equation (1) becomes + z tan x = sec x Equations of the
dx First Order
which is linear in z with P = tan x, Q = sec x
P dx tan x dx
I.F. = e
I
e I
e log sec x = sec x NOTES
The solution is z . sec x = I sec x . sec x dx + c
1 1
or sec x = tan x + c or = sin x + c cos x.
y y
EXERCISE B
Solve the following differential equations:
dy y
1. = y2 2. y + y = y2
dx x
dy dy 2 x3
3. = x3y3 xy 4. 3 y 2
dx dx x 1 y
dy y dy
5. = x2y6 6. x + y = x3y4
dx x dx
dy
7. 2y tan x = y2 tan2 x 8. (y log x 1)y dx = x dy
dx
dy dy tan y
9. + x sin 2y = x3 cos2 y 10. = (1 + x) ex sec y
dx dx 1 x
11. (x y2) dx + 2xy dy = 0 12. cos x dy = y(sin x y) dx
dy 2
y3 e x
3
13. xy 14. ( xy2 e1/ x ) dx x2y dy = 0
dx
dy
15. ey 1 ex 16. (xy 2x log x) dy = 2y dx
dx
dy y
17. = y2 log x 18. y (2xy + ex) dx = exy
dx x
dy dy
19. x + y = y2x3 cos x 20. sin y = cos y (1 x cos y)
dx dx
Answers
1 1
1. + log x = c 2. y =
xy 1 ce x
x 6 2x5 x 4
3. y2 = x2 + 1 + ce x
2
4. y2(x + 1)2 = c
6 5 4
1 5 3 1
5. x + cx5 6. = 3x3 log x + cx3
y5 2 y3
1 tan3 x 1
7. sec2 x c 8. = log x + 1 + cx
y 3 y
1 2 3
9. tan y = ( x 1) ce x 10. sin y = (1 + x)(ex + c)
2
1
11. y2 = x (c log x) 12. = sin x + c cos x
y
Self-Instructional Material 95
Ordinary Differential 1
Equations 2
13. y2 . e x = 2x + c 14. 3y2 = 2x 2e x cx 2
3
1 2x y2
15. ex y e c 16. y log x = +c
2 4
NOTES 1 1
17. (log x )2 cx 18. ex = y(c x2)
y 2
1
19. = x sin x cos x + c 20. sec y = x + 1 + cex
xy
So far, we have discussed differential equations of the first order and first degree.
Now we shall study differential equations of the first order and degree higher than the
dy
first. For convenience, we denote by p.
dx
A differential equation of the first order and nth degree is of the form
pn + P1pn1 + P2pn2 + ...... + Pn = 0 ...(1)
where P1, P2, ......, Pn are functions of x and y.
Since it is a differential equation of the first order, its general solution will
contain only one arbitrary constant.
In the various cases which follow, the problem is reduced to that of solving one
or more equations of the first order and first degree.
EQ
EQUUATIONS SOLVABLE FOR p
SOLV
Resolving the left hand side of (1) into n linear factors, we have
[p f1(x, y)] [p f2(x, y)], ......, [p fn(x, y)] = 0
which is equivalent to p f1(x, y) = 0, p f2(x, y) = 0, ......, p fn(x, y) = 0
Each of these equations is of the first order and first degree and can be solved
by the methods already discussed.
If the solutions of the above n component equations are
F1 (x, y, c) = 0, F2 (x, y, c) = 0, ......, Fn (x, y, c) = 0
then the general solution of (1) is given by
F1 (x, y, c) F2 (x, y, c) ...... Fn (x, y, c) = 0.
SOLVED EXAMPLES
dy 2
dy
Example 15. Solve: x2
dx xy
dx
6y2 = 0.
96 Self-Instructional Material
dy linear Differential
Sol. The given equation is x2p2 + xyp 6y2 = 0 where p = Equations of the
dx
First Order
Factorising (xp + 3y) (xp 2y) = 0
xp + 3y = 0 or xp 2y = 0
NOTES
Now, xp + 3y = 0
dy dy dx
x + 3y = 0 or 3 =0
dx y x
Integrating, log y + 3 log x = log c or x3y = c
Also, xp 2y = 0
dy dy dx
x 2y = 0 or 2 =0
dx y x
y
Integrating, log y 2 log x = log c or = c or y = cx2
x2
The general solution of the given equation is (x3y c) (y cx2) = 0.
Example 16. Solve xyp2 + p(3x2 2y2) 6xy = 0.
Sol. Solving the given equation for p, we have
(3 x 2 2 y 2 ) (3 x 2 2 y 2 ) 2 24 x 2 y 2
p=
2 xy
(2 y 2 3 x 2 ) (3 x 2 2 y2 ) 2 y 3x
= or
2 xy x y
2y dy 2 y dy 2dx
Now, p= or =0
x dx x y x
y
Integrating, log y 2 log x = log c or = c or y = cx2
x2
3x dy 3x
Also, p= or ydy + 3xdx = 0
y dx y
y2 3 x 2
Integrating, = C or y2 + 3x2 = c
2 2
The general solution of the given equation is (y cx2) (y2 + 3x2 c) = 0.
Example 17. Solve p2 + 2py cot x = y2.
Sol. The given equation can be written as (p + y cot x)2 = y2 (1 + cot2 x)
or p + y cot x = y cosec x
The component equations are
p= y ( cot x + cosec x) ...(1)
and p= y ( cot x cosec x) ...(2)
dy
From (1), = y ( cot x + cosec x)
dx
dy
or = ( cot x + cosec x) dx
y
x
x c tan
Integrating, log y = log sin x + log tan + log c = log 2
2 sin x
Self-Instructional Material 97
Ordinary Differential x
Equations c tan
2 c c
or y=
x x x 2 1 cos x
2 sin cos 2 cos
2 2 2
NOTES or y(1 + cos x) = c
dy
From (2), = y( cot x cosec x)
dx
dy
or = ( cot x cosec x) dx
y
x c
Integrating, log y = log sin x log tan + log c = log
2 x
sin x tan
2
c c
or y= or y(1 cos x) = c
x 1 cos x
2
2 sin
2
The general solution of the given equation is
[y(1 + cos x) c] [y(1 cos x) c] = 0.
EXERCISE C
Solve the following equations:
dy 2
dy
1. p2 7p + 12 = 0 2. xy
dx (x2 + y2)
dx
+ xy = 0
dy 2
dy
3. yp2 + (x y)p x = 0 4. x2
dx + 3xy
dx
+ 2y2 = 0
dy dx x y
5. 6. p2 2p sinh x 1 = 0
dx dy y x
7. p (p + y) = x (x + y) 8. 4y2p2 + 2pxy (3x + 1) + 3x3 = 0.
Answers
1. (y 4x c) (y 3x c) = c 2. (y2 x2 c) (y cx) = 0 3. (y x c) (x2 + y2 c)
=0
4. (xy c)(x2y c) = 0 5. (xy c)(x2 y2 c) = 0 6. (y ex c) (y ex c)
=0
1 2 1 2
7. (y x + c) (y + x + cex 1) = 0 8. (y2 + x3 c)(y2 + x c) = 0.
2 2
EQ
EQUUATIONS SOLVABLE FOR y
SOLV
98 Self-Instructional Material
Now elimination of p from (1) and (3) gives the required solution. linear Differential
Equations of the
If p cannot be easily eliminated, then we solve equations (1) and (3) for x and y
First Order
to get
x = 1 (p, c), y = 2 (p, c)
NOTES
These two relations together constitute the solution of the given equation with
p as parameter.
SOLVED EXAMPLES
dp
Discarding the factor (1 2px3), which does not involve , we have
dx
dp dp dx
2p + x = 0 or 2 =0
dx p x
Integrating, log p + 2 log x = log c
or log px2 = log c
c
or px2 = c .p=
x2
c
Putting this value of p in (1), we have y = + c2.
x
Example 19. Solve y = 2px p2.
Sol. The given equation is y = 2px p2 ...(1)
dy dp dp
Differentiating both sides w.r.t. x, = p = 2p + 2x 2p
dx dx dx
dp
or p + (2x 2p) =0
dx
dx
or p + 2x 2p = 0
dp
dx 2
or x=2 ...(2)
dp p
which is a linear equation.
I 2
p
dp
e 2 log p
p2
I
I.F. = e
The solution of (2) is x I.F. = 2 I.F.dp + c
or xp2 = I 2 p2dp + c
Self-Instructional Material 99
Ordinary Differential 2 2 3
Equations or xp2 = p + cp2
p + c or x= ...(3)
3 3
2
Putting this value of x in (1), we have y = 2p
3
p cp2 p2
NOTES 1
or y = p2 + 2cp1. ...(4)
3
Equations (3) and (4) together constitute the general solution of (1).
EXERCISE D
Solve the following equations:
1. xp2 2yp + ax = 0 2. y 2px = tan1 (xp2) 3. 16x2 + 2p2y p3x = 0
4. y = x + 2 tan1 p 5. y = 3x + log p 6. x yp = ap2
dy 4
dy
7. x2
dx + 2x
dx
y = 0. 8. y = 2px xp2
Answers
a
1. 2y = cx2 + 2. y = 2 cx + tan1 c 3. 16 + 2c2y c3x2 = 0
c
p1 p 1 3
4. x = log tan1 p + c, y = log + tan1 p + c 5. y = 3x + log
p2 1 2
p 1 1 ce3 x
p 1
6. x= (c + a sin1 p), y = (c + a sin1 p) ap. 7. y = c2 + 2 cx .
2
1 p 1 p2
8. y = 2 cx c.
EQ
EQUUATIONS SOLVABLE FOR x
SOLV
dx 1
dp
Differentiating (1) w.r.t. y, we get F y, p,
dy p
dy ...(2)
dx 1 1 1 y dp dp
= 2 2 yp2 2 y 2 p
dy p 2 p p dy dy
dp dp
or 2p = p y 2 yp4 2 y2 p3
dy dy
dp dp dp
or p + 2yp4 + y + 2y2p3 = 0 or p(1 + 2yp3) + y (1 + 2yp3) = 0
dy dy dy
p y dp (1 + 2yp ) = 0
dy
or 3
dp
Discarding the factor (1 + 2yp3) which does not involve , we have
dy
dp dy dp
p+y = 0 or + =0
dy y p
c
Integrating, log y + log p = log c or py = c or p=
y
Putting this value of p in the given equation, we have
2cx c 3
y= or y2 = 2cx + c3
y y
which is the required solution.
p .
Example 21. Solve p = tan x 1 p 2
p
Sol. Solving for x, we have x = tan1 p + ...(1)
1 p2
Differentiating both sides w.r.t. y,
dx 1 1 dp (1 p2 ) 2 p2 dp
dy p 1 p2 dy (1 p2 )2 dy
1 2(1 p2 ) 2 p2 dp 2p
or or dy = dp
p (1 p2 ) 2 dy (1 p2 ) 2
1
Integrating, y=c ...(2)
1 p2
Equations (1) and (2) together constitute the general solution.
Answers
1. y3 = 3cx + 6c2 2. y2 = 2cy + c2 3. 64y = c(c 4x)2
p
4. log y = cx + c2 5. x = c + a log , y = c a log (p 1)
p 1
6. x = 2p 2 log (1 p) + c, y = p2 2p 2 log (1 p) + c.
CLAIRAUTS EQ
CLAIRAUTS EQUUATION
SOLVED EXAMPLES
EXERCISE F
Solve the following equations:
a
1. y = xp + 2. y = px + a2 p2 b2
p
3. sin px cos y = cos px sin y + p 4. xp2 yp + a = 0
5. (x a)p2 + (x y)p y = 0 6. p = log (px y)
7. p = sin (y px) 8. p2(x2 1) 2pxy + y2 1 = 0
9. e3x (p 1) + p3e2y = 0 10. x2 (y px) = yp2
11. (y + px)2 = x2p.
Answers
a
1. y = cx + 2. y = cx + a2c2 b2 3. y = cx sin1 c
c
a ac 2
4. y = cx + 5. y = cx 6. y = cx ec
c c 1
7. y = cx + sin1 c 8. (y cx)2 = 1 + c2 9. ey = cex + c2
10. y2 = cx2 + c2 [Hint. Put x2 = X, y2 = Y] 11. xy = cx c2. [Hint. Put xy = v]
STRUCTURE
Definitions
The Operator D
Theorems
Auxiliary Equation (A.E.)
Rules for Finding The Complementary Function
The Inverse Operator
Rules for Finding The Particular Integral
Method of Variation of Parameters to Find P.I.
Homogeneous Linear Equations (Cauchy-Euler Equations)
Legendres Linear Differential Equation
Linear Differential Equations of Second Order
Complete Solution in Terms of Known Integral
To Find a Particular Integral of + P + Qy = 0
Removal of the First Derivative (Ruduction to Normal Form)
Transformation of theEquation by Changing the Independent Variable
Method of Variation of Parameters
DEFINITIONS
THE OPERATOR D
d dy
The part of the symbol may be regarded as an operator such that when
dx dx
it operates on y, the result is the derivative of y.
d2 d3 dn
Similarly, , , ...... , may be regarded as operators.
dx 2 dx 3 dx n
d d2 2 dn
For brevity, we write D, D , ......, Dn
dx dx 2 dx n
Thus, the symbol D is a differential operator or simply an operator.
Written in symbolic form, equation (1) becomes
(Dn + a1Dn1 + a2Dn2 + ...... + an1D + an)y = X
or f(D)y = X
where f (D) = Dn + a1 Dn1 + a2Dn2 + ...... + an1D + an
i.e., f (D) is a polynomial in D.
The operator D can be treated as an algebraic quantity.
Thus D(u + v) = Du + Dv
D(u) = Du
DpDq u = Dp+q u
DpDqu = DqDpu
The polynomial f(D) can be factorised by ordinary rules of algebra and the factors
may be written in any order.
THEOREMS
Theorem 1
If y = y1, y = y2,..., y = yn are n linearly independent solutions of the differential
equation
(Dn + a Dn1 + a Dn2 + ... + a )y = 0
1 2 n ...(i)
then u = c1y1 + c2y2 + ... + cnyn is also its solution, where c1, c2, ..., cn are arbitrary
constants.
Theorem 2
If y = u is the complete solution of the equation f(D)y = 0 and y = v is a particular
solution (containing no arbitrary constants) of the equation f(D)y = X, then the complete
solution of the equation f(D)y = X is y = u + v.
Proof. Since y = u is the complete solution of the equation f(D)y = 0 ...(i)
f(D)u = 0 ...(ii)
Also y = v is a particular solution of the equation f(D)y = X ...(iii)
f(D)v = X ...(iv)
Adding (ii) and (iv), we have f(D)(u + v) = X
Thus y = u + v satisfies the equation (iii), hence it is the complete solution
(C.S.) because it contains n arbitrary constants.
The part y = u is called the complementary function (C.F.) and the part y = v
is called the particular integral (P.I.) of the equation (iii).
The complete solution of equation (iii), is y = C.F. + P.I.
Thus in order to solve the equation (iii), we first find the C.F. i.e., the C.S. of
equation (i) and then the P.I. i.e., a particular solution of equation (iii).
its solution is I
y . e m1 x 0 . e m1 x dx c1 or y = c1e m1 x
m x
Similarly, the solution of (D m2)y = 0 is y = c2 e 2
................................................................
the solution of (D mn)y = 0 is y = cn e mn x
Hence the complete solution of equation (i) is
or
its solution is y . e m1 x I c1e m1 x . e – m1 x dx + c2 = c1x + c2
y = (c1x + c2) e m1 x
Thus, the complete solution of equation (i) is
If, however, three roots of the A.E. are equal, say m1 = m2 = m3, then proceeding
as above, the solution becomes
x i x
= e (c1e c2 e
i x
) c3em3x ...... cnemn x
= ex[c1(cos x + i sin x) + c2 (cos x i sin x)]
y = ex [(c1x + c2) cos x + (c3x + c4) sin x] + c5e 5 ...... cn e n .
mx m x
d4 y d2 y
Example 3. Solve: 13 + 36y = 0.
dx 4 dx 2
Sol. Given equation in symbolic form is (D4 + 13D2 + 36)y = 0
Its A.E. is D4 + 13D2 + 36 = 0
or (D2 + 4)(D2 + 9) = 0 D = 2i, 3i
Hence the C.S. is y= e0x (c1 cos 2x + c2 sin 2x) + e0x (c3 cos 3x + c4 sin 3x)
or y= c1 cos 2x + c2 sin 2x + c3 cos 3x + c4 sin 3x.
d4 x
Example 4. Solve: + 4x = 0.
dt 4
d
Sol. Given equation in symbolic form is (D4 + 4)x = 0, where D =
dt
Its A.E. is D4 + 4 = 0 or (D4 + 4D2 + 4) 4D2 = 0
or (D2 + 2)2 (2D)2 = 0 or (D2 + 2D + 2)(D2 2D + 2) = 0
2 4 2 4
whence D= and i.e., D = 1 i and 1 i
2 2
Hence the C.S. is x = et (c1 cos t + c2 sin t) + et (c3 cos t + c4 sin t).
Example 5. Solve: y 2y + 10y = 0, given y(0) = 4, y(0) = 1.
Sol. Given equation in symbolic form is
(D2 2D + 10)y = 0
Its A.E. is D2 2D + 10 = 0
2 4 40 2 6i
D= = 1 ± 3i
2 2
The C.S. is y = ex (c1 cos 3x + c2 sin 3x) ...(1)
Now y(0) = 4 y = 4, when x = 0
4 = c1
Equation (1) becomes y = ex (4 cos 3x + c2 sin 3x) ...(2)
so that y = ex (4 cos 3x + c2 sin 3x) + ex ( 12 sin 3x + 3c2 cos 3x)
Since y(0) = 1 i.e., y = 1, when x = 0
1 = 4 + 3c2 c2 = 1
Equation (2) becomes y = ex (4 cos 3x sin 3x), which is the required particular
solution.
d2 y dy d2 y dy
1. 4 5y 0. 2. ( a b) aby 0 .
NOTES dx 2 dx dx 2 dx
d2 y dy d2 x dx
3. 2
4 y 0. 4. 8 16 x 0 .
dx dx dt 2 dt
3 2
d y d y dy d3 y d2 y dy
5. 3
3 2
3 y 0. 6. 3
6 11 6y 0 .
dx dx dx dx dx2 dx
d4 y d2 y d4 y d2 y
7. 4
5 2
4y 0. 8. 6 9y 0 .
dx dx dx4 dx2
d3 y
9. (D2 + 1)3 (D2 + D + 1)2 y = 0. 10. y 0.
dx3
d2 y
11. + y = 0, given that y(0) = 2 and y = 2.
dx 2 2
d2 x dx dx
12. 3 + 2x = 0, given that, when t = 0, x = 0 and 0.
2 dt dt
dt
d2 y dy dy
13. 4 + 29y = 0, given that, when x = 0, y = 0 and = 15.
dx2 dx dx
d4 x
14. If = m4x, show that x = c1 cos mt + c2 sin mt + c3 cosh mt + c4 sinh mt.
dt4
15. Solve the differential equation: 9y+ 3y 5y + y = 0.
d3 y d2 y dy
16. Solve the differential equation 6 12 8 y = 0 under the conditions y(0) =
3
dx dx 2 dx
0, y(0) = 0 and y(0) = 2.
d2i R di i
17. Solve the differential equation 2
= 0, where R2C = 4L and R, C, L are
dt L dt LC
constants.
Answers
1. y = c1 e5x + c2ex 2. y = c1 eax + c2ebx
3. y = c1 e(2 3)x
c2 e(2 3)x
4. x = (c1 + c2t) e4t
5. y = (c1 + c2x + c3x2) ex 6. y = c1ex + c2e2x + c3e3x
7. y = c1ex + c2ex + c3e2x + c4e2x 8. y = (c1 + c2x) cos 3 x + (c3 + c4x) sin 3 x
9. y = (c1 + c2x + c3x ) cos x + (c4 + c5x + c6x2) sin x
2
1
x
2
(c c8 x) cos
3
x (c9 c10 x) sin
3
x
"#
+e
! 7
2 2 #$
3x 3x
10. y = c1ex + ex/2 c2 cos 2
c3 sin
2 11. y = 2 (cos x sin x)
1
x
12. x=0 13. y = 3e2x sin 5x 15. y = c1ex + (c2 + c3x) e 3
Rt
16. y = x2e2x 17. i = (c1 + c2t) e 2L .
NOTES
1
Definition. X is that function of x, free from arbitrary constants, which
f (D)
when operated upon by f(D) gives X.
%& 1 X() X
Thus f(D)
' f (D) *
1
f(D) and are inverse operators.
f (D)
1
Theorem 1. X is the particular integral of f(D)y = X.
f ( D)
Proof. The given equation is f(D)y = X ...(1)
1
Putting y = X in (1), we have
f (D)
%& 1 X () = X
f(D)
' f (D) * or X = X
which is true.
1
y= X is a solution of (1).
f (D)
Since it contains no arbitrary constants, it is the particular integral of f(D) y = X.
Theorem 2.
1
D
X I X dx.
1
Proof. Let Xy
D
1 X = Dy dy
Operating both sides by D, we have D
D or X=
dx
Integrating both sides w.r.t. x
y= I X dx ,
1
no arbitrary constant being added since y = X contains no arbitrary constant.
D
1
D
X= I X dx.
Theorem 3.
1
Da
X e ax I Xe ax dx .
1
Proof. Let X y
D a
I
= e ax
or y = eax I X eax dx
Hence,
1
Da
X e ax I e ax X dx .
I
Higher Order
1 1 1 ax
= . e ax e e ax . e ax dx [By
(a) D a (a) NOTES
Theorem 3]
=
1 ax
(a)
e I 1 dx x .
1 ax
(a)
e ...(ii)
SOLVED EXAMPLES
1
=5. e0 x (replacing D by 0)
002
5
= .
2
Example 8. Find the P.I. of (D3 3D2 + 4)y = e2x.
1
Sol. P.I. = 3 2
e2 x .
D 3D 4
Here the denom. vanishes, when D is replaced by 2. It is a case of failure.
We multiply the numerator by x and differentiate the denominator w.r.t. D.
1
P.I. = x . e2 x 2
3D 6D
It is again a case of failure. We multiply the numerator by x and differentiate
the denominator w.r.t. D.
1 1 x2 2x
P.I. = x2 . e2 x x 2 . e2x e .
6D 6 6(2) 6 6
1 4D
= sin (2x + 3) [Putting D2 = 22]
1 16( 2 2 )
1
= [sin (2x + 3) + 4D sin (2x + 3)]
65
1 ' d "#
=
65
[sin (2x + 3) + 8 cos (2x + 3)]
! D=
dx $
Example 10. Find the P.I. of (D2 + 4)y = cos 2x.
1
Sol. P.I. = 2
cos 2 x
D 4
Here the denominator vanishes when D is replaced by 22 = 4. It is a case of
failure. We multiply the numerator by x and differentiate the denominator w.r.t. D.
P.I. = x .
1
2D
cos 2 x
x
2
cos 2 x dx I 1
D
'
f ( x) f ( x) dx
! I "#
$
x
= sin 2x.
4
Case III. When X = xm, m being a positive integer.
1
Here, P.I. = xm
f (D)
Take out the lowest degree term from f(D) to make the first term unity (so that
Binomial Theorem for a negative index is applicable).
The remaining factor will be of the form 1 + (D) or 1 (D)
Take this factor in the numerator. It takes the form
[1 + (D)]1 or [1 (D)]1
Expand it in ascending powers of D as far as the term containing Dm, since
Dm+1 (xm) = 0, Dm+2 (xm) = 0 and so on.
Operate on xm term by term.
Example 11. Find the P.I. of (D2 + 5D + 4)y = x2 + 7x + 9.
1 1
( x 2 7 x 9)
Sol. P.I. = 2
D 5D 4 5D D 2 (x2 + 7x + 9)
4 1
4
4
1 5D D 2 "# 1
! #$
= 1 (x2 + 7x + 9)
4 4 4
1 5D D 2
25D 2
= 1 ...... (x + 7x + 9)
NOTES
4
2
4 4 16
1 5D 21D 2
= 1 ...... (x + 7x + 9)
4
2
4 16
1 5 21 "
2 2
D ( x 7 x 9)#
2 2
4!
( x 7 x 9) D ( x + 7 x 9) +
=
4 16 $
1 21 " 1 23
(2)# x x
$ 4 2 8 .
2 5 9 2
4!
= ( x 7 x 9) (2 x 7)
4 16
Case IV. When X = eax V, where V is a function of x.
Let u be a function of x, then by successive differentiation, we have
D(eax u) = eax Du + a eax u = eax (D + a)u
D2 (eax u) = D [eax (D + a) u] = eax (D2 + aD) u + aeax (D + a)u
= eax (D2 + 2aD + a2) u = eax (D + a)2 u
Similarly, D3 (eax u) = eax (D + a)3 u
In general, Dn (eax u) = eax (D + a)n u
f(D) (eax u) = eax f (D + a) u
1
Operating on both sides by ,
f (D)
1 1
[f(D) (eax u)] = [eax f(D + a)u]
f (D) f (D)
1
eax u = [eax f(D + a)u] ...(i)
f (D)
1
Now let f(D + a) u = V, i.e., u= V
f (D + a)
1 1
From (i), we have eax V= (e ax V )
f (D + a) f (D)
1 1
or (eax V) = eax V.
f (D) f (D a)
1
Thus eax which is on the right of may be taken out to the left
f (D)
provided D is replaced by D + a.
Example 12. Find the P.I. of (D2 4D + 3)y = ex cos 2x.
1 1
Sol. P.I. = 2
ex cos 2x = ex cos 2x
2
D 4D 3 (D 1) 4(D 1) 3
1 1
= ex cos 2x = ex 2
cos 2x [Putting D2 = 22]
2
D 2D 2 2D
1 x 1 1 2D
= e cos 2x = e x cos 2 x
2 2D 2 (2 D)(2 D)
1 x 1 x
= e (2 cos 2x D cos 2x) = e (2 cos 2x + 2 sin 2x)
16 16 NOTES
1 x
= e (cos 2x + sin 2x).
8
Case V. When X is any other function of x.
Resolve f(D) into linear factors.
Let f(D) = (D m1)(D m2) ...... (D mn)
1 1
Then P.I. = X= X
f (D) (D m1 )(D m2 ) ...... (D mn )
A1 A2 An
= ...... X (Partial Fractions)
D m1 D m2 D m2
1 1 1
= A1 X A2 X ...... A n X
D m1 D m2 D mn
I
= A 1 e m1 x X e m1 x dx A 2 e m2 x I X e m2 x dx ...... A n e mn x I X e mn x dx
=8
1 e2 x
1
sin 2 x
1
x2
"#
! (D 2) 2
(D 2) 2
(D 2) 2
$
1 1
Now, 2
e2x = x . e2x | Case of failure
(D 2) 2 (D 2)
1 2x
= x2 . e | Case of failure
2
x 2 2x
= e
2
1 1 1
2
sin 2 x 2
sin 2 x 2
sin 2x
(D 2) D 4D 4 2 4D 4
[Putting D2 = 22]
=
1
4D
sin 2 x
1
4
sin 2 x dx
1
4
Icos 2 x
2
1
cos 2x
8
1 1 1 1 D 2
x2
(D 2) 2
x2
(2 D) 2
x2
D 2
x2
4
1
2
4 1
2
1 D ( 2)( 3) D ......"# x 2
! #$
= 1 2 2
4 2 2 2
1 3 "
1 D D ......# x 2 2
4! $
=
4
1 3 "
x D ( x ) D ( x )#
2 2 2 2
4! $
=
4
x e 1 cos 2 x 1 x 2 x 3 "#
2
4 2 $
2x 2
P.I. = 8
!2 8
= 4x2 e2x + cos 2x + 2x2 + 4x + 3
Hence the C.S. is y = (c1 + c2x) e2x + 4x2 e2x + cos 2x + 2x2 + 4x + 3.
=
1
(e2x + ex ex)
' sinh x
e x e x "#
(D 2)(D 1) 2
! 2 $
118 Self-Instructional Material
Now
1
e 2 x
1 1
e 2 x
1 "# 1
e 2 x
"# Linear Differential
Equations of Second and
(D 2)(D 1) 2 !
D 2 (D 1) 2
$
D 2 ( 2 1) 2 ! $ Higher Order
1 1
= . e2x | Case of failure
9 D2 NOTES
1 1 2 x x 2 x
= x. e e
9 1 9
1
ex
1 1
ex
1 1 ex
"# "#
(D 2)(D 1) 2 2
(D 1) D 2 2
!
(D 1) 1 2 $ ! $
1 1
= . ex | Case of failure
3 (D 1) 2
1 1
= .x ex | Case of failure
3 2 (D 1)
1 2 1 x 1 2 x
= .x . e x e
3 2 6
1 1 1
ex e x ex
(D 2)(D 1) 2 (1 2)( 1 1) 2
4
x 2x x 2 x 1 x
e + P.I. = e + e
9 6 4
x x 2 x 1 x
Hence the C.S. is y = c1e2x + (c2 + c3x)ex + e2x + e + e .
9 6 4
d2 y
Example 16. Solve 4y = x sinh x.
dx 2
Sol. Given equation in symbolic form is (D2 4)y = x sinh x
A.E. is D2 4 = 0 so that D=2
C.F. = c1 e2x + c2e2x
1 1
e x e– x
P.I. = 2
D 4
x sinh x =
D2 4
x
2
=
1 1
ex . x 2
1
ex . x
"#
!2 D2 4 D 4 $
1 "
x#
1x 1 x
= e xe
2 ! (D 1) 4 (D 1) 4 $
2 2
1 e 1
x
xe
1
x#
" x
2 ! D 2D 3 D 2D 3 $
= 2 2
1 e 1x 1 "
x #
x
=
31
2D D xe
2
2D D ## 2
3
3 1
3
2
! 3
3
$
%K 2D D (K %K1 2D D (K x"#
2
1
2
1
e &1 ) &K 3 3 )K #
1 x x
! K' 3 3 K*
= x e
6 ' * $
Self-Instructional Material 119
Ordinary Differential
1 x 2D 2D
Equations = e 1 ...... x e x 1 ...... x
6 3 3
1 x 2 x 2
NOTES = e x 3 e x 3
6
x e x ex 2 e x e x x 2
=
3 2
9 2 3 9
sinh x cosh x
x 2
Hence the C.S. is y = c1e2x + c2e2x sinh x cosh x.
3 9
d4 y
Example 17. Solve y = cos x cosh x.
dx 4
Sol. Given equation in symbolic form is (D4 1)y = cos x cosh x
A.E. is D4 1 = 0 or (D2 1)(D2 + 1) = 0 so that D = 1, i
C.F. = c1 ex + c2 ex + e0x (c3 cos x + c4 sin x)
= c1 ex + c2 ex + c3 cos x + c4 sin x
e e
cos x
1
x x
2
1
P.I. = 4
cos x cosh x = 4
D 1 D 1
1 1 "
e cos x #
1 x x
= e cos x
2 !D 1 4
D 1 $ 4
1 "
cos x #
1
x 1 x
= e cos x e
2 ! (D 1) 1 4
(D 1) 1 $ 4
1 x 1 x 1 "#
= e cos x e cos x
2 ! D 4D 6D 4D 4 3
D 4D 2 4 3 2
6D 4D $
1 x 1
= e cos x
2 ! ( 1 ) 4D( 1 ) 6 ( 1 ) 4D
2 2 2 2
ex
1
cos x
"#
2 2 2
( 1 ) – 4D( 1 ) 6 ( 1 ) – 4D 2
$
1 x 1 1 1 e x e x "# cos x
=
2
e
!5
cos x e x
5
cos x
5 2 $
1
= cosh x cos x
5
1
Hence the C.S. is y = c1ex + c2ex + c3 cos x + c4 sin x cos x cosh x.
5
dy d2 y
Example 18. Solve + y = xex sin x.
2
2
dx dx
Sol. Given equation in symbolic form is (D2 2D + 1)y = xex sin x
A.E. is D2 2D + 1 = 0 or (D 1)2 = 0 so that D = 1, 1
C.F. = (c1 + c2x)ex
1 1
P.I = ex . x sin x = ex . x sin x
(D 1) 2 (D + 1 1) 2
I
Higher Order
1 1 "#
= ex
D !
x ( cos x) 1( cos x) dx e x
D
( x cos x + sin x)
$ NOTES
= ex
I ( x cos x sin x) dx e x
=
1
e 2 x 1 e (2 x 1)
e log 3
x "# ' cosh t e t
e t
and u elog u
"#
2
D 4 2 ! $! 2 $
=
1 1
e2 x 1 2
1
e ( 2 x 1)
"# 1 e x log 3
! 2
2 D 4 D 4 $ D 4
2
1 1 1
2 x 1 "# 1 ( 2 x 1)
x. e x log 3
2 ! 2D $ (log 3)
= x. e e 2
2D 4
1 1 1
2 x 1 "# 1 ( 2 x 1)
e x log 3
2 ! 2D $ (log 3)
= x. e x. e 2
2D 4
= e
x
4! I 2 x 1
dx e dx"
# I 3
$ (log 3) 4
(2 x 1)
x
x e 2 x1
e "# 3 ( 2 x 1) x
=
4! 2 2 $ (log 3) 4 2
x e e 2 x 1 "# 3 ( 2 x 1) x
=
4! 2 $ (log 3) 4 2
x 3x
= sinh (2x 1) +
4 (log 3) 2 4
x 3x
Hence the C.S. is y = c1e2x + c2e2x +sinh (2x 1) + .
4 (log 3) 2 4
Example 20. Solve (D2 + 1)y = x2 sin 2x.
Sol. A.E. is D2 + 1 = 0 D=i
C.F. = c1 cos x + c2 sin x
1 1
P.I. = 2 x2 sin 2x = I.P. of 2 x2e2ix
D 1 D 1
= I.P. of
3 !
1 3 #$ 2
1
4 iD D 4 iD D 2 "# 2 2
1 3
#
2ix ...... 2
= I.P. of e
3
! 3
$
x
1 1 4iD 1 16 D ......"# x
! 3 3 9
2ix 2 2
= I.P. of e
3 $
1 x 4i (2x) 13 (2)"#
2
! 3 $
= I.P. of e 2ix
3 9
1
= [24x cos 2x + (9x2 26) sin 2x]
27
1
Hence the C.S. is y = c1 cos x + c2 sin x [24x cos 2x + (9x2 26) sin 2x].
27
Example 21. Solve (D4 + 2D2 + 1)y = x2 cos x.
Sol. A.E. is (D2 + 1)2 = 0 D = i, i
C.F. = (c1x + c2) cos x + (c3 x + c4) sin x
1 1
P.I. = 2 2
x2 cos x = R.P. of x2 (cos x + i sin x)
(D 1) (D 1) 2
2
1 1
= R.P. of x2eix = R.P. of eix x2
(D 2 1) 2 [(D i) 2 1]2
1 1
= R.P. of eix x2 = R.P. of eix 2
(D 2iD) 2
2
2iD 1 D #" x 2
! 2i $
1
1 i2D
ix 2
1 e
4 D
2
= R.P. of eix x = R.P. of . x2
iD
4D 1 –
2 2
2
2
1 iD iD "#
2
1 2 3
1
= R.P. of e .
4 D !
ix
2 2 2
...... x2
#$
1 1 3 "
x i(2 x) (2)# 2
D ! $
= R.P. of e . ix
2
4 4
= R.P. of
1 ix x
e
4
i
x3 3x 2
"#
4 !
12 3 4 $ NOTES
1
= R.P. of (cos x + i sin x) [(x4 9x2) + (4x3)i]
48
1
= [(x4 9x2) cos x 4x3 sin x]
48
Hence the C.S. is
1
y = (c1x + c2) cos x + (c3x + c4) sin x [(x4 9x2) cos x 4x3 sin x].
48
d2 y
Example 22. Solve + y = cosec x.
dx 2
Sol. Given equation in symbolic form is (D2 + 1)y = cosec x
A.E. is D2 + 1 = 0 D = i
C.F. = c1 cos x + c2 sin x
1 1
P.I. = 2 cosec x = cosec x
D 1 (D i)(D i)
1 1 1
=
2i D i D i
cosec x (Partial Fractions)
1 1
=
2i D i
cosec x
1
Di
cosec x
Now
1
Di
cosec x = e ix
I
cosec x e dx
' 1 X = e Xe dx"#
! Da
ix
$
ax
I – ax
= eix I
= eix (log sin x ix)
cosec x (cos x i sin x) dx = eix I (cot x i) dx
1
Changing i to i, we have cosec x = eix (log sin x + ix)
Di
1 ix
P.I. = [e (log sin x ix) eix (log sin x + ix)]
2i
e ix
e ix eix e ix
= log sin x 2i
x
2
= log sin x . sin x x cos x
Hence the C.S. is y = c1 cos x + c2 sin x + sin x log sin x x cos x.
d2 y
Example 23. Solve + a2y = tan ax.
dx 2
Sol. Given equation in symbolic form is (D2 + a2)y = tan ax
A.E. is D2 + a 2 = 0 D = ia
C.F. = c1 cos ax + c2 sin ax
1 1
P.I. = 2 2 tan ax = tan ax
D a (D ia)(D ia)
Self-Instructional Material 123
Ordinary Differential
Equations =
1 1
1 "#
tan ax (Partial Fractions)
!
2ia D ia D ia $
1 1 1 "
= tan ax tan ax#
NOTES 2ia ! D ia D ia $
Now
1
D ia
tan ax = eiax I tan ax . eiax dx
cos ax
= eiax
cos ax i sin ax "#
! a
log (sec ax tan ax) i
a a $
1 iax
= e [(cos ax i sin ax) + i log (sec ax + tan ax)]
a
1 iax iax
= e [e + i log (sec ax + tan ax)]
a
1
= [1 + ieiax log (sec ax + tan ax)]
a
1 1
Changing i to i, we have tan ax = [1 ieiax log (sec ax + tan ax)]
D ia a
1 1
P.I. =
2ia a
1 ieiax log sec ax tan ax
1
{1 ie iax log (sec ax tan ax )
a
1 eiax e iax
= log (sec ax + tan ax)
a2 2
1
= log (sec ax + tan ax) . cos ax
a2
1
Hence the C.S. is y = c1 cos ax + c2 sin ax cos ax log (sec ax + tan ax).
a2
EXERCISE B
Solve the following differential equations:
d3 y d2 y
1. 3 + y = 3 + 5ex.. 2. 4y = (1 + ex)2.
dx dx2
d2 y dy d2 y dy
3. 2
4 + 5y = 2 cosh x. 4. 2
2 + 5y = sin 3x.
dx dx dx dx
d y 3 2
d y dy d2 y dy
5. (i) + y = sin 2x. (ii) cos 2 x
dx 3
dx 2 dx dx2 dx
1 2 x 1 2x
2. y = c1e2x + c2e2x e xe
4 3 4
1 x 1 x
3. y = e2x(c1 cos x + c2 sin x) e e
10 2
1
4. y = ex(c1 cos 2x + c2 sin 2x) + (3 cos 3x 2 sin 3x)
26
1 1
(cos x sin x)
2 4
x
c 3 3 1
(ii) y c1e x
e 2
2 cos
2
x c3 sin
2
x 1
65
(cos 2 x 8 sin 2 x)
1 2x 1 1
(iii) y = (c1 + c2x)ex + e (3 cos 2x 4 sin 2x)
9 2 50
2
x x 1
(iv) y = c1 + (c2 + c3x)ex e (3 cos 2x 4 sin 2x)
2 50
y y 2 y
(v) z c1 c2 e c3e y y 2 sin y ye
x
c 3 3 3 1 3
(vi) y e 2
1 cos x c2 sin x 2 cos x sin 2 x cos 2 x
2 2 2 13 26
1 1
7. y = c1ex + c2e3x + (10 cos 5x 11 sin 5x) + (sin x + 2 cos x)
884 20
3 3x 1
8. y = c1ex + c2e2x + e + (3 cos 2x sin 2x)
10 20
1 x
9. y = c1 cos 2x + c2 sin 2x + e x cos 2x
5 4
1 2x
10. y = c1 + ex (c2 cos 3 x + c3 sin 3 x) + (e + sin 2x)
8
1 1
11. y = c1e2x + c2e2x x 2 2 x
4 2
1 x2 25
12. y = c1 + c2e3x + c3e2x
18
x3
2
6
x
x3
13. y = c1 + c2ex + + 4x
3
1 2x 1
14. y = c1 cos x + c2 sin x + e cosh 2x + x3 6x
5 5
8 x x x
15. y = c1ex + c2e2x e 2 sin cos
5 2 2
1 1
16. y = c1ex + c2e2x + e3x (2x 3) + (3 cos 2x sin 2x)
4 20
1 x
17. y = c1ex + c2ex + c3 cos x + c4 sin x e cos x
5
1 x x 1 x
18. (i) y = c1 + c2e2x e sin x (ii) y e (c1 cos x c2 sin x ) ( x 1 xe sin x )
2 2
1 2x
19. y = e2x (c1 cos 2x + c2 sin 2x) + e (3x sin 2x + cos 4x)
2
e3 x
x2
12 50 ex
20. (i) y = c1 cos 2 x + c2 sin 2x+
11 11
x
121
17
(4 sin 2x cos 2x)
1 x 1 3x 5
(ii) y = c1ex + c2e3x e (sin x + 2 cos x) + e x
5 24 12
1 1 x2 x NOTES
23. y = (c1 + c2x)ex + (c3 + c4x)ex + cos x + e +x
2 8 8
1
24. y = c1 cos 2x + c2 sin 2x + (3x sin x 2 cos x)
9
1 2
25. y = c1 ex + c2 ex x cos x (x 1) sin x
2
1
26. y = c1 e3x + c2 e3x (13x cos 2x 4 sin 2x)
169
1 1
27. y = c1ex + c2ex (x sin x + cos x) + xex (2x2 3x + 9)
2 12
1 3 1
28. y = c1ex + c2ex x sin 3 x cos 3 x cos x
10 5 2
1 log cos ax
29. y = c1 cos ax + c2 sin ax + x sin ax cos ax
a a
Note 1. As the solution is obtained by varying the arbitrary constants c1, c2 of the C.F.,
the method is known as variation of parameters.
Note 2. Method of variation of parameters is to be used if instructed to do so.
SOLVED EXAMPLES
Example 24. Apply the method of variation of parameters to solve
d2 y
+ 4y = 4 sec2 2x.
dx 2
Sol. Given equation in symbolic form is (D2 + 4)y = 4 sec2 2x
Its A.E. is D2 + 4 = 0 so that D = 2i
C.F. is y = c1 cos 2x + c2 sin 2x
Here, y1 = cos 2x, y2 = sin 2x and X = 4 sec2 2x
y1 y2 cos 2x sin 2x
W= =2
y1 y2 2 sin 2x 2 cos 2x
P.I. = y1 I y2 X
W
dx y2 I y1X
W
dx
= cos 2x
I sin 2 x . 4 sec 2 2 x
2
dx sin 2 x I cos 2 x . 4 sec 2 2 x
2
dx
sec 2 x 1
= 2 cos 2x . + 2 sin 2x . log (sec 2x + tan 2x)
2 2
= 1 + sin 2x log (sec 2x + tan 2x)
Hence the C.S. is y = c1 cos 2x + c2 sin 2x 1 + sin 2x log (sec 2x + tan 2x).
Example 25. Solve by the method of variation of parameters:
d2 y dy e 3x
2
6 9y 2 .
dx dx x
Sol. Given equation in symbolic form is
e3 x
(D2 6D + 9)y =
x2
P.I. = y1 I y2 X
W
dx y2 I y1X
W
dx
e3x e3x
= e3x I xe 3 x .
e 6x
x 2 dx xe 3 x
I e3 x .
e 6x
x 2 dx
= e3x I 1
x
dx xe 3 x I 1
x2
dx
1 = (1 + log x) e
= e3x log x + xe3x
x 3x
y1 y2 ex ex
W= x =2
y1 y2 e ex
P.I. = y1 I y2 X
W
dx y2 I y1X
W
dx
e x [ e x sin ( e x ) cos ( e x )]
= ex 2
dx
e x [ e x sin ( e x ) cos ( e x )]
ex 2
dx
=
1 x
e I ex [ex sin (ex) + cos (ex)] dx
I
2
1 x
e ex [ex sin (ex) + cos (ex)] dx ...(1)
2
I
Equations
= ex cos (ex)
| Form I ex [ f (x) + f (x)] dx = exf(x)
EXERCISE C
Solve by the method of variation of parameters:
d2 y
1. + y = cosec x.
dx2
d2 y d2 y
2. (i) 2 + 16 y = 32 sec 2x (ii) + a2y = sec ax
dx dx2
(iii) y + y = sec2 x (iv) y + 3y + 2y = sin (ex)
d2 y d2 y
3. + y = tan x. 4. + 4y = tan 2x.
dx2 dx2
d2 y
5. (i) + y = x sin x. (ii) (D2 + 1)y = cosec x cot x
dx2
d2 y dy
6. (i) y 2y + 2y = ex tan x. (ii) 2 = ex sin x.
dx2 dx
d2 y dy 1 d2 y dy 12e4 x
7. 6 9 y 3 e3x. 8. 8 16 y .
dx2 dx x dx2 dx x4
d2 y dy
9. 4 + 4y = e2x sec2 x. 10. y 2y + y = ex log x.
dx2 dx
d2 y 2 d2 y 1
11. y . 12. y .
dx 2
1 e x
dx2 1 sin x
Answers
1. y = c1 cos x + c2 sin x x cos x + sin x log sin x
2. (i) y = c1 cos 4x + c2 sin 4x + 8 cos 2x 4 sin 4x log (sec 2x + tan 2x)
1 1
(ii) y = c1 cos ax + c2 sin ax + 2 cos ax log (cos ax) + x sin ax
a a
(iii) y = c1 cos x + c2 sin x 1 + sin x log (sec x + tan x)
(iv) y = c1ex + c2e2x + e2x sin (ex)
x x2 NOTES
5. (i) y = c1 cos x + c2 sin x + sin x cos x
4 4
(ii) y = c1 cos x + c2 sin x + cos x log sin x x sin x
6. (i) y = ex (c1 cos x + c2 sin x) ex cos x log (sec x + tan x)
1 x
(ii) y1 = c1 + c2 e2x e sin x
2
1 3x 2 e
8. y = c1 c2 x
7. y = c1 c2 x 4x
e 2
2x x
1 2 x
9. y = (c1 + c2x log cos x) e2x 10. y = (c1 + c2x) ex + x e (2 log x 3)
4
11. y = c1ex + c2ex 1 xex + (ex ex) log (1 + ex)
12. y = c1 cos x + c2 sin x + sin x log (1 + sin x) x cos x 1.
d3y 2
2 d y 1
Example 27. Solve x3
dx 3
2x
dx 2
+ 2y = 10 x
x
.
NOTES
Sol. Given equation is a Cauchys homogeneous linear equation.
Put x = ez i.e., z = log x
dy d2 y
so that x = Dy, x2 = D(D 1)y
dx dx 2
d3 y d
x3 = D(D 1)(D 2)y, where D =
dx 3 dz
Substituting these values in the given equation, it reduces to
[D(D 1)(D 2) + 2D(D 1) + 2]y = 10(ez + ez)
or (D3 D2 + 2)y = 10(ez + ez)
which is a linear equation with constant co-efficients.
Its A.E. is D3 D2 + 2 = 0 or (D + 1)(D2 2D + 2) = 0
2 48
D = 1, = 1, 1 i
2
c1
C.F. = c1ez + ez (c2 cos z + c3 sin z) = + x[c2 cos (log x) + c3 sin (log x)]
x
1 1 1
P.I. = 10 3
D – D2 2
(ez + ez) = 10
D3 D2 2
ez 3
D D2 2
e z
1 1
= 10 3 2
ez z . 2
ez
1 1 2 3D 2D
1 z 1
= 10 e z . 2
ez
2 3( 1) 2( 1)
2
= 5ez + 2zez = 5x + log x
x
c1 2
Hence the C.S. is y = + x[c2 cos (log x) + c3 sin (log x)] + 5x + log x.
x x
2
d y dy
Example 28. Solve x2 x 3y = x2 log x.
dx 2 dx
Sol. Given equation is a Cauchys homogeneous linear equation.
Put x = ez i.e., z = log x so that
dy d2 y d
x = Dy, x2 = D(D 1)y, where D = .
dx dx 2 dz
Substituting these values in the given equation, it reduces to
[D(D 1) D 3]y = ze2z or (D2 2D 3)y = ze2z
which is a linear equation with constant co-efficients.
Its A.E. is D2 2D 3 = 0 or (D 3)(D + 1) = 0
D = 3, 1
c2
C.F. = c1e3z + c2ez = c1x3 +
x
! #$
= e2z z e 1 z
2D D 2 3 3 3
3 1 3
3
1 2z 2D D2
= e 1 ...... z
3 3 3
1 2z 2 x2 2
= e z 3 3 log x 3
3
c2 x 2 2
Hence the C.S. is y = c1x3 +
x
3
log x
3
.
d2 y dy
Example 29. Solve x2 2
x + y = log x sin (log x).
dx dx
Sol. Given equation is a Cauchys homogeneous linear equation.
dy d2 y
Put x = ez i.e., z = log x so that x = Dy, x2 = D(D 1)y
dx dx 2
d
where D= .
dz
Substituting these values in the given equation, it reduces to
[D(D 1) + D + 1]y = z sin z
or (D2 + 1)y = z sin z
Its A.E. is D2 + 1 = 0 so that D=±i
C.F. = c1 cos z + c2 sin z = c1 cos (log x) + c2 sin (log x)
1 1
P.I. = 2 z sin z = Imaginary part of 2
zeiz
D 1 D 1
1 1
= I.P. of eiz z = I.P. of eiz 2 z
(D i) 2 1 D 2iD
1 1
= I.P. of eiz
D
z = I.P of eiz
iD z
2iD 1 +
2i 2iD 1
2
1 iz 1 iD
1
1 iz 1
iD
= I.P. of
2i
e
D
1
2
z = I.P. of
2i
e
D
1
2
...... z
= I.P. of
1 iz 1
2i
e
D
z
i
2
= I.P. of
1 iz
2i
e
z
i
2
dz I
i
= I.P of e
z i z = I.P. of e i z z
2
2 2 4 4
iz iz 2
2
i z z cos z z sin z
= I.P. of (cos z + i sin z) z
2
4 4 4
2
4
1 1
= (log x)2 cos (log x) + log x sin (log x)
4 4
Self-Instructional Material 133
Ordinary Differential Hence the C.S. is
Equations y = c1 cos (log x) + c2 sin (log x) 1
(log x)2 cos (log x) + 1
log x sin (log x).
4 4
d2 y
dy
Example 30. Solve: x2 4x + 2y = ex.
NOTES dx 2 dx
Sol. Given equation is a Cauchys homogeneous linear equation.
Put x = ez i.e., z = log x so that
dy d2 y d
x = Dy, x2 = D(D 1)y, where D =
dx dx 2 dz
Substituting these values in the given equation, it reduces to
z z
[D(D 1) + 4D + 2]y = e e or (D2 + 3D + 2)y = e e
Its A.E. is D2 + 3D + 2 = 0 or (D + 1)(D + 2) = 0
D = 1, 2
C.F. = c1ez + c2e2z = c1x1 + c2x2
1 z 1 z
P.I. = 2
ee ee
D 3D 2 (D 1)(D 2)
1 1 e ez 1 z
ee
1
ee
z
=
D 1 D 2
D ( 1) D ( 2)
= ez I z
e e . e z dz e 2 z I z
e e . e 2 z dz ' D 1 a X e X . e
ax ax
dx
= ez I z
e e . e z dz e 2 z I z
e e . ez . ez dz | Put ez = t
= ez I et dt e2z
| Integrating by parts
z z
= ez . e e e2z(ez 1) e e
z z
= (ez ez + e2z) e e = e2z . e e
= x2ex
Hence the C.S. is y = c1x1 + c2x2 + x2ex or y = (c1x + c2 + ex )x2.
b2 dy b d2 y b b2 d y dy
2
= .
(a bx) 2 dz a bx dz 2 a bx (a bx) 2
dz dz
2
NOTES
d2 y
or (a + bx)2 = b2 (D2y Dy) = b2 D(D 1)y
dx 2
d3 y
Similarly, (a + bx)3 = b3 D(D 1)(D 2)y.
dx 3
Substituting these values in equation (i), we get a linear differential equation
with constant co-efficients, which can be solved by the methods already discussed.
SOLVED EXAMPLES
d2 y dy
Example 31. Solve (3x + 2)2 + 3(3x + 2) 36y = 3x2 + 4x + 1.
dx 2 dx
Sol. Given equation is a Legendres linear equation.
dy
Put 3x + 2 = ez i.e., z = log (3x + 2) so that (3x + 2) = 3Dy,
dx
d2 y d
(3x + 2)2 = 32 D(D 1)y, where D = .
dx 2 dz
Substituting these values in the given equation, it reduces to
e 2
z 2
e 2 + 1
z
[32 D(D 1) + 3.3D 36]y = 3 3 4 3
1 2z 1 1
or 9(D2 4)y =e or (D2 4)y = (e2z 1)
3 3 27
which is a linear equation with constant co-efficients.
Its A.E. is D2 4 = 0 D = ± 2
C.F. = c1e2z + c2e2z = c1 (3x + 2)2 + c2 (3x + 2)2
P.I. =
1
.
1
( e 2 z 1)
1 1
e2 z 2
1
e0 z
"#
27 D 2 4 27 D 2 – 4 !
D 4 $
=
!
1
27
z.
1 2z
2D
e
1
04
e0 z
"# 1 z
$ 27 ! 2 I e 2 z dz
1
4
"#
$
1 z 2z 1"# 1 1
( ze 2 z 1)
27 ! 4
= e
4
$ 108 108
[(3x + 2)2 log (3x + 2) + 1]
Answers
1. (i) y = c1 x1 + c2 x2 (ii) y = x4 [c1 cos (3 log x) + c2 sin (3 log x)] +
2
c2 1 2 1 x2 x 1
2. y = c1x2 + x log x 3. y = c1x5 + c2x4
x 3 x 14 9 20
2 1
4. y = c1 + c2x3 + c3x4 + x 5. (i) y = (c1 + c2 log x)x + c2x1 +
log x
3 4x
1 1 x
(ii) y = c1x2 + c2x3 x2 log x (iii) y = (c1 c2 log x) log
x x 1 x
1
14. y = c1 x2 3 + c2 x2 3 + log x {5 sin (log x ) 6 cos (log x )}
61x
2 1
{27 sin (log x ) 191 cos (log x )} (1 + log x)
61 6x
15. (i) y = c1 + c2 log x + 2 (log x)3
16. (i) y = c1 cos [log (1 + x)] + c2 sin [log (1 + x)] + 2 log (1 + x) sin [log (1 + x)]
(ii) y = c1 cos [log (1 + x)] + c2 sin [log (1 + x)] log (1 + x) cos [log (1 + x)]
1
(iii) y = c1 cos [log (1 + x )] + c2 sin [log (1 + x)] sin [2 log (1 + x)]
3
17. y = c1 + c2 log (x + 1) + [log (x + 1)]2 + x2 + 8x
18. y = (1 + 2x)2[c1 + c2 log (1 + 2x) + {log (1 + 2x)}2]
3 3
19. y = c1(2x + 3)1 + c2(2x + 3)3 (2x + 3) + .
16 4
I.F. = e
I 2
u
. u1 P dx
e
I 2
u
du I Pdx
2 log u P dx
= e u2 e P dx
We have p.u2 e I P dx
= I R 2
.u e I P dx dx + c
I
1
u
p = u2 e I P dx
4Rue 9 dx + c u e
I P dx 2 I P dx
I
1
or v1 =
dv
dx
= u2 e I P dx
4Rue 9 dx + c u I P dx
1
2
e I P dx
v= I u2 e I P dx
y = uv = u
I u 2 e I P dx
y = xm is a solution
If y = xm
dy d2 y
Then = mxm1 and = m(m 1)xm2
dx dx 2
If y = xm is a solution of (1), then m(m 1)xm2 + Pmxm1 + Qxm = 0
or m(m 1) + Pmx + Qx2 = 0.
Deduction. (i) y = x is the solution of (1), if P + Qx = 0.
(ii) y = x2 is the solution of (1), if 2 + 2Px + Qx2 = 0.
Note. One integral belonging to the complementary function can be found by
inspection. For this following rules are observed :
(i) y = x is a part of C.R., if P + Qx = 0
(ii) y = ex is a part of C.F., if 1 + P + Q = 0 (i.e., sum of the co-efficients are zero)
(iii) y = ex is a part of C.F., if 1 P + Q = 0
P Q
(iv) y = eax is a part of C.F., if 1 + =0
a a2
(v) y = x2 is part of C.F., if 2 + 2Px + Qx2 = 0.
SOLVED EXAMPLES
d2 y dy
Example 32. Solve: x2 2
2x(1 + x) + 2(1 + x)y = x3.
dx dx
Sol. The given equation can be written as
d2 y 1 1 dy + 2 1 1 y = x
dx 2 2 x dx x x 2
1 1 1 = 0
P + Qx = 2 1 + 2x
x
where 2
x x
y = x is a part of C.F.
Putting y = vx so that
dy dv
= x+v
dx dx
d2 y d 2v dv
and = +2 , we get
dx 2
dx 2 dx
d 2v dv dp dv
2 =1 or 2p = 1 where p=
dx 2
dx dx dx
I
I
1 2x
pe2x = 1 . e2x dx + c1 = e + c1
NOTES 2
dv 1
p= = + c1e2x
dx 2
1 c1 2 x
Integrating, we get v = x + e + c2
2 2
The complete solution is
1 2 c1 2 x
y = vx = x + xe + c2 x.
2 2
d2 y dy
Example 33. Solve: x2 2
2 (x + 2x) + (x + 2)y = x3ex.
dx dx
Sol. The given equation can be written as
d2 y 2
dy 1 2 y = xe
dx 2
1
x
dx
+ 2
x x x
Here
2
P = 1 , Q=
1
2
and R = xex
x x x2
Since P + Qx = 0
y = x is a part of the C.F.
Putting y = vx, so that
dy dv d2 y d2v dv
= .x+v and 2 = x+2
dx dx dx dx 2 dx
d2v dv dp dv
We get = ex or p = ex, where p =
dx 2 dx dx dx
which is a linear equation,
dx
I.F. = e = ex
I
pex = I ex . ex dx + c1 = x + c1
dv
p= = xex + c1ex
dx
Integrating, we get v = xex ex + c1ex + c2
The complete solution is
y = vx = x2ex xex + c1x ex + c2 x.
d2y
Example 34. Solve: sin2 x . = 2y given y = cot x is a solution.
dx 2
Sol. Putting y = v cot x, so that
dy dv
= cot x v cosec2 x
dx dx
d2 y d2v dv
and 2
= 2
cot x 2 cosec2 x + 2v cosec2 x cot x
dx dx dx
dy d2 y
Example 35. Solve : x
dx
y = (x 1)
dx 2
x1 .
Sol. The given equation may be written as
d2 y x dy y
. + =x1
dx 2 x 1 dx x1
Here P + Qx = 0
y = x is a part of C.F.
dy dv d2 y d2v dv
Putting y = vx, so that = x+v and 2 = x+2
dx dx dx dx 2 dx
I.F. = e
I
x
x1
dx I 2
x
dx
e
I
1
1
dx
x 1 I 2
x
dx
x log ( x 1) 2 log x x2 x
= e e
x1
p
x2 e x
x1
= I x 1 x2
x
.
x1
ex dx + c1 = I xex dx + c1 = xex ex + c1
dv x 1 ( x 1) c1 ( x 1) e x 1 1 1 e
p=
dx
=
x
x 2
x 2 = 1 + 2 c1
x
2
x x x
d2 y x cos x dy cos x
. y = sin x(x sin x + cos x)
dx 2 x sin x cos x dx x sin x cos x
Here P + Qx = 0 y = x is a part of C.F.
Putting y = vx the equation reduces to
d 2v 2
x cos x dv sin x ( x sin x cos x)
dx 2 +
x x sin x cos x dx
=
x
dp 2
x cos x sin x
or
dx
+
x x sin x cos x
p=
x
(x sin x + cos x)
which is a linear equation.
I.F. = e
I
2
x cos x
dx
x x sin x cos x
=e
2 log x log ( x sin x cos x)
x2
( x sin x cos x)
p.
x2
( x sin x cos x)
= I x sin x dx + c1 = x cos x + sin x + c1
dv 1 c
p= = 2 ( x cos x + sin x)(x sin x + cos x) + 12 (x sin x + cos x)
dx x x
dy 1 1 1 1
dx
= sin x cos x cos 2x + 2 sin x cos x + c1
x x x
sin x 2 cos x
x
Integrating,
v=
1
2
cos2 x I 1
x
cos 2x dx + I 1
2x 2 sin 2x dx + c1 I 1
x
1
sin x 2 cos x dx
x
1 1 c1
= cos2 x sin 2x cos x + c2
2 2x x
The complete solution is
x 1
y = vx = cos2 x sin 2x c1 cos x + c2 x.
2 2
d2 y dy
Example 37. Solve: (1 x2) +x y = x(1 x2)3/2.
dx 2 dx
Sol. The given equation can be written as
d2 y x dv 1
2 + 2
y = x(1 x2)1/2
dx 1 x dx 1 x2
I.F. = e
I
x 2
dx
1 x2 x
= e
1
2 log (1 x 2 ) 2 log x
x2
1 x2
p.
x
1 x2
= I x2 dx + c1 =
x3
3
+ c1
dv 1 2 c1
p= = x 1 x 2 1 x2
dx 3 x
1 1
x 1 x 2 c1 (1 x 2 ) 1/ 2 . 2
=
3 x
Integrating,
1
v = (1 x2)3/2 + c1 (1 x2)1/2
9
1
x
c1
I dx
1 x2
+ c2
1 c1
= (1 x2)3/2 (1 x2)1/2 c1 sin1 x + c2
9 x
The complete solution is
1
y = vx = x(1 x2)1/2 c1 {x sin1 x + 1 x 2 } + c2 x.
9
d2 y dy
Example 38. Solve cot x (1 cot x) y = ex sin x.
dx 2 dx
Sol. From the above equation, we have P + Q + 1 = 0
y = ex is a part of C.F.
dy dv x
Putting y = vex so that = e + v. ex
dx dx
d2 y d 2v x dv x
and 2
= 2
e +2 e + vex
dx dx dx
d2v dv
We have, + (2 cot x) = sin x
dx 2 dx
dp dv
or + (2 cot x) p = sin x, where p =
dx dx
which is linear equation.
(2 cot x ) dx e2 x
I.F. = e
I
e 2 x log sin x
sin x
p
e2x
sin x
= I e2x
sin x
1
sin x dx + c1 = e2x + c1
2
I.F. = e
I 2x 3
x2
dx
e
I
2
1
dx
x 2
=e
2 x log ( x 2)
e2 x
x2
p.
e2 x
x2
= I x1
( x 2) 2
ex dx + c1
= I %&' 1
1
x 2 ( x 2) 2
e x
dx c1
()
*
ex
x2
+ c1
dv
p= = ex + c1 e2x (x + 2).
dx
1 1
Integrating, v = ex c e2x(x
2 1
+ 2) c e2x
4 1
+ c2
1
= ex c
4 1
(2x + 5)e2x + c2
The complete solution is
1
y = ve2x = ex c
4 1
(2x + 5) + c2e2x.
d2 y dy
Example 40. Solve : x 2
(2x + 1) + (x + 1)y = (x2 + x 1)e2x.
dx dx
Sol. The given equation can be written as
d2 y 1 dy 1
1 2x
dx 2
2
x dx
+ 1
x
y = x 1
xe
144 Self-Instructional Material
Here 1 + P + Q = 0 y = ex is a part of C.F. Linear Differential
Equations of Second and
Putting y = vex the equation reduces to
Higher Order
d 2v 1 dv
1 xs
dx 2
x dx
= x 1
x
e
NOTES
dp 1 1
e, dv
or
dx
p = x 1
x x
x where p =
dx
which is a linear equation.
I.F. = e
I 1
x
dx 1
e log x
x
p.
1
x
= I x 1
1 x 1
x
e . dx + k
x
= I ex
1 x
x
1
e 2 e x dx + k = ex +
x
ex
x
+k
dv
p= = xex + ex + kx,
dx
k 2 k
Integrating, v = xex + x + c2 or v = xex + c1x2 + c2, where c1 =
2 2
The complete solution is
y = vex = xe2x + c1x2ex + c2ex.
EXERCISE E
Solve the following differential equations:
d2 y dy d2 y dy
1. x 2
(3 x ) + 3y = 0 2. x 2
(2x 1) + (x 1)y = 0
dx dx dx dx
d2 y dy 1
3. x2 x y = 0, given that y = x + is a solution.
dx 2 dx x
4. (2 + x)y (9 + 4x)y + (7 + 3x)y = 0
5. x(x cos x 2 sin x)y + (x2 + 2)sin x . y 2(x sin x + cos x)y = 0
d2 y dy d2 y dy
6. x ( x 2) + 2y = x3. 7. x 2( x 1) + (x + 2)y = (x 2)ex.
dx 2 dx dx 2 dx
d2 y dy d2 y dy
8. 2
(1 x ) + xy = x 9. ( x 1) 2 2( x 3) + (x + 5) = ex
dx dx dx dx
10. (x x2)y (1 2x)y + (1 3x + x2)y = (1 x)3
Answers
1. y = c1(x3 + 3x2 + 6x + 6) + c2ex 2. y = (c1 log x + c2)ex
c1 1
3. y= c2 x 4. y = c1(2x + 3)e3x + c2ex
x x
5. y = c1 sin x + c2x2 6. y = c1(x2 + 2x + 2) + c2ex x3
1
1 2 x 1 3 x x x2
7. y= x e + xex + c1x e + c2ex 8. y = c1e x e 2 dx + c2ex + 1
2 3
1 x 1 x 1
9.
5
y = xe c1e ( x 1) + c2ex 10. y = c1x 2e x + c2ex x
4 5 2
d 2v du dv + d u P du Qu v = R
2
u
dx 2
+ Pu 2
dx dx dx dx 2
d v P 2 du dv 1 d u P du Q
2
2 R
+
or
dx 2 u dx dx u dx u dx u
v=
2 ...(2)
2 du
Let us choose u such that P+ =0
u dx
du P du 1
or = u or = Pdx
dx 2 u 2
1
2
Pdx
u= e
From equation (2), we have
1 u dP P du P du Q"# v = R e I
d2v
1
P dx
! u 2 dx 2 dx u dx $
2
2
+
dx
d v2 1 dP P P u P P u Q"# v = R e I 1
P dx
! 2 dx 2u 2 u 2 $
+
or 2
2
dx
d v 1 dP 1 P "# v = R. e I
2
2
1
P dx
! 2 dx 4 $
+ Q
or 2
2
dx
d 2v
or + Xv = Y ...(3)
dx2
where X=Q
1 dP 1 2
2 dx 4
P and Y = Re 2
1
P dx I
The equation (3), may easily be integrated. Equation (3) is said to be the normal
form of the equation (1).
Note. Remember equation (3), and the values of u, X and Y.
where u = e
1
2 I P dx
e
1
2 I x 1/3 dx
e
1 2/ 3
3
x
1 dP 1 2
X=Q P
2 dx 4
1 1 6 1 1 1 6
=
4 x2/ 3
6x 4 /3
2
x 2
x 4 / 3 x2/3 = 2
3 4 x
and Y = Re 2 =0
1
I P dx
d2v
Putting y = uv, the normal form is + Xv + Y
NOTES dx 2
where u= e
1
2 I Pdx
=e
I
1+
1
dx
x
e x log x = xex
1 dP 1 2 2 2 1 2 1 1 2
X=Q P = 1 + 2 . 2 4 1
2 dx 4 x x 2 x 4 x =0
1 Pdx
Y = Re2
I
and =0
2
d v
The normal form is =0
dx 2
Integrating, v = c1x + c2
The solution of the given equation is y = uv = xex (c1x + c2).
d2 y dy
Example 43. Solve 2
2 tan x + 5y = ex sec x.
dx dx
Sol. Here P = 2 tan x, Q = 5 and R = ex sec x
Putting y = uv is the given equation, the equation reduces to
d 2v 1 Pdx
+ Xv = Y, where u = e
I
2
2
dx
log sec x
tan x dx
=e =e I
= sec x
1 dP 1 2
X=Q P
2 dx 4
1 1
=5+ 2 sec2 x . 4 tan2 x = 6
2 4
1 Pdx
Y = Re2 = ex
I
d2v
The reduced equation is + 6v = ex
dx 2
where C.F. = c1 cos 6 x + c2 sin 6x
1 ex
and P.I. = ex =
D2 6 7
ex
v = c1 cos 6 x + c2 sin 6x +
7
The solution of the given equation is
1 x .
y = uv = sec x c1 cos 6 x c2 sin 6 x
7
e
d2 y dy x2
Example 44. Solve 2
4x + (4x2 3)y = e .
dx dx
x2
Sol. Here P = 4x, Q = 4x 3, R = e
2
2
dx
1 dP 1 1 1
X=Q P2 = 4x2 3 ( 4) (16x2) = 1
2 dx 4 2 4
1 Pdx
Y = Re
I
and 2
=1
148 Self-Instructional Material
d 2v Linear Differential
The normal form is v=1 Equations of Second and
dx 2 Higher Order
where C.F. = c1 ex + c2 ex
1 NOTES
and P.I. = 2 . 1 = (1 D2)1. 1 = 1
D 1
v = c1ex + c2ex 1
The solution of the given equation is
2
y = uv = e x (c1ex + c2ex 1).
d2 y dy x2
Example 45. Solve 2
4x + (4x2 1) y = 3 e sin 2x.
dx dx
x2
Sol. Here P = 4x, Q = 4x2 1 and R = 3 e sin 2x
d2v
Putting y = uv, the equation reduces to + Xv = Y
dx 2
1 2
Pdx x
where u= e =e
I
2
1 dP 1 2 1 1
X=Q P = 4x2 1 ( 4) 16x2 = 1
2 dx 4 2 4
1 Pdx
Y = Re 2
I
= 3 sin 2x.
d2v
The reduced equation is + v = 3 sin 2x
dx 2
whose C.F. = c1 cos x + c2 sin x
1 3
and P.I. = 2 ( 3 sin 2x) = sin 2x = sin 2x
D 1 22 1
v = c1 cos x + c2 sin x + sin 2x
The solution of the given equation is
2
x
y = uv = e (c1 cos x + c2 sin x + sin 2x).
d2 y 2 dy 2
Example 46. Solve
dx 2
x dx
+ 1 2 y = xex.
x
2 2
Sol. Here P = , Q = 1 + 2 and R = xex
x x
d2v
Putting y = uv, the normal form is + Xv = Y
dx 2
1 Pdx 1/ x dx log x
where u= e =e =e =x
I I
2
1 dP 1 2 1 Pdx
P = 1 and Y = R e 2
I
X=Q
2 dx 4
= xex e dx/x = xex e log x = ex
I
d 2v
+ Xv = Y
dx 2
where u = e
1
2 I P dx
e
I 1
x log x
dx
e (log log x) log x
1 dP 1 2
X=Q P
2 dx 4
2 1 2 1 2 (log x 1) 1 4 2
= 2 2
2
2 2
2
= 2
2 4
x (log x) x log x x 2 x (log x) ( x log x) x
1 Pdx
Y = Re
I
and =12
where u= e
1
e x /2
2 I P dx 2
1 dP 1 2
X=Q P =0
2 dx 4
and Y = Re
1
2 I P dx
= (x3 + 3x) e x
2/ 3
d2v 2/3
The transformed equation is, 2
= (x3 + 3x) e x
dx
Integrating,
dv
dx
= I x3 e x
2
/2
dx + 3 I xex
2
/2
dx + c1
= I x2 (x e x
2
/2
) dx + 3 e x
2
/2
+ c1
= x2 e x
2
2
/2
2 I
2
xex
2
/2
2
dx + 3 e x
2
/2
+ c1
2
= x2 e x /2
2ex /2
+ 3 ex /2
+ c1 = (x2 + 1) e x /2
+ c1
Integrating again = I x2 e x
2
/2
dx + I ex
2
/2
dx + c1 x + c2
=
The solution of the given equation is
I x (x e x
2
/2
) dx + I ex
2
/2
dx + c1x + c2 = x e x
2
/2
+ c1x + c2
2
y = uv = x + (c1x + c2) e x /2
d2 y 1 dy 1
Example 49. Solve + ( 8 + x + x) y = 0.
dx 2
x dx 4x 2
1 1
Sol. Here, P= ,Q ( 8 + x + x) and R = 0
x 4x2
Putting y = uv, the given equation is transformed to
d 2v
+ Xv = Y
dx 2
where u= e
1
2 I P dx
e
1
2 I 1
x
dx
= e
x
1 dP 1 2 2
X=Q P = 2
2 dx 4 x
Y = Re
1
2 I P dx
=0
1 dP 1 2
X=Q P = x2 + 5 1 x2 = 4
2 dx 4
1 Pdx
and Y = Re 2 = x.
I
11 D2 1
x
P.I. = 2
D 4
x=
4
1
4 x=
4
2
The solution is y = uv = e x /2
[c1 cos (2x + c2) + 1
4 x].
EXERCISE F
Solve the following differential equations:
d2 y 2 dy 2 2 y = 0 d2 y dy
1.
dx2
x
+ a 2
dx x 2. (x3 2x2)
dx2
+ 2x2
dx
12(x 2) y = 0
d2 y dy d2 y 2 dy
3. 2 tan x + 5y = 0 4. + + n2y = 0
dx 2 dx dx2 x dx
d2 y dy d dy dy
5. 2
2bx + b2x2y = 0 6. x x y 2x + 2y + x2y = 0
dx dx dx dx dx
d 2 dy d2 y dy
7. cos x + y cos2x = 0 8. 2 y cot x 2 y tan x = sec x
dx dx dx dx
1
7.
y = sec x c1 cos 2x c2 sin 2x 8. y =
2
(sin x x cos x ) + (c1x + c2)cos x
x2 1 2 x2
1 (x 3x 2
1
2 x )
3 x
9. y= ez c1 cos 3x c2 sin 3x e 2 10. y = e 2
c1e c2e x 2
4 9 9
x2
11. y = (c1e2x + c2)x 12. y = (c1 + c2x) sin x
2
2
13. y = ex (c1 cos x + c2 sin x + 5 + cos 2x)
d2 y dy
2 +P + Qy = R ...(1)
dx dx
Let the independent variable be changed from x to z, where z is a function of x.
dy dy dz d2 y d dy = d dy . dz
dx
= .
dz dx
and
dx 2
=
dx dx dx dz dx
d 2 y dz dy d 2 z 2
dz 2 dx
= + .
dx dx 2
Substituting in equation (1), we have
dz 2
d2 y
d2 z dz dy
dx dz 2
+
dx 2
P
dx dz
+ Qy = R
d2 y dy
or 2 + P1 + Q1y = R1 ...(2)
dz dz
d2z dz
P
dx 2
dx Q R
where P1 =
dz
2
, Q1 =
dz
2 and R1 =
dz
2
dx dx dx
P1, Q1, R1 are functions of x but may be expressed as functions of z by the given
relation between z and x.
I
dx
dz dz P dx
Integrating, log= Pdx or = e
I
dx dx
Then the equation (2) is reduced to
d2 y
+ Q1y = R1
dz 2
which can be solved easily provided Q1 comes out to be a constant or a constant multi-
1
plied by .
z2
Again, if we choose z such that
Q
Q1 =
dz 2 = a2 (constant)
dx
dz 2
dz
i.e., a2
dx =Q or a
dx
= Q
az =
Then equation (2) is reduced to
I Q dx
d2 y
dx
+ a2y = R1
2 + P1
dz dz
which can be solved easily provided P1 comes out to be a constant.
Note. It is advised to remember the equation (2) and the values of P1, Q1 and R1.
SOLVED EXAMPLES
d2 y 2 dy a 2
Example 51. Solve : + y = 0.
dx 2 x dx x 4
Sol. Choose z such that
dz 2
a2
dx =Q=
x4
dz a a
= 2 ,z=
dx x x
a
Changing the independent variable from x to z by the relation z = , we get
x
d2 y dy
+ P1 + Q1y = R1
dz 2 dz
d2z dz 2a 2 a
dx 2
P
dx
x 3
x
2
x
where P=
dz
2
a
2 =0
dx
2
x
154 Self-Instructional Material
Q R Linear Differential
Q1 = and R1 = =0
dz 2
dz 2 Equations of Second and
dx dx Higher Order
d2 y NOTES
The transformed equation is +y=0
dz 2
a a
y = c1 cos z + c2 sin z = c1 cos + c2 sin .
x x
d2 y dy
Example 52. Solve : + 4x3y = x5.
x
dx 2 dx
Sol. The given equation can be written as
d2 y 1 dy
2 + 4x2y = x4
dx x dx
dz
dz 2
Choosing z, such that
dx
dx
= 2x
= Q = 4x2
z = x2 or
Now changing the independent variable from x to z by the relation z = x2, we get
d2 y dy
2 + P 1 dz + Q1 y = R1
dz
d2 y dz 1
where P1 = dx 2
P
dx
=
2
x
2x
=0
dz
2
(2 x) 2
dx
Q R x4 x2 1
Q1 =
dz 2 =1 and R1 =
dz 2 =
(2 x) 2 =
4
z
4
dx dx
The given equation is transformed to
d2 y 1
2 +y= z
dz 4
whose C.F. = c1 cos z + c2 sin z
1 1 1 1 1
and P.I. = . 2 z = (1 + D2)1 z = 4
(1 D2 + D4 .....) z = 4
z
4 D 1 4
1
y = c1 cos z + c2 sin z + 4
z
1
or y = c1 cos x2 + c2 sin x2 + 4
x2.
d2 y dy
Example 53. Solve : 2 + cot x + 4y cosec2 x = 0.
dx dx
Sol. Choosing z, such that
dz 2
dx = 4 cosec2 x, so that
dz x
= 2 cosec x or z = 2 log tan .
dx 2
dx
Q R
Q1 =
dz
2 = 1 and R1 =
dz
2 =0
dx
dx
The given equation is transformed to
d2 y
+y=0
dz 2
y = c1 cos z + c2 sin z or y = k1 cos (z + k2)
1
or
y = k1 cos 2 log tan
2
x k2 .
d2 y dy
Example 54. Solve: (1 + x2)2 2
2 + 2x(1 + x ) dx + 4y = 0.
dx
Sol. The given equation can be written as
d2 y 2 x dy 4
2
+ + y=0
dx 1 x 2 dx (1 x) 2
Choosing z, such that
dz 2
4 dz x
dx =Q=
1 x2
dx
=
1 x2
or z = 2 tan1 x
Changing the independent variable from x to z by the relation z = 2 tan1 x,
we get
d2 y dy
2 + P1 + Q1y = R1
dz dz
d z 2
dz 4x 2x 2
P 2 2
.
2
dx (1 x ) 1 x 1 x2
2
where P1 = dx =0
dz
2
4
dx (1 x 2 ) 2
Q R
Q1 =
dz
2 = 1
and R1 =
dz
2 = 0
dx dx
d2 y
The transformed equation is +y=0
dx 2
y = c1 cos z + c2 sin z
or y = c1 cos (2 tan1 x) + c2 sin (2 tan1 x)
dx dx
R 1 2z
and R1 = = 2 2 3
dz
2
a x a
dx
The transformed equation is
d2 y 2z
+y= 3 whose C.F. = c1 cos z + c2 sin z
dz 2 a
a c a
= c1 cos
2x 2
2x 2 sin
2
a a
= c1 cos + c2 sin
2x2 2 x2
1
2z 2
and P.I. = 2
D 1 a
3 = 3 (1 + D2)1 z
a
2 2z 1
= 3 (1 D2 + D4 ....) z = 3 = 2 2
a a a x
a a 1
y= c1 cos 2
+ c2 sin 2 + 2 2
.
2x 2x a x
d2 y
dy
Example 56. Solve : cos x 2y cos3 x = 2 cos5 x.
2
+ sin x
dx dx
Sol. The given equation can be written as
d2 y dy 2 5
2 + tan x dx (2 cos x) y = 2 cos x
dx
dz
= cos x or z = sin x
dx
NOTES
Changing the independent variable from x to z by the relation z = sin x, we have
d2 y dy
2 + P1 dz + Q1 y = R 1
dz
2
d z dz
P
2
dx sin x tan x cos x 1
where P1 = dx = 0, Q1 = =2
dz
2
cos 2 x dz 2
dx dx
R
and R1 = = 2 cos2 x = 2(1 z2)
dz
2
dx
The transformed equation is
d2 y
2y = 2(1 z2)
dz 2
whose C.F. = c1 e 2 z c2 e 2z
1 D
= 1
2 1
and P.I. =
D2 2
. 2(1 z2)
2 (1 z2)
D 2
D4
= 1 ..... (1 z )
2
4 2
1
= (1 z2) + 2
(+2) = z2
y = c1 e
2z
c2 e 2z
+ z2
Required solution is = c1 e
2 sin x
c2 e 2 sin x
+ sin2 x.
d2y 1 dy
Example 57. Solve:
dx 2 + 1
x dx
+ 4x2e2xy = 4(x2 + x3) e3x.
dx
dx
d2 y dy
Example 58. Solve : x 2
4x3y = 8x3 sin x2.
dx dx
Sol. The given equation can be written as
d2 y 1 dy
4x2y = 8x2 sin x2
dx 2 x dx
Choosing z, such that
dz 2
dz
dx = 4x2 or
dx
= 2x z = x2
EXERCISE G
Solve the following differential equations:
d2 y dy d2 y dy
1. x 2 + (4x2 1) + 4x3y = 2x3 2. x4 + 2x3 + n2 y = 0
dx dx dx2 dx
Self-Instructional Material 159
Ordinary Differential d2 y dy d2 y dy
Equations 3. 2 cot x y sin2 x = 0 4. 2 + tan x + y cos2 x = 0
dx dx dx dx
d2 y dy
5. (1 + x)2 2 (1 x ) + y = 4 cos [log(1 + x)]
dx dx
NOTES
d2 y dy
6. (3 sin x cot x ) + 2y sin2 x = ecos x sin2x
dx 2 dx
d2 y dy
7. cot x y sin2x = cos x cos3x
dx 2 dx
d2 y dy
8. 2
(tan x 3 cos x ) + 2y cos2 x = cos4 x
dx dx
x
9. y (1 + 4ex)y + 3e2xy = e2(x + e ) 10. y (8e2x + 2)y + 4e4xy = e6x
Answers
x 2 1 n x
1. y = e (c1x2 + c) + 2. y = c cos
2 x
3. y = c1 e cos x c2 ecos x 4. y = c1 sin (sin x + c2)
5. y = c1 cos log(1 + x) + c2sin log(1 + x) + 2 log(1 + x) sin log(1 + x)
Here we shall explain the method of finding the complete primitive of a linear
equation whose complimentary function is known.
Let y = A(x) + B(x) be the complimentary function of the linear equation of
second order
d2 y dy
2 + P + Qy = R ...(1)
dx dx
where A and B are constants and (x) and (x) are functions of x
Since, y = A(x) + B (x)
d2 y dy
Satisfies the equation 2 +P + Qy = 0
dx dx
[A(x) + B(x)] + P [A(x) + B(x)] + Q [A (x) + B (x)] = 0
or A [ (x) + P (x) + Q(x)] + B [ (x) + P(x) + Q(x)] = 0
(x) + P (x) + Q(x) = 0 ...(2)
and (x) + P (x) + Q (x) = 0 ...(3)
Now let us assume that
y = A(x) + B(x) ...(4)
is the complete primitive of (1) where A and B are not constants but functions of x to be
so chosen that (1) will be satisfied.
dy dA dB
= A (x) + B (x) + (x) + (x)
dx dx dx
Let A and B satsify the equation,
dA dB
(x) + (x) =0 ...(5)
dx dx
dA R ( x ) R ( x )
=
dx ( x ) ( x ) ( x ) ( x ) W
dB R ( x ) R ( x )
and =
dx ( x ) ( x ) ( x ) ( x ) W
( x ) ( x )
where W= is called the Wronskian of (x) and (x).
( x ) ( x )
R ( x ) R ( x )
Integrating (7), A =
W
dx c1 , B = W
dx c2
Substituting these values of A and B in (4), we get the complete solution of (1).
Note 1. As the solution is obtained by varying the arbitrary constants of the
complementary function, the method is known as variation of parameters.
2. Method of variation of parameters is to be used if instructed to do so.
SOLVED EXAMPLES
d
Let D , the equation (2) becomes
dz
[D(D 1) + 2D 12]y = 0 or (D2 + D 12)y = 0
Its A.E. is m2 + m 12 = 0 or (m + 4) (m 3) = 0
m = 3, 4
Solution of (2) is y = c1e3z + c2e4z = c1x3 + c2x4
1
Parts of C.F. of (1) are (x) = x3 and (x) =
x4
Wronskian of (x) and (x) is
1
x3
( x ) ( x ) x4 4 3 7
W= = = .
( x ) ( x ) 4 x 2
x 2
x2
3x 2 5
x
B
Let y = A(x) + B(x) = Ax3 + be the complete solution of (1) where A and B
x4
are functions of x determined as follows:
1
x log x .
R (x ) x 4 dx c
A= dx c1 = 1
W 7
x2
1 1 1 (log x )2 1
=
7 (log x ) . dx c1 = .
x 7 2
c1
14
(log x )2 c1
R (x ) x log x . x 3
and B= W
dx c2 = 7
dx c2
2
x
1 1 x7 1 x7
.
7
= (log x ) . x 6 dx c2 = (log x ) . dx c2
7 7 x 7
x7 1 x7 x7 1
= log x . c2 log x c2
49 49 7 49 7
y = A (x) + B (x)
1 x7 1 1
= (log x )2 c1 x 3 log x c2 4
14 49 7 x
c 3 3
3 x x 1
= c1 x 24 (log x )2 log x .
x 14 49 7
dy dv d2 y d2v dv
Put y = vx so that = x v and 2
= 2
x2
dx dx dx dx dx
Substituting in (2), we have
d 2v dv x dv 1
x 2
2 x v .vx = 0
dx dx 1 x dx 1x
d 2v dv x 2 dv
or x 2 =0
dx 2 dx 1 x dx
d2v2 x dv
or
2 =0
dx x 1 x dx
dp 2 1 dv
or 1 p = 0 where p =
dx x 1 x dx
dp 1 2
or = 1 dx
p 1x x
Integrating, log p = x + log (1 x) 2log x + log c1
c1 (1 x )e x
= log
x2
c1 (1 x )ex dv c1 (1 x )e x
or p= or =
x2 dx x2
1 1
or dv = c1 2 e x dx Form [ f ( x ) f ( x )] e x
x x
1
Integrating, v = c1 e x c2 | f(x)ex
x
Solution of (2) is y = vx = c1ex + c2x
Parts of C.F. of (1) are (x) = ex and (x) = x
( x ) ( x ) ex x
Wronskian of (x) and (x) is W= = = (x 1)ex
( x ) x e x
1
R ( x ) (1 x )x
A= dx c1 = dx c1 xe x dx c1
NOTES W ( x 1)e x
= x ( e x ) 1 . ( e x ) dx c1 = xex ex + c1
= (x + 1)ex + c1
R (x ) (1 x ) ( e x )
and B= W
dx c2 = ( x 1)e x
dx c2 dx c2
= x + c2
Hence, the complete solution of (1) is
y = A(x) + B(x)
= [ (x + 1)ex + c1] ( ex) + (x + c2)x
or y = c1ex + c2x + x + 1 + x2
where c1 and c2 are arbitrary constants of integration.
EXERCISE H
Using method of variation of parameters, solve the following differential equations:
d2 y dy d2 y dy 1
1. x2 2
4x 2 y ex 2. x 2 4x 2y x 2 2
dx dx dx 2 dx x
d2y dy d2 y dy
3. x2 2
x 9 y 48 x 5 4. x 2 x y x log x
dx dx dx 2 dx
d2y dy d2 y dy
5. x2 2
4x 6 y = sin (log x) 6. x 2 x x 3ex
dx dx dx 2 dx
d2 y dy d2 y dy
7. x2 x y x 2 log x 8. (1 x ) x y 2(1 x )2 e x
dx 2
dx dx 2 dx
Answers
c1 c2 1 c1 c2 1 2 1
1. y= 2
2
ex 2. y = x 2 log x
x x x x x2 12 x
x
3. y = c1 x3 + c2x3 + 3x5 4. y = c1 x log x + c2x + (log x )3
6
1
5. y = c1 x2 + c2x3 + [(sin (log x) + cos (log x)]
10
c2 1 3 4
6. y = c1 + c2 x2 + (x 1)ex 7. y = c1x x log x x 2
x 3 9
1
8. y = c1x + c2ex + x e x
2
NOTES
5. POWER SERIES SOLUTIONS
STRUCTURE
Introduction
Definitions
Power series solution, when x = 0 is an ordinary point of the equation
d2 y dy
2
P(x ) + Q(x)y = 0
dx dx
Frobenius Method: Series solution when x = 0 is a regular singular point of
the differential equation
d2 y dy
2
P(x ) + Q(x)y = 0
dx dx
INTRODUCTION
The solution of ordinary linear differential equations of second order with variable
coefficients in the form of an infinite convergent series is called solution in series or
integration in series.
The series solution of certain differential equations give rise to special functions
such as Bessels function, Legendres polynomials, Laguerres polynomial, Hermites
polynomial, Chebyshev polynomials. These special functions have wide applications
in engineering.
In this unit, we will discuss methods of solution of second order linear differential
equations with variable coefficients in series along with Bessels function, Legendres
polynomial and their properties.
DEFINITIONS
Power Series
An infinite series of the form
n0
an (x x0)n = a0 + a1 (x x0) + a2 (x x0)2 + ...
n0
an xn = a0 + a1x + a2x2 + ...
NOTES
xn x 2 x 3 ...
e.g., ex =
n0
n!
1 x
2! 3!
+
Analytic Function
A function f(x) defined on an interval containing the point x = x0 is called analytic at x0
f ( n) ( x0 )
if its Taylor series
n0
n!
(x x0)n exists and converges to f(x) for all x in the
Ordinary Point
A point x = x0 is called an ordinary point of the equation
d2 y dy
P( x) + Q(x) y = 0 ...(1)
dx 2 dx
if both the functions P(x) and Q (x) are analytic at x = x0.
Remark 2. When x = 0 is a regular singular point of equation (1), at least one of its
solution can be expressed as
y = a0xm + a1xm+1 + a2xm+2 + ... = xm (a0 + a1x + a2x2 + ...) = a
n0
n xm n
SOLVED EXAMPLES
Example 1. Solve in series the differential equation
d2 y
+ xy = 0.
dx 2
d2 y dy
Sol. Comparing the given equation with the form + P(x) + Q(x) y = 0,
dx 2 dx
we get P(x) = 0, Q(x) = x
At x = 0, both P(x) and Q(x) are analytic, hence x = 0 is an ordinary point.
Assume its solution to be
y = a0 + a1x + a2x2 + a3x3 + ... + anxn + ... ...(1)
dy
Then, = a1 + 2a2x + 3a3x2 + 4a4x3 + ... + n anxn1 + ...
dx
d2 y
and = 21 a2 + 32 a3x + 43 a4x2 + 54 a5x3 + ... + n(n 1) anxn2 + ...
dx 2
Substituting these values in the given differential equation, we get
[21 a + 32 a x + 43 a x2 + 54 a x3 + ... + n(n 1) a xn2 + ...]
2 3 4 5 n
+ x [a0 + a1x + a2x2 + a3x3 + ... + anxn + ...] = 0
21 a2 + (32 a3 + a0) x + (43 a4 + a1)x2 + (54 a5 + a2)x3 + ...
+ {(n + 2) (n + 1) a + a }xn + ... = 0
n+2 n1
NOTES Coefficient of x = 0
32 a3 + a0 = 0
a0 a0
a3 = a3 =
32 3!
Coefficient of x2 = 0
43 a4 + a1 = 0
a1 2 a1
a4 = or a4 =
4 3 4!
Coefficient of x3 = 0
54 a5 + a2 = 0
a2
a5 = or a5 = 0
54
Coefficient of x4 = 0
65 a6 + a3 = 0
a3 a0 4 a0
a6 = or a6 =
6 5 6 53 ! 6!
and so on.
Coefficient of xn = 0
(n + 2) (n + 1) an+2 + an1 = 0
an 1
an+2 =
(n 2) (n 1)
which is the recurrence relation.
Putting n = 5, 6, 7, ..., successively in recurrence relation, we obtain
5 2a1 74
a7 = , a8 = 0, a9 = a0 and so on.
7! 9!
Substituting these values in (1), we get
a 2a 4a 5.2 a1 7 7.4
y = a0 + a1x 0 x 3 1 x 4 0 x 6 x a x9 + ...
3! 4! 6! 7! 9! 0
y = a0 1
x 3 1.4 6 1.4.7 9 ...
x x
"#
+ a1 x x
2 4 2.5 7 ...
x
"#
! 3! 6! 9! #$ 4! !7! $
where a0 and a1 are constants.
Example 2. Solve in series the differential equation
d2 y dy
(1 + x2)
2
x y = 0 about the point x = 0.
dx dx
Sol. Comparing the given differential equation with the form
d2 y dy
P( x) + Q(x) y = 0, we get
dx 2 dx
x 1
P(x) = and Q(x) = .
1 x 2 1 x2
170 Self-Instructional Material
Both P(x) and Q(x) are analytic at x = 0 Power Series Solutions
x = 0 is an ordinary point of the given differential equation.
Assume the solution to be
y = a0 + a1x + a2x2 + a3x3 + ... + anxn + ... ...(1) NOTES
dy
Then, = a1 + 2a2x + 3a3x2 + ... + nanxn1 + ...
dx
d2 y
and = 21 a2 + 32 a3x + ... + n(n 1)an xn2 + ...
dx 2
Substituting these values in given equation, we get
(1 + x2) [21 a + 32 a x + 43 a x2 + ... + n(n 1) a xn2 + ...]
2 3 4 n
+ x [a1 + 2a2x + 3a3x2 + 4a4x3 + ... + n anxn1 + ...]
[a + a x + a x2 + a x3 + ... + a xn + ...] = 0
0 1 2 3 n
0
Coefficient of x0 = 0
a0
2.1. a2 a0 = 0 a2 =
2
Coefficient of x = 0
3.2 a3 + a1 a1 = 0 a3 = 0
Coefficient of x2 = 0
2.1 . a2 + 4.3.a4 + 2a2 a2 = 0
4.3 a4 + 3a2 = 0
a2 a a0
a4 = 0 or a4 =
4 8 8
Coefficient of x3 = 0
5.4. a5 + 3.2. a3 + 3a3 a3 = 0
20a5 + 8a3 = 0 a5 = 0
4
Coefficient of x = 0
6.5 . a6 + 4.3 . a4 + 4a4 a4 = 0
30a6 + 15a4 = 0
a4 a a0
a6 = = 0 or a6 =
2 16 16
Similarly, a7 = 0, a9 = 0, a11 = 0 and so on.
Also, Coefficient of xn = 0
(n + 2) (n + 1) an+2 + n(n 1)an + nan an = 0
n 1 a '
an+2 =
n 2 n | n+10
x 4 x 6 ...
y = a0 1 x 2
3
5
+ a1x
where a0 and a1 are constants.
...(1)
dy
dx n 0
n an x n 1 ...(2)
d2 y
dx 2
n(n 1) a
n0
n xn 2 ...(3)
n0
n(n 1) an x n 2
n0
an[n(n 1) + 2n p(p + 1)]xn = 0
n0
n(n 1) an x n 2 a (n p) (n + p + 1)x
n0
n
n =0
This is an identity in x.
Coefficient of xn = 0
(n + 2) (n + 1) an+2 (n p) (n + p + 1) an = 0
(n p) (n p 1)
an+2 = an
(n 2) (n 1)
Putting n = 0, 2, 4, ..... etc., we get
p ( p 1)
a2 = a0
2.1
(2 p) (3 p) ( p 2)( p) ( p 1) ( p 3)
a4 = a2 a0 etc.
4 .3 4!
Again, putting n = 1, 3, 5, ...... etc., we get
(1 p) ( p 2) ( p 1) ( p 2)
a3 = a1 a1
3. 2 3!
(3 p) ( p 4) ( p 3) ( p 1) ( p 2) ( p 4)
a5 = a3 a1 etc.
5.4 5!
y = a0 1
p ( p 1) 2 ( p 2) p ( p 1)( p 3) 4 ......
x x
"#
! 2! 4! $
+ a1 x
( p 1)( p 2) 3 ( p 3)( p 1)( p 2)( p 4 ) 5 ......
x x
"# NOTES
! 3! 5! $
Note. Above method is an aliter to the method of solution in series discussed before and
preferred when, we get the recurrence relation in between an and an+2.
d 2 y d dy
d dy d2 y
dx 2 dx dx
dt dt
dt 2
The given equation becomes,
d2 y
+ t2y 4ty = 0 ...(1)
dt 2
Now, t = 0 is an ordinary point. | given
Assume the solution to be
y = a0 + a1t + a2t2 + a3t3 + ... + antn + ... ...(2)
then y = a1 + 2a2t + 3a3t2 + ... + n an tn1 + ...
and y = 2a2 + 3.2. a3t + ... + n (n 1) an tn2 + ...
Substituting these values in eqn. (1), we get
[2a2 + 3.2. a3t + 4.3. a4 t2 + ... + n(n 1) an tn2 + ...]
+ t2 [a1 + 2a2t + 3a3 t2 + 4a4t3 + ... + n antn1 + ...]
4t [a0 + a1t + a2t2 + a3t3 + ... + antn + ...] = 0
Coefficient of t0 = 0
2a2 = 0 a2 = 0
Coefficient of t = 0
2a0
3.2. a3 4a0 = 0 a3 =
3
Coefficient of t2 = 0
4.3. a4 + a1 4a1 = 0
a1
12a4 = 3a1 a4 =
4
Coefficient of t3 = 0
5.4. a5 + 2a2 4a2 = 0 a5 = 0
EXERCISE A
Answers
x x x ... a x x x ... = a cosh x + a
2 4 6 3 5
1. 2 ! 4 ! 6 ! 3 ! 5 !
y = a0 1 1 0 1 sinh x
1 x x ... a x x x ...
4 8 5 9
2. y = a0
3 . 4 3.4 .7.8 4 .5 4 .5.8.9
1
x x 3 x 3.5 x ... + a x
(ii) y = a 1
2 4
0
2! 4! 6! 8! 6 8
1
NOTES
x x ... a x x x x ...
(i) y = a 1
4 6 3 5 7
4. 0
12 90 6 40 144 1
x x ... a x x x ...
(ii) y = a 1
4 6 3 5
0
12 90 6 40 1
x x ... a x x x ...
y = a 1
2 4 3 5
5. 0
2 8 2 40 1
x x 5x ... a x x x ...
y = a 1
2 3 4 3 4
6. 0
4 12 96 6 24 1
3 5
y = a 1 x 3 x ... a x x x 3 x ...
3 4 5
7. 0 6 40 6 12 40 1
dy d 2 y
2. Substitute from (1) for y, , in given equation.
dx dx 2
3. Equate to zero the coefficient of lowest power of x. This gives a quadratic
equation in m which is known as the Indicial equation.
4. Equate to zero, the coefficients of other powers of x to find a1, a2, a3, ... interms
of a0.
y = c1 ( y) m1 c2 ( y) m2
SOLVED EXAMPLES
! 2m 5 2m 7 2 m 9
...(2)
$
Now, y1 = (y)m=0
3 3 3
x x 2 x 3 ...
"#
y1 = a0 1
! 5 7 9 $ ...(3)
Coefficient of xm+2 = 0
2(m + 2) (m + 1) a2 + 2(m + 1) a1 (m + 2) a2 + a2 = 0
(2m2 + 5m + 3) a2 + 2(m + 1) a1 = 0
(2m + 3) (m + 1) a2 + 2(m + 1) a1 = 0
y1 = a0 x 1
2 x 4 x 2
8
x 3 ...
"#
! 3 3.5 3.5.7 $
or y1 = a0
2 2 x
x 1 x
2
2
23
x3
... ...(3)
3 3.5 3.5.7
and y2 = (y)m=1/2
1 2 1 3 "#
!
y2 = a0 x1/2 1 x
2
x x ...
6 $ ...(4)
EXERCISE B
Solve in series:
d2 y dy d2 y dy
1. 9x (1 x) 12 + 4y = 0 2. x (2 + x2) 2
6xy = 0
2 dx dx dx
dx
d2 y dy d2 y dy
3. 3x 2 y0 4. 2x2 x + (1 x2) y = 0
dx 2 dx dx2 dx
Answers
3 1 6 3 2 3.1 4 5.3.1 6
2. y = A 1 3 x 2 x 4 x ... + Bx3/2 1 x x x ...
5 15 8 8.16 8.16.24
x x2 x3 x x2 x3
3. y = A 1
2 20 480
... + Bx1/3 1
4 56 1680
...
x2 x4 x2 x4
4. y = Ax 1
2.5 2.4 .5.9
... + Bx1/2 1
2.3 2.4 .3.7
...
5. y = Ax 1 1 x 1 x 2 ... Bx 1/2 1 2
1 x 2 x ...
5 70
2 3 4
6. y = A 1 3x 3x 3x 3x ... + B x (1 x)
1.3 3.5 5.7
x x2 x3 x x2 x3
7. y c1x5 / 2 1
9 198 7722
... c2 x 1 1
5 30 90
...
8. y = A x + Bx1/4
x2 x4 x2 x4
9. y = Ax 1
10 360
... Bx1/2 1
6
168
...
x x2 x3 1 x x x ... . 2 3
10. y = A 1
2 3
2 .1 ! 2 .2 ! 2 .3 !
... + B x
1.3 1.3.5 1.3.5.7
7 49 2 3 21 49 2
11. y = A x 1 x x + Bx 1 x x
18 264 5 5
x2 x3 2 x 4 x 2 8x 3
12. y = Ax1/2 1 x .... + Bx 1
....
2 6 5 35 315
d2 y dy
Example 9. Solve in series: x (x 1) 2
(3x 1) y 0.
dx dx
NOTES
Sol. Comparing the given equation with
d2 y dy
2
P( x) + Q(x) y = 0, we get
dx dx
3x 1 1
P(x) = and Q(x) =
x ( x 1) x ( x 1)
At x = 0, Both P(x) and Q(x) are not analytic, hence x = 0 is a singular point.
3x 1 x
Now, x P(x) = and x2 Q(x) =
x1 x1
Both x P(x) and x2 Q(x) are analytic at x = 0, hence x = 0 is a regular singular
point.
Let us assume
y = a0 xm + a1xm+1 + a2 xm+2 + a3 xm+3 + ... ...(1)
Then, y = m a0 xm1 + (m + 1) a1 xm + (m + 2) a2 xm+1 + (m + 3) a3 xm+2 + ...
and y = m (m 1) a0 xm2 + (m + 1)m a1 xm1
+ (m + 2) (m + 1) a2 xm + (m + 3) (m + 2) a3 xm+1 + ...
Substituting these values in given equation, we get
x (x 1) [m (m 1) a0 xm2 + (m + 1) m a1 xm1 + (m + 2) (m + 1) a0 xm
+ (m + 3) (m + 2) a3 xm+1 + ...]
+ (3x 1) [m a0 xm1 + (m + 1) a1 xm + (m + 2) a2 xm+1 + (m + 3) a3 xm+2 +...]
+ [a0 xm + a1 xm+1 + a2 xm+2 + a3 xm+3 + ...] = 0
Now, Coefficient of lowest power of x = 0
Coefficient of xm1 = 0
m (m 1) a0 m a0 = 0 m2 a0 = 0
m2 = 0 '
( a0 0)
which is Indicial equation
a1 = a0 '
( m 1)
Coefficient of xm+1 = 0
(m + 1) m a1 (m + 2) (m + 1) a2 + 3(m + 1) a1 (m + 2)a2 + a1 = 0
(m + 2)2 a1 (m + 2)2 a2 = 0
a2 = a1 ( '
m 2)
a2 = a0
Similarly, we can show that
a3 = a0
a4 = a0 and so on.
y
NOTES
y2 =
m m0
= [a0 (1 + x + x2 + x3 + ...) xm log x]m=0
d2 y dy
Example 10. Solve in series the differential equation: x y 0.
2
dx dx
d2 y dy
Sol. Comparing with the equation 2
P( x) + Q(x) y = 0, we get
dx dx
1 1
P(x) =
and Q(x) =
x x
Since at x = 0, both P(x) and Q(x) are not analytic x = 0 is a singular point.
Also, x P(x) = 1 and x2 Q(x) = x
Both x P(x) and x2 Q(x) are analytic at x = 0 x = 0 is a regular singular
point.
Let us assume
y = a0 xm + a1 xm+1 + a2xm+2 + a3 xm+3 + ... ...(1)
Then, y = m a0 xm1 + (m + 1) a1 xm + (m + 2) a2 xm+1 + (m + 3) a3 xm+2 + ...
and y = m(m 1) a0 xm2 + (m + 1) m a1 xm1
+ (m + 2) (m + 1) a2 xm + (m + 3) (m + 2) a3 xm+1 + ...
Substituting these values in the given equation, we get
x [m (m 1) a0 xm2 + (m + 1)m a1 xm1 + (m + 2) (m + 1) a2xm
+ (m + 3) (m + 2) a3 xm+1 + ...]
+ [m a0 xm1 + (m + 1) a1 x + (m + 2) a2 xm+1 + (m + 3) a3 xm+2 + ...]
m
a0
a1 =
(m 1) 2
Coefficient of xm+1 = 0
(m + 2) (m + 1) a2 + (m + 2) a2 a1 = 0 (m + 2)2 a2 = a1
a0
Similarly, a3 = and so on. NOTES
(m 1) (m 2) 2 (m 3) 2
2
From (1),
y = a0xm 1
x
x2
x3
...
"#
! (m 1) 2 (m 1) 2 (m 2) 2 (m 1) 2 (m 2) 2 (m 3) 2 $
...(2)
Now, y1 = (y)m=0 = a0 1 x
x2
x3
...
"# ...(3)
! (2 !) 2 (3 !) 2 $
To get the second independent solution, differentiate (1) partially w.r.t. m.
y x x2 x3
= a0xm log x 1 2
2 2
2 2 2
...
m (m 1) (m 1) (m 2) (m 1) (m 2) (m 3)
+ a0xm
2x %& 1 1 () x
2 2
(m 1) ( m 1) (m 2) ' m 1 m 2 *
! 3 2 2
2 %& 1 1 1 () x ..."# 3
( m 1) ( m 2) (m 3) ' m 1 m 2 m 3 *
2 2
$ 2
y = a log x 1 x x x ..."#
The second solution is y =
2 3
m
2
! (2 !)
m0 (3 !) $0 2 2
EXERCISE C
Solve in series:
d2 y dy
1. (i) xy + (1 + x) y + 2y = 0 (ii) x 2
xy 0
dx dx
Answers
3 2 4 3 13 2
1. (i) y = A 1 2x x x ... B y1 log x a0 3x 4 x ...
2! 3!
x2 x4 x2 3x 4
(ii) y = (A + B log x) 1 2 2 2 ... B 2 ...
2 2 .4 2 2.4 3
y = Ax + B x log x x
x2
...
"#
2.
! 4 #$
3. y = A (12 + 22x + 32x2 + 42x3 + ...) + B [y1 log x 2a0 (1.2x + 2.3 x2 + 3.4 x3 + ...)]
2 2.5 3 2 14 3
4. y = A 1 x x 2 x ... + B y1 log x a0 2x x x ...
4 4.9 27
1 1 3 3
5. y = Ax 1 x x 2 x ... B y1 log x a0 x 2 1 4 x ...
2 2.3
y = A 1
x3
x6
x9 "#
...
6.
! 3 2 4
3 (2 !) 2 6
3 (3 !) 2
#$
x
3
1 1 6
+ B y1 log x 2 a0 3 5 1 2 x ...
3 3 (2 !)2
x2 x4 x6 x 2 1 1
7. y = A 1 2 2 2 2 2 2 ... + B y1 log x a0 2 2 2 1 x 4
2
2 2 .4 2 .4 .6 2 2 .4
1 1 1 6
+ 2 1 x ... .
2 2
2 .4 .6 2 3
Case III. When Roots are Distinct, Differ by Integer and Making a
Coefficient of y Infinite
Let m1 and m2 be the roots such that m1 > m2.
In this case, if some of the coefficients of y become infinite when m = m2, we
modify the form of y by replacing a0 by b0 (m m2).
Complete solution is
y
y = c1 ( y) m c2
1
m m2
.
Example 11. Obtain the series solution of the Bessels equation of order two
d2 y dy
x2
2
x + (x2 4) y = 0 near x = 0. NOTES
dx dx
Sol. Comparing the given equation with the form
d2 y dy
2
P( x) + Q(x)y = 0, we get
dx dx
1 x2 4 4
P(x) = and Q(x) = 1 2
x x 2
x
At x = 0, both P(x) and Q(x) are not analytic.
Therefore x = 0 is a singular point.
Also, x P(x) = 1 and x2 Q(x) = x2 4
Both x P(x) and x2 Q(x) are analytic at x = 0
x = 0 is a regular singular point.
Let us assume,
y = a0xm + a1 xm+1 + a2 xm+2 + a3 xm+3 + ... ...(1)
dy
Then, = m a0 xm1 + (m + 1) a1 xm + (m + 2) a2 xm+1 + (m + 3) a3 xm+2 + ...
dx
d2 y
and = m(m 1) a0 xm2 + (m + 1) ma1 xm1 + (m + 2) (m + 1) a2 xm
dx 2
+ (m + 3) (m + 2) a3xm+1 + ...
Substituting these values in the given equation, we get
x2 [m(m 1) a0 xm2 + (m + 1) ma1 xm1
+ (m + 2) (m + 1) a2 xm + (m + 3) (m + 2) a3 xm+1 + ...]
+ x [m a0 xm1 + (m + 1) a1 xm + (m + 2) a2 xm+1 + (m + 3) a3 xm+2 + ...]
+ (x2 4) [a0 xm + a1 xm+1 + a2 xm+2 + a3 xm+3 + ...] = 0
Now, Coefficient of lowest power of x = 0
Coefficient of xm = 0
m(m 1)a0 + m a0 4a0 = 0 (m2 4) a0 = 0
m2 4 = 0 (Indicial equation) | ' a0 0
m = 2, 2
a1 = 0 Since m 1, and
m3
Coefficient of xm+2 = 0
(m + 2) (m + 1) a2 + (m + 2) a2 + a0 4a2 = 0
(m2 + 4m) a2 + a0 = 0
Coefficient of xm+3 = 0
NOTES (m + 3) (m + 2) a3 + (m + 3) a3 + a1 4a3 = 0
(m + 1) (m + 5) a3 = a1
a3 = 0 ' a1 0
Similarly, a5 = a7 = a9 = ... = 0
a0
a6 = etc.
m (m 2) ( m 4) 2 (m 6) (m 8)
Substituting above obtained values in assumed y given by eqn. (1), we get
y = a0xm 1
x2
x4
! m (m 4) m (m 2) (m 4) (m 6)
x6
...
"# ...(2)
m (m 2) (m 4) 2 (m 6) (m 8) $
Putting m = 2 (the greater of the two roots) in (2), the first solution is
x2 x4 x6
y1 = a0x2 1
2.6 2.4.6.8 2.4.6 2 .8.10
...
If we put m = 2 in (1), the coefficients become infinite due to the presence of
the factor (m + 2) in the denominator. To overcome this difficulty, let a0 = b0 (m + 2) so
that
(m 2)x 2 x4 x6
y = b0xm (m 2) 2
...
m (m 4) m (m 4)(m 6) m (m 4) (m 6) (m 8)
Differentiating partially w.r.t. m, we get
y
= b0xm log x (m 2)
(m 2) x 2
x4
...
"#
m ! m (m 4) m (m 4) (m 6) $
(m 2) 1 %&1 1 () x
+ b0xm 1
! ' *
2
m (m 4) m 2 m m 4
+
1 %&
1
1
1
x 4 ...
() "#
'
m (m 4) (m 6) m m4 m6 * $
y
The second solution is y2 =
m m 2
1 x 1 1 1 1 x ..."#
2
= b0x2
1 x ..."# b x 1 x x ..."#
log x
2
2
2 4
! 2 . 4 2 . 4 .6 $
2 3 0
! 2 2 .4 $ 2 2 2
d2 y dy
Example 12. Solve in series the differential equation x2 2
5x x2 y 0 .
dx dx
Sol. Comparing the given equation with the form
d2 y dy
P( x) + Q(x) y = 0, we get
dx 2 dx
5
P(x) = , Q(x) = 1
x
At x = 0, since P(x) is not analytic x = 0 is a singular point.
Also, x P(x) = 5
x2 Q(x) = 0
Since both x P(x) and x2 Q(x) are analytic at x = 0 x = 0 is a regular singular
point.
Let us assume
y = a0 xm + a1xm+1 + a2 xm+2 + a3 xm+3 + ... ...(1)
dy
= m a0 xm1 + (m + 1) a1 xm + (m + 2) a2 xm+1 + ... ...(2)
dx
d2 y
and = m (m 1) a0 xm2 + (m + 1) m a1 xm1 + (m + 2) (m + 1) a2 xm + ...
dx 2
...(3)
Substituting the above values in given equation, we get
x2 [m(m 1) a0 xm2 + (m + 1) ma1 xm1 + (m + 2) (m + 1) a2 xm + ...]
+ 5x [ma0 xm1 + (m + 1) a1 xm + (m + 2) a2 xm+1 + ...]
+ x2 [a0xm + a1 xm+1 + a2 xm+2 + ...] = 0 ...(4)
Equating the coefficient of lowest power of x to zero, we get
m(m 1) a0 + 5ma0 = 0 [Coeff. of xm = 0]
(m2 + 4m) a0 = 0
m(m + 4) = 0 (Indicial equation) '
( a0 0)
m = 0, 4
Self-Instructional Material 189
Ordinary Differential Hence the roots are distinct and differing by an integer. Equating to zero, the
Equations coefficients of successive powers of x, we get
Coefficient of xm+1 = 0
(m + 1) m a1 + 5(m + 1) a1 = 0
NOTES
(m + 5) (m + 1)a1 = 0 a1 = 0 ...(5)
| ' m 5, 1
Coefficient of xm+2 =0
(m + 2) (m + 1) a2 + 5(m + 2)a2 + a0 = 0
(m + 2) (m + 6) a2 + a0 = 0
a0
a2 = ...(6)
(m 2) (m 6)
a3 = 0 ...(7)
Similarly, a5 = a7 = a9 = ... = 0
Now, Coefficient of x m+4 =0
(m + 4) (m + 3) a4 + 5(m + 4) a4 + a2 = 0
(m + 4) (m + 8) a4 = a2
a2 a0
a4 = etc. ...(8)
(m 4) (m 8) (m 2) (m 4) (m 6) (m 8)
x2 x4
These give y = a0 xm 1 ...
(m 2)(m 6) (m 2)(m 4)(m 6)(m 8)
...(9)
Putting m = 0 in (9), we get
2 4
y1 = (y)m=0 = a0 1 x x ... ...(10)
2.6 2.4.6.8
If we put m = 4 in the series given by eqn. (9), the coefficients become infinite.
To avoid this difficulty, we put a0 = b0 (m + 4), so that
(m 4) x 2 x4
y = b0 xm (m 4) ... ...(11)
( m 2)( m 6) ( m 2)( m 6)( m 8)
EXERCISE D
Solve in series:
d2 y dy d2 y dy
1. x (1 x) 2
3x y0 2. x2 2
x + (x2 1) y = 0
dx dx dx dx
(Bessels equation of order one)
d2 y dy d2 y dy
3. (x + x2 + x3) 2
3 x2 2y 0 4. x (1 x) (1 3 x) y 0.
dx dx 2 dx
dx
Answers
1. y = (A + B log x) (x + 2x2 + 3x3 + 4x4 + ...) + B (1 + x + x2 + x3 + ...)
x2 x4
x2 x4
2.
y = Ax 1 2
2.4 2.4 .6
... + Bx1 log x
2 2 ...
2 .4
1 x 2
3
x 4 ...
"#
+ Bx 1
! 2 2 2
2 .2 3
#$
1 2 1 3 "
3. y = Ax 1 x x x ...# + B log x (2x + 2x2 x3 + ...) + B(1 x 5x2 x3 + ...)
! 2 2 $
Case IV. When Roots are Distinct, Differ by Integer and Making One or
More Coefficients Indeterminate
Let the roots be m1 and m2. If one of the coefficients (suppose a1) become indeterminate
when m = m2, the complete solution is given by putting m = m2 in y which then contains
two arbitrary constants.
Note. The result contained by putting m = m1 in y merely gives a numerical multiple of
one of the series contained in the first solution. Hence we reject the solution obtained by putting
m = m1.
m = 0, 1
a0
a2 =
(m 2) (m 3)
192 Self-Instructional Material
Coefficient of xm+2 = 0 Power Series Solutions
(m + 3) (m + 2) a3 + 2(m + 3) a3 + a1 = 0
(m + 3) (m + 4) a3 + a4 = 0
– a1 NOTES
a3 =
(m 3)(m 4)
Coefficient of xm+3 = 0
(m + 4) (m + 3) a4 + 2(m + 4) a4 + a2 = 0
(m + 4) (m + 5) a4 = a2
a2
a4 =
(m 4) (m 5)
a0
a4 =
(m 2) (m 3) (m 4) (m 5)
Coefficient of xm+4 = 0
(m + 5) (m + 4) a5 + 2(m + 5) a5 + a3 = 0
(m + 5) (m + 6) a5 = a3
a1
a5 =
(m 3) (m 4) (m 5) (m 6)
and so on.
Substituting these values in eqn. (1), we get
a0 a1 a0
y = xm a0 a1 x x2 x3 x4
(m 2)( m 3) ( m 3)(m 4) ( m 2)( m 3)(m 4)( m 5)
a1
x 5 ...
"#
+
(m 3) (m 4) (m 5) (m 6) $
% x2 x4 ()
&
y = xm a0 1 ...
! ' (m 2) (m 3) (m 2) (m 3) (m 4) (m 5) *
%&
+ a1 x
x3
x5 ()"#
...
' (m 3) ( m 4) ( m 3) (m 4) (m 5) ( m 6) *#$
x2 x4 x x x ... "#
3 5
! 2.3 2.3.4.5 #$
Now, (y)m=1 = x1 a0 1 ... a1
1.2 1.2.3.4
1
y= (a cos x + a1 sin x).
x 0
Note. All those problems, in which x = 0 was an ordinary point of y + P(x) y + Q(x) y =
0, can also be solved by Frobenius method as given in Art. 2.4.4 and explained in above illustrative
example.
n (n 1) 2 (n 2) n (n 1) (n 3) 4 "
3. y = a0 1 x x ...#
! 2! 4! $
(n 1) (n 2) 3 (n 3) (n 1) (n 2) (n 4) 5 "
+ a1 x x x ...#
! 3! 5! $
STRUCTURE
Introduction
Legendres Function of First Kind Pn(x)
Legendres Function of Second Kind Qn(x)
Solution of Legendres Equation
Generating Function for Pn(x)
Rodrigues Formula
Recurrence Relations
Beltramis Result
Orthogonality of Legendre Polynomials
Laplaces Integral of First Kind
Laplaces Integral of Second Kind
Cristoffels Expansion Formula
Cristoffels Summation Formula
Expansion of a Function in a Series of Legendre Polynomials
(Fourier-Legendre Series)
INTRODUCTION
dy d2 y
The differential equation (1 x2)
+ n(n + 1)y = 0
2
2x ...(1)
dx dx
where n is real number, is called Legendres differential equation. This equation
is of considerable importance in applied mathematics, particularly in boundary value
problems involving spherical configurations.
Though n is a real number, in most physical applications, only integral values
of n are required. Also, equation (1) can be solved in series of ascending or descending
powers of x. The solution in descending powers of x is more important than the one in
ascending powers.
Let y=
k0
ak x m k
d2 y
then
dy
dx
(m k) a k x m k1
and
dx 2
k0
(m k) (m k 1) ak x m k 2
k0
or
k0
( m k) (m k 1) ak xmk2 [(m k) (m k 1)
k0
+ 2(m k) n (n + 1)]ak xmk = 0
or
k0
( m k) (m k 1) ak xmk2 (m k)
k0
2 n2 + (m k) n] ak xmk = 0
or (m k) (m k 1) a k xmk2
k0
[(m k n) (m k + n + 1) ak xmk = 0.
k0
Equating to zero the coefficient of highest power of x, i.e., xm, we get the indicial
equation
(m n) (m + n + 1) a0 = 0
whence m = n or m = (n + 1) since a0 0
Equating to zero the coefficient of the next lower power of x, i.e., xm1, we get
(m + n) (m n 1) a1 = 0 or a1 = 0,
since (m + n) and (m n 1) are not zero for m = n or (n + 1).
Equating to zero the coefficient of xmk, we get the recurrence relation
[m (k 2)] [m (k 2) 1] ak2 (m k n) (m k + n + 1) ak = 0
(m k 2) (m k 1)
or ak = ak 2 ...(2)
(n m k) (n m k 1)
1. 3 . 5 ...... (2n 1)
When n is a positive integer and a0 = ,
n!
the first solution given by (3) is denoted by Pn (x) and is called Legendres function of
first kind.
Thus,
n!
When n is a positive integer and a0 = ,
1. 3 . 5 ... (2n 1)
the second solution is denoted by Qn (x) and is called Legendres function of second
kind.
Since y = Pn (x) and y = Qn (x) both are the solutions of the given equation hence the
most general solution is given by
We shall show that Pn(x) is the coefficient of hn in the expansion of (1 2xh + h2)1/2 in
ascending powers of h.
1 . 3 . 5 ...... (2n 1) n n (n 1) n 2 n (n 1) (n 2) (n 3) n 4
x x x ... = Pn(x)
"#
n! ! 2 (2n 1) 2 . 4 . (2n 1) (2n 3) $
Thus, in the expansion of (1 2xh + h2)1/2, P1 (x), P2(x), P3(x), ......, Pn(x), ...... are
the coefficients of h, h2, h3, ......, hn, ..... respectively.
The function (1 2xh + h2)1/2 is called the generating function for Pn(x).
SOLVED EXAMPLES
h
n0
n
Pn (1) (1 2 h h2 ) 1/ 2 (1 h) 1
=1+h+ h2 + ...... + hn + ...... = h
n0
n
n0
( h)n Pn(x) = (1 + 2xh + h2)1/2
NOTES
From (2) and (3), hn Pn ( x ) ( 1)n hn Pn ( x )
n0 n0
Putting x = 0, we get h
n0
n
Pn (0) (1 h2 ) 1/ 2
(ii) Equating the coefficients of h2n+1 on both sides, we get P2n+1 (0) = 0, since the
right-hand side contains only even power of h.
Example 3. Prove that:
1
(i)
n0
P n (x)
2 2x
(ii) Pn (1) = (1)n.
(1 2x h + h2)1/2 = h
n0
n
Pn ( x) ...(1)
1
2 2x
P
n0
n ( x)
(x z) (1 2zx + z2)3/2 = nz
n0
n1
Pn ( x) ...(2)
1 z2
(1 2 zx z 2 ) 3 / 2
n0
(2n 1) z n
Pn ( x) .
1 z 1
Example 5. Prove that:
z 1 2xz z 2
z n0
(Pn Pn 1 ) z n .
Sol. RHS =
n0
Pn z n P
n0
n1 zn
1
= P
n0
n zn
z n0Pn 1 z n 1
1
= P
n0
n zn (P z + P2z2 + P3z3 + ... + Pnzn + ...)
z 1
1
= P
n0
n zn
z
{ P0 + P0z0 + P1z + P2z2 + ... + Pnzn + ...} | ' P0 = 1
1 1
1 P
1
=
n0
Pn z n
z z n0
Pn z n = 1
z n0
n zn
z
...(1)
1 z 1
1 1
LHS =
z 1 2 xz z 2
z
= 1
z
(1 2xz + z2)1/2
z
1
= 1 z
1
z
n
Pn ...(2)
n0
z
Hence the proof. | Since LHS = RHS
Self-Instructional Material 201
Ordinary Differential Example 6. Prove that:
Equations
n (n 1)
(i) Pn (1) =
2
n(n 1) n (n 1)
(ii) Pn ( 1) = ( 1)n1 . Or Pn ( 1) = ( 1)n+1 . .
NOTES 2 2
Sol. Legendres differential equation is
d2 y dy
(1 x2) 2
– 2x + n(n + 1) y = 0
dx dx
y = Pn(x) is a solution of it. So,
(1 x2) Pn (x) 2x Pn (x) + n(n + 1) Pn (x) = 0 ...(1)
(i) Put x = 1 in (1), 2Pn (1) + n(n + 1) Pn (1) = 0
n (n 1)
Pn (1) =
2
| ' Pn (1) = 1
(ii) Put x = 1 in (1),
2Pn (1) + n(n + 1) Pn(1) = 0
2Pn (1) + n(n + 1) (1)n = 0 | See Ex. 3 (ii)
2Pn (1) = n(n + 1) (1)n
Case I. 2Pn (1) = (1)1 . n(n + 1) (1)n = ( 1)n+1 . n(n + 1)
n (n 1)
Pn (1) = (1)n+1 . .
2
Case II. 2Pn (1) = (1)1 . n(n + 1) . (1)n = (1)n1 . n(n + 1)
n (n 1)
Pn (1) = (1)n1 .
2
RODRIGUES FORMULA
1 dn
The relation Pn(x) = (x2 1)n is known as Rodrigues formula.
2 n ! dx n
n
dv
To prove it, let v = (x2 1)n then v1 = = n(x2 1)n1 . 2x
dx
Multiplying both sides by (x2 1), we get
(x2 1)v1 = 2nx (x2 1)n = 2nxv
or (1 x2)v1 + 2nxv = 0
Differentiating (n + 1) times by Leibnitzs theorem, we have
(1 x 2 )vn 2 (n 1) ( 2 x)vn 1
(n 1)n "#
( 2)vn + 2n [xvn+1 + (n + 1) vn] = 0
! 2! $
or (1 x2)vn+2 2xvn+1 + n(n + 1)vn = 0
d(vn )d 2 (vn )
or (1 x2) 2x
+ n(n + 1)vn = 0
2
dx dx
which is Legendres equation and vn is its solution. But the solutions of Legendres
equations are Pn (x) and Qn (x).
dn
Since vn = n
( x 2 1) n contains only positive powers of x, it must be a constant
dx
multiple of Pn(x).
1 d3 1 d3
P3(x) = ( x 2 1) 3 (x6 3x4 + 3x2 1)
23 (3) ! dx 3 48 dx 3
1
= 2
(5x3 3x)
1
Similarly, P4(x) = (35x4 30x2 + 3)
8
1
P5(x) = (63x5 70x3 + 15x)
8
1
P6(x) = (231x6 351x4 + 105x2 5) etc.
6
SOLVED EXAMPLES
1
Example 7. Show that x4 = [8P4 (x) + 20P2 (x) + 7P0 (x)].
35
Sol. We know that P4 (x) = 81 (35x4 30x2 + 3)
1
P2(x) = 2
(3x2 1), P0(x) = 1
1 1
[8P4(x) + 20P2(x) + 7P0(x)] = [35x4 30x2 + 3) + 10(3x2 1) + 7] = x4.
35 35
I1
1
Pn ( x) dx
1
2n n ! I 1
1 dx n
dn
(x2 1)n dx
= n
1 dn 1 2
( x 1) n
"# 1
=0
2 n ! dx n 1 ! $ 1
When n = 0,
I 1
1
P0 (x) dx = I 1
1
1 dx = 2. |Q P0 (x) = 1
1 + ... + P 1 P 1 .
Equations Solutions
1 P 1 P 1 P 1 P
Pn
2 2 2 2 0 2n 1 2n 1
2 2 2
2n 0
h
n0
n
Pn ( x) (1 2 hx h2 ) 1/ 2 ...(1)
1
Put x = in (1),
2
h
1 = (1 h + h )
2
n
Pn 2 1/2 ...(2)
n0
1
Put x = in (1),
2
h
1 = (1 + h + h )
2
n
Pn – 2 1/2 ...(3)
n0
h 1 = (1 + h
2
2n
Pn 2 + h4)1/2 = [(1 + h2)2 h2]1/2
n0
= (1 + h2 + h)1/2 (1 + h2 h)1/2
1 . h
1
h n Pn
2 2
n
= Pn | From (2) and (3)
n0 n0
2 2
Equating the coefficients from both sides of the above equation, we get the
required result.
1 1 1 1 1 ...... 1 1
Pn P0 2 . P2n 2 P1 2 P2n 1 2 P2n 2 P0 2 .
2
Example 12. Let Pn (x) be the Legendre polynomial of degree n. Show that for
any function f(x), for which the nth derivative is continuous
I1
1
f(x) Pn(x) dx =
( 1)n
2 n!n I 1
1
(x2 1)n f (n)(x) dx.
Sol. I1
1
f(x) Pn(x) dx = I 1
1
f (x) .
1
2 n n ! dx n
dn
(x2 1)n dx
Using Rodrigue’s
formula
= n
1
2 n! I1
1
f(x) .
dn
dx n
(x2 1)n dx
I
NOTES
=
1
0
1
f ( x)
dn 1
( x 2 1) n dx
"#
2n n ! ! 1 dx n 1 $
=
( 1)1
n
2 n! I
1
1
f (x )
d n 1
dx n 1
( x 2 1)n dx
=
( 1) 2
2n n ! I 1
1
f ( x)
dn 2
dx n 2
( x 2 1) n dx Integrating by parts again
=
( 1) n
n
2 n! I 1
1
f ( n) ( x) ( x 2 1) n dx .
Integrating by parts
(n 2) times
RECURRENCE RELATIONS
(x h) 0
h n Pn ( x) = (1 2xh + h2) nh
0
n1
Pn ( x)
NOTES
(x h) (1 2hx + h2)3/2 =
0
n h n 1 Pn ( x) ...(2)
(x h) (1 2hx + h2)3/2 = (x h) h
0
n1
Pn ( x) ...(3)
nh n1
Pn ( x) = (x h) h
0
n1
Pn ( x)
0
Comparing the coefficient of hn1 on both sides, we get
n Pn (x) = x Pn (x) Pn1 (x)
BELTRAMIS RESULT
%K0 ,
I1
1
Pm ( x) Pn ( x) dx &K 2 ,
' 2n 1
if m n
if m n
I " 1
uv dx (1 x ) (u v v u)#
1
2
(n m) (n + m + 1) =0
1
! #$
I
1
1
Hence Pm(x) Pn(x) dx = 0, since m n.
1
(1 2xh + h2)1 =
n0
[ h n Pn ( x)]2
n0
h 2 n [Pn ( x)]2 2
m0 n0
hm+n Pm(x) Pn(x)
( m n)
Integrating w.r.t. x between the limits 1 to 1, we have
I
n0
1
1
h 2n [Pn ( x)]2 dx 2
m0 n0
( m n)
I
1
1
h m n Pm ( x) Pn ( x) dx I 1
1
dx
1 2 xh h 2
I I
1 1 dx
or h 2n [ Pn ( x)] 2 dx
1 1 1 2 xh h 2
n0
=
1
log (1 2 xh h 2 )
"# 1
1
[log (1 h)2 log (1 + h)2]
2h ! #$ 1
2h
1
= [log (1 + h) log (1 h)]
h
1 h h h h ...... h h h h
2 3 4 2 3 4 "#
! 2 3 4 2 3 4 #$
= ......
h
2 3 " 5
......#
h h
= h
h! 3 5 $
or
n0
h 2n I1
1
n
2 h h ...... h ......
[P ( x)] dx 2 1
3 5 2n 1
2 4 2n
NOTES
I 1
1
[Pn ( x)]2 dx
2 .
2n 1
1
Pn(x) = {x x 2 1 cos } n d
0
We know that,
I
0
d
a b cos
a b2
2
;a>b ...(1)
I
0
d
1 xz z x 1 cos 2
1 2 xz z 2
I
0
d
1 z { x x 1 cos }2
= (1 2xz + z2)1/2
2
Let z{x x 1 cos } = t, then
I0
d
1 t
n0
z n Pn ( x)
...(2)
If | t | < 1, then (1 t)1 = t
n0
n
From (2), I
0
n0
[z n
{ x x 2 1 cos } n ] d z
n0
n
Pn ( x)
Pn(x) =
1
I0
{x x 2 1 cos } n d.
1 d
Pn(x) =
0
.
{ x x 2 1 cos }n 1
I
Equations Solutions
d
;a>b ...(1)
0 a b cos a b2
2
NOTES
Put a = xz 1 and b = z x 2 1 so that
a2 b2 = (xz 1)2 z2(x2 1) = 1 2xz + z2
With above substitutions, (1) becomes,
I
0
d
2
xz 1 z x 1 cos
1 2 xz z 2
1
= = Pn ( x)
1 x 1 2 z n0
zn
z 1 2
z z
I
0
d
z { x x 2 1 cos } 1
zn 1
P (x)
n 0
n
I 0
d
t 1 zn 1 n0
Now, LHS = I
0
t 1
d
1
t
I0
1
t
1
1
t
1
d
If
1
t
< 1, then LHS = 0
1
t
1 1
t t t
1
1 2 ...... n ...... d =
I
0
n0
t
1
n1
d
= I
0
n0
z n1
1
{ x x 1 cos } n 1
2
d
1
Now, comparing and equating the coefficients of n1 on both sides of eqn.(2),
z
we get
Pn(x) =
1
I
0
{ x x 2 1 cos }n 1
d
.
Pn¢(x) = (2n 1) Pn1(x) + (2n 5) Pn3(x) + (2n 9) Pn5(x) + ...... + Last term.
# # #
To determine an, multiplying both sides of (1) by Pn(x) and integrating w.r.t. x
from 1 to 1, we have
I 1
1
f ( x) Pn ( x) dx an I 1
1
Pn 2 ( x) dx an
2
2n 1
an = n
1
2
I
1
1
f ( x) Pn ( x) dx
SOLVED EXAMPLES
Sol.
I1
1
(1 x 2 ) Pm Pn dx
1
1 d
= (1 x 2 ) Pm Pn Pn {(1 x 2 ) Pm } dx
1 1 dx
| Integrating by parts
= I 1
1
Pn
d
dx
{(1 x2) Pm} dx
= I 1
1
Pn { m (m + 1) Pm} dx | From Legendres differential equation
= m(m + 1) I1
1
Pn Pm dx = m(m + 1) . 0 = 0
(2n + 3) (2n 1) I 1
x 2 Pn+1Pn1 dx
I I
1
= n (n + 1)
1
Pn 2 dx + n(n + 2)
I 1
Pn Pn 2 dx + (n2 1)
1
1
Pn 2 Pn dx
I
1 1
1
+ (n + 2) (n 1) Pn 2 Pn 2 dx
1
2
= n(n + 1) . [Using orthogonal properties]
2n 1
I1
1
x 2 Pn 1 Pn 1 dx
2n (n 1)
(2n 1)(2n 1) (2n 3)
.
I1
0
Pn2 ( x) dx
I
0
1
Pn2 ( y) dy I0
1
Pn2 ( x) dx
2
2n 1
I
0
1
Pn2 ( x) dx I0
1
Pn2 ( x) dx
2
2n 1
I 0
1
( 1) 2n Pn2 ( x) dx I1
0
Pn2 ( x) dx
2
2n 1
2 I0
1
Pn2 ( x) dx
2
2n 1
I0
1
Pn2 ( x) dx
1
2n 1
.
I1
1
Pn (x) (1 2xt t 2 )1/2 dx
2t n
2n 1
where n is a positive integer.
I
1
1
= Pn ( x) { t n Pn ( x)} dx
1
= tn
I 1
1
Pn2 ( x) dx
I
All other terms vanish since
1
1
Pm ( x) Pn ( x) dx 0 ; m n
NOTES
2
= tn . . | By II orthogonal property
2n 1
Sol. I 1
1
x m Pn ( x) dx I1
1
xm
1
2 n n ! dx n
dn
(x2 1)n dx (Using Rodrigues Formula)
2n n !
=
1
I 1
1
xm
dn
dx n
(x2 1)n dx
Similarly,
1 I 1 m(m 1)
x m Pn ( x) dx ( 1) 2
2n n !
Integrating (m 2) times in all, we get
I 1
1
xm 2
dn 2
dx n 2
(x2 1)n dx
I = ( 1)m
m(m 1) ... 1
2n n ! I 1
1
dn m
dx n m
(x2 1)n dx
=
( 1) m m !
2n n ! I 1
1
dn m
dx n m
(x2 1)n dx
=
( 1) m m ! d n m 1
( x 2 1) n
"# 1
2n n ! dx n m 1 ! $ 1
= 0.
Example 19. Prove that: xPn = nPn + (2n 3) Pn2 + (2n 7) Pn4 + ...
Sol. From Recurrence relation (2),
xPn = n Pn + Pn1 ...(1)
I I
Sol.
1 1
Pn ( x Pn ) dx = Pn [nPn ( 2n 3) Pn 2 ( 2n 7) Pn 4 ... ] dx
I I I
1 1
1 1 1
= nPn2 dx ( 2n 3) Pn Pn 2 dx ( 2n 7) Pn Pn 4 dx ...
1 1 1
2
=n. + 0 + 0 + ... | Using orthogonal property
2n 1
2n
=
2n 1
Now,
I 1
( x 2 1) Pn 1 Pn dx = I 1
{( x 2 1) Pn } Pn 1 dx
I I I
1 1
1 1 1
= n ( xPn Pn 1 ) Pn 1 dx = n xPn Pn 1 dx n Pn 1 Pn 1 dx
I ' I
1 1 1
1 1
=n xPn Pn 1 dx ...(2) Pn 1 Pn 1 dx 0
1 1
From (2), I
1
1
( x 2 1) Pn+1 Pn dx = n I !
1
1
(n 1) Pn 1 n Pn 1
2n 1
"# . P
$ n+1 dx
=
n (n 1)
2n 1 I1
1
P n 1 dx
n2
2n 1 I1
1
Pn 1 Pn 1 dx
n (n 1) 2 2n (n 1)
= . +0= .
(2n 1) 2 (n 1) 1 (2n 1) (2n 3)
n0
hn 1
n1
Pn ( x) I 0
h dh
1 2hx h 2
= I h
0
dh
(h x) 2 (1 x 2 )
; if | x | <
(h x) h2 2hx 1
= log
1 x
Putting h = x in the expression, we get
1 – x2 1 1 x
xn 1
P (x) = log = 2 log 1 x .
n0
n1 n 1 x
0 , 1 x 0 ()
Example 23. If f(x) =
x , 0 x1
show that:
*
1 1 5 3
f(x) = P0 (x) P1 (x) P2 (x) P (x) + ...
4 2 16 32 4
Sol. We know that
f(x) =
n0
an Pn ( x) ...(1) where an = n
1
2
I
–1
1
f ( x)Pn ( x) dx
=
2n 1
2 ! I 0
1
f ( x) Pn ( x) dx I 1
0
f ( x) Pn ( x) dx
"#
$
=
2n 1
2 I
0
1
f ( x) Pn ( x) dx ...(2)
I 1 1
1 1 1 x2
a0 =
2 0
x P0 ( x) dx 4 2 2 0
I 3x 3 1
3 1 1
x P ( x) dx
2 3
a1 = 1
2 0 2 0
a2 =
5
2 I
0
1
x P ( x) dx
2
5
2
x I
3x 1 dx 5
2 16
1
0
2
a3 =
7
2 I
0
1
x P ( x) dx
3
7
2
x
2 I
5x 3 x dx = 0
0
1 3
a4 =
9
2 I
1
0
x P ( x) dx
4
9
2
x
I
35x 30 x 3 dx 3 and so on.
8
1
0
32
4 2
I I
n0
2n 1 1 2n 1
where, an = f ( x) Pn ( x) dx = f ( x) Pn ( x) dx ...(2)
2 1 2
a0 =
1
2 I 1
2
P0 ( x) dx =
1
4
(2) =
1
2
|Q P0 (x) = 1
3 1
a1 =
2 2
Therefore an = 0 for all n odd.
I
P1 ( x) dx =
3
4 I
x dx = 0
Now, a2 =
5
2 I 1
2
P2 ( x) dx =
5
4 I
3x2 1
2
dx =
5
8
x3 x
5 3 5
a2 = ( ) = (2 1)
4 4
Also, a4 =
9 1
2 2 I
P4 ( x) dx
=
9
4 I
35 x 4 30 x 2 3
8
dx =
9
32
7 x 5 10 x 3 3 x
9
a4 = (75 102 + 3)
16
and so on ......
Hence the Fourier-Legendre series is
1 5 9
f(x) = P (x) + (2 1) P2(x) + (75 102 + 3) P4 (x) + .....
2 0 4 16
n
Example 25. Prove that: (2r 1) P
r0
r = Pn + Pn+1
I "#
Equations 1
1 1
Pn ( x) dx Pn 1 ( x) Pn – 1 ( x)
x 2n 1 ! #$ x
NOTES
1
= [{Pn+1(1) Pn1(1)} {Pn+1(x) Pn1(x)}]
2n 1
=
1
{P (x) Pn+1(x)}.
' Pn 1 (1) 1
2n 1 n1 and Pn 1 (1) 1
1
x 2 Pn2 dx
1
3
1
8 (2n 1) 4(2n 1) 8 (2n 3)
.
(2n + 1)2 I
1
1
x 2 Pn 2 dx = (n + 1)2
I1
1
Pn2 1 dx n2 I1
1
Pn2 1 dx + 2n (n + 1) I
1
1
Pn – 1 Pn 1 dx
2 2
= (n + 1)2 . n2 . +0
[2 (n 1) 1] [2 (n 1) 1]
I 1
1
x 2 Pn2 dx =
2
(2n 1) 2
2n 3
(n 1) 2
!
n2
2n 1
"#
$
1 3 1
=
8 (2n 1) 4 (2n 1) 8 (2n 3)
(on resolving into partial fractions)
1
xPn (x) Pn 1 (x) dx
2n
4n 2 1
.
Therefore, I1
1
xPn ( x) Pn 1 ( x) dx
=
1
2n 1 I 1
1
[ nPn 2 (n 1) Pn Pn 2 ] dx
n 2
=
2n 1 2n 1 | Using orthogonal property
2n
= .
4n2 1
I
1
1
2
Sol. [Pn ( x)] dx
1
= I1
1
[(2n 1) Pn 1 (2n 5) Pn 3 (2n 9) Pn 5 ...]2 dx
= I1
1
(2n 1) 2 P 2 n 1 dx I 1
1
(2n 5) 2 P 2 n 3 dx I 1
1
(2n 9) 2 P 2 n 5 dx ...
+2
I 1
1
(2n 1) (2n 5) Pn 1 Pn 3 dx 2
2 2
I1
1
(2n 1) (2n 9) Pn 1 Pn 5 dx ...
I
2
1
Hence, [Pn ( x)]2 dx = n(n + 1).
1
Example 32. Prove that (1 2xz + z2) 1/2 is a solution of the equation
2
z (z ) (1 x 2 ) = 0.
z 2 x x
Sol. We have, = (1 2xz + z2)1/2 =
n0
z n Pn ( x) or z =
n0
z n 1Pn ( x)
2
z
z 2
( z)
n0
(n 1) n zn Pn(x) ...(1)
Also,
x n 0
z n Pn ( x)
%& () "#
(1 x 2 )
' (1 x 2 ) z n Pn ( x)
* #$
x x x n0 !
= (1 x2)
n0
z n Pn ( x) 2 x z
n0
n
Pn ( x) ...(2)
n0
[(n + 1)n zn Pn(x) + (1 x2) zn Pn(x) 2xzn Pn(x)]
NOTES
= z
n0
n [(1 x2) Pn (x) 2x Pn (x) + n(n + 1) Pn(x)]
EXERCISE
1. Show that:
1 2 2 3
(i) x2 = P0 ( x) P2 ( x) (ii) x3 = P3 ( x) P1( x)
3 3 5 5
8 7 27 "
5
(iii) x = P5 ( x) P3 ( x) P1 ( x) #
63 ! 2 8 $
(i)
I 1
P2n ( x) P2n 1 ( x) dx
I 1
P2n ( x) P2 n 1 ( x) dx (ii)
I 1
x6 P4 ( x) dx
16
I
0 0 2 231
1
(iii) P2n 1 ( x ) dx ( 1)n .
0
9. Prove that:
(i)
I 1
Pn ( x) dx
1 P
n1
n1 (0) (ii)
I 1
P32 ( x)
2
7
I
–1
I
0
1 1
(iii) P n 1 ( x) P n 1 ( x) dx n (n 1) (iv) P2n ( x) dx 0
–1 0
Answers
2 2
2. (i) P0(x) + P1(x) P2(x)
3 3
8 6 2 34 224
(ii) P (x) + P3 ( x) P2 ( x) P1 ( x) P (x) (iii) 2P3 (x) + 4P1(x)
35 4 5 21 5 105 0
2 10 33 2
(iv) P (x) P2 ( x) P1 ( x) P0(x)
5 3 3 5 3
1 3 7 1 3 1 7
5. P0 ( x) P1( x) P (x) + ... 6. P (x) + P1(x) + P2 ( x) P (x) + ...
2 4 16 3 6 0 8 3 48 3
x
7. cos = 0.6366 P0 0.6871 P2 + .0518 P4 .0013 P6 + ...
2
NOTES
7. BESSELS DIFFERENTIAL
EQUATION
STRUCTURE
Introduction
Solution of Bessels equation
Series Representation of Bessel functions
Recurrence Relations for Jn(x)
Generating function for Jn(x)
Integral Form of Bessel Function
Equations Reducible to Bessels Equation
Modified Bessels Equation
BER and BEI Functions
Orthogonality of Bessel Functions
Fourier-Bessel Expansion of F(x)
INTRODUCTION
d2 y dy
x2 2
x + (x2 n2)y = 0
dx dx
is called Bessels differential equation of order n, where n is a positive constant.
This equation can also be put in the form
d dy + (x
x x 2 n2) y = 0
dx dx
The particular solutions of this equation are called Bessels functions of order n.
Bessels equation is
d2 y dy
x2 2
x + (x2 n2)y = 0 ...(1)
dx dx
dy
Then,
dx
k0
( m k) ak x m k 1
2
d y
and
dx 2
k0
( m k) (m + k 1) ak xm+k2
dy d2 y
Substituting for y, and in (1), we get
dx dx 2
x2
k0
(m k ) (m k 1) ak x m k 2 x
k0
(m k ) ak x m k 1
+ (x2 n2) a
k0
k xm k = 0
or [(m k)
k0
2
(m k) (m k) n 2 ] ak x m k a
k0
k xm k 2 = 0
or [(m k)
k0
2
n 2 ] ak x m k a
k0
k xm k 2 = 0
y1 = a0xn 1
x 2
x "
......#
4
! 2 (2n 2) 2 4 ( 2n 2)( 2n 4 ) #$
.
= a0xn
1 ( 1) 1 x 2
( 1)
x 2
"
...#
4
! 2
2 (1) ! (n 1) 2 ( 2) ! (n 1)(n 2) #$
4
( 1) k
= a0xn
k0 2 2 k ( k) ! ( n 1) ( n 2) ...... (n k)
x2k
( 1) k (n 1)
y1 = a0xn
k0
2k
2 . k ! (n k 1)
x2k ...(3)
2
( 1) k ( n 1)
( 1) k
x
n 2k
y1 =
2 n ( n 1) k0
2k
. k ! ( n k 1)
x 2k
k0
( k ) ! ( n k 1) 2
This is called Bessel function of the first kind of order n and is denoted by
Jn(x). Thus,
( 1) k x n 2k
Jn(x) =
k0
(k) ! ( n k 1) 2
which is called Bessel function of the first kind of order n.
When n is not an integer, Jn(x) is distinct from Jn(x). Hence the complete solution
of the Bessels equation may be expressed as
y = xm 1
x2
x4
x6 "#
......
! (m 2) 2
(m 2) (m 4 ) 2 2 2
(m 2) (m 4 ) (m 6) 2 2
#$
which is a solution if m = 0.
The first solution is given by
( 1) k x 2k
J0(x) =
k0
(k !) 2 2 , since (k + 1) = k !
Now,
y
x m log x 1
x2
x4
......
"#
m !
(m 2 )2 (m 2) 2 (m 4 ) 2 #$
+ xm
x 2
.
2
x4 %& 2 2 () ......"#
! (m 2) 2 m 2 (m 2 )2 (m 4 )2 'm 2 m 4 * #$
y
The second independent solution is given by
m m0
x 1 1 1 x 1 1 1 1 x ......"#
2 4 6
= J (x) log x +
0
! 2 (2 !) 2 2 (3 !) 2 3 2
2
#$ 2
= J (x) log x +
( 1) 1 1 1 ...... 1 x
k 1 2k
0
k 1 ( k !) 2 3 2 k 2
It is denoted by Y0(x) and is called Bessel function of the second kind of
order zero or Neumann function.
Thus the complete solution of the Bessels equation of order zero is
y = AJ0(x) + BY0(x)
Case III. When n is an integer, the two functions Jn (x) and Jn(x) are not
independent but are connected by the relation
Jn(x) = (1) n Jn(x).
Now, when n is an integer, y2 fails to give a solution for positive values of n and
y1 fails to give a solution for negative values of n. Let us find an independent solution
of Bessels equation (1), when n is an integer.
Let y = u(x) Jn(x) be a solution of (1) when n is integral.
dy d2 y
Then u J n uJ n and u J n 2u J n uJ n
dx dx 2
Self-Instructional Material 227
Ordinary Differential
dy d2 y
Equations Substituting the values of y, and in (1), we get
dx dx 2
x2(uJn + 2uJn + uJn) + x(uJn + uJn) + (x2 n2)uJn = 0
I
or u Jn2x = B
B dx
or u = or u=B A
xJ 2n xJ 2n
Substituting the value of u in the assumed solution y = u(x) Jn(x), we have
y= B
! I dx
xJ 2n ( x)
"#
A Jn(x)
#$
or y = AJn (x) + BYn(x), where
xJ 2n ( x)
The function Yn(x) is called the Bessel function of the second kind of order
Yn(x) = Jn(x) I dx
n or Neumann function.
J (x) =
( 1) k
x
n 2k
Since n
k0
k ! (n k 1) 2
J (x) =
( 1) x = ( 1) x , Q
k 2k k 2k
k ! ( k 1) 2 ( k !) 2
0 2 (k + 1) = k !
k0 k0
x 1 x 1 x ......
=1
2 4 6
2 (2 !) 2 (3 !) 2
2 2
x2 x4 x6
= 1 ......
22 22 . 4 2 2 2 . 4 2 . 62
( 1) k x 1 2k
( 1) k x 1 2k
J1(x) =
k0
k ! ( k 2 ) 2
k0
k ! ( k 1) ! 2
x 1 x
1
3
x 5
......
x x3 x5
......
=
2 2! 2
2!3! 2 2 2 2
2 2 .4 2 .4 .6
In particular, J0(0) = 1 and J1(0) = 0.
The values of J0(x) and J1(x) are given in Jahnke Emdes tables to four decimal
places at intervals of 0.1.
Jn =
( 1) r
r ! n r 1 2
x n 2r
r0
( 1)r ( n 2r ) 1 x n 2r 1
Jn =
r0
.
r ! n r 1 2 2
Multiplying both sides by x and breaking it into two terms
( 1)r x n 2r
( 1)r x n 2r 1
xJn = n
r0
r ! n r 1 2 +x
r 1
(r 1) ! n r 1 2
( 1) s 1 x n 2s 1
= nJn + x
s0
s ! n s 2 2 | Here r = s + 1
= nJn x
( 1) s
s ! n s 2 2
x ( n 1) 2s
s0
We know that
( 1)r (n 2r ) x n 2r
( 1)r ( 2n 2r n ) x n 2r
xJn =
r0
r ! n r 1 2 =
r0
r ! n r 1 2
=n
( 1) r
r ! n r 1 2
x n 2r
+x
( 1) r ( n r ) x
r ! n r 1 2
n 2r 1
r0 r0
( 1)r x n 2r 1
= nJn + x
r0
r ! n r 2
( 1)r x n 1 2r
= nJn + x
r0
r ! (n 1) r 1 2
xJn = nJn + xJn1 ...(2)
d
(xn Jn) = xn Jn+1
dx
d
(xn Jn) = xn Jn1
dx
Multiplying eqn. (2) by xn1, we get
xn Jn = n xn1 Jn + xn Jn1
xn Jn + nxn1 Jn = xn Jn1
d
(xn Jn) = xn Jn1 ...(6)
dx
SOLVED EXAMPLES
( 1) r x n 2r
Jn (x) =
rn
r! n r 1 2
Putting r = n + s, we get
( 1) n s
x
n 2s
Jn(x) =
s0
( n s ) ! s 1 2
( 1)r x n 2r
( 1)r
x
n 2r
= (1)n
r0
( n r ) ! r 1 2 = (1)n
r0
n r 1 r ! 2
Jn (x) = (1)n Jn(x).
Example 2. Prove that: J0 (x) = J1 (x).
d
Sol. We know that [xn Jn(x)] = xn Jn+1 (x)
dx
d
Putting n = 0, we get [J (x)] = J1(x) J0(x) = J1(x).
dx 0
1
(i) Putting n = in (1),
2
x 1 x x ..."#
2 4
J1/2(x) =
2 3/2 ! 3! 5! #$
1 "
3 3
...# =
x x x 2
. x
=
2.
1
x ! 3! 5! #$ x
sin x
2
1
(ii) Putting n = in (1),
2
x 1/ 2 1 x x ... =
2 4
J1/2(x) =
2 1/ 2 2 ! 4 !
1/2
2
x
cos x .
Substituting these values of Jn and Jn+1 from (2) and (3) in eqn. (1), we get
n J n1
LHS = 2Jn
x n
J n 1 2J n 1
x
Jn 1 Jn
n 2 n 1 J
2
x
2
=2 Jn n 1 = RHS
x
Hence the result.
Example 5. Prove that:
2 sin x "#
=
x x ! cos x
$ | Using results of Ex. 3
2 cos x "#
=
x x! sin x
$ | Using results of Ex. 3
! x
Example 6. Prove that: J5/2 (x) =
x 2
x #$
Sol. From Recurrence relation (4),
2nJn(x) = x[Jn 1 (x) + Jn + 1 (x)]
2n
Jn + 1 (x) = J (x) Jn 1 (x) ...(1)
x n
Putting n = 1/2, 3/2 in (1), we get
1
J3/2 (x) = J (x) J 1/2 (x) ...(2)
x 1/2
3
J5/2 (x) = J (x) J1/2 (x) ...(3)
x 3/2
From (2) and (3),
3 1 "#
J 1/ 2 ( x) J 1/ 2 ( x) J1/2 (x)
J5/2 (x) =
!
x x $
3 1 J (x) 3 J (x) = 3 x 2 sin x 3 . 2
2
x
= 2
x 1/2 x x
1/2 x 2
x
cos x
2 3 x "
cos x # .
2
3
=
x ! x sin x
2
x #$
48 8 J (x) 1 24 J (x) .
Example 7. Prove that: J (x) =
x x
4 3 1 2 0
x
Hence or otherwise find J6 (x) in terms of J0(x) and J1(x).
Sol. From Recurrence relation (4),
2nJn (x) = x [Jn 1 (x) + Jn + 1 (x)]
2n
Jn + 1 (x) = J (x) Jn 1 (x) ...(1)
x n
232 Self-Instructional Material
Putting n = 1, 2, 3 in eqn. (1), we get Bessels Differential
Equation
1
J2 (x) =
[2J1 (x) xJ0 (x)] ...(2)
x
1
J3 (x) = [4J2 (x) xJ1 (x)] ...(3) NOTES
x
1
J4(x) = [6J3 (x) xJ2 (x)] ...(4)
x
From (2) and (3),
8 4
J3 (x) = 2
J 1 ( x) J 0 ( x) J 1 ( x)
x x
8 x J
=
2
4
x 2 1 ( x)
x
J 0 ( x) ...(5)
=
48 8 J (x) 1 24 J (x) .
x x 3 x 1 2 0 ...(6)
=
80 1 48 8 J (x) 1 24 J (x)"#
x ! x x
2 x $
3 1 2 0
10 8
1 J (x) 4x J (x)"#$ | Using eqn. (5) and
x ! x 2 1 0
(6)
3840 768 18 J (x) + 144 1920 1 J (x).
J6(x) =
x x x 5 3 x x 1 2 4 0
I x n J n 1 ( x) dx = x n Jn (x) ...(1)
I
Equations
= x2 . [ x2 J2 (x)] 2 x . [ x 2 J 2 ( x)] dx
NOTES = J2 (x) + 2
I x –1 J 2 ( x)dx
2
= J2 (x) + 2 [ x1 J1 (x)] = J2 (x) J 1 ( x)
I
x
2
or J 3 ( x) dx J 2 ( x) J 1 ( x) = 0.
x
Sol. I x J 20 ( x) dx J 20 ( x) .
x2
2
I 2J 0 ( x) J 0 ( x) .
x2
2
dx c
=
x2 2
2
J 0 ( x) I x 2 J 0 ( x) { J 1 ( x)} dx c [ ' J0 (x) = J1 (x)]
=
x2 2
2
J 0 ( x) I xJ 1 ( x) . xJ 0 ( x) dx c
=
x2 2
2
J 0 ( x) I xJ 1 ( x) .
d
dx
[ xJ 1 ( x)] dx c
| Using Recurrence relation
x2 2 [ xJ 1 ( x)]2 x2 2
= J 0 ( x) c = [ J 0 ( x) J 21 ( x)] c .
2 2 2
Example 10. Prove that: 4 Jn (x) = Jn 2 (x) 2Jn (x) + Jn + 2 (x).
Sol. From Recurrence relation (3), we have
2Jn = Jn 1 Jn + 1 ...(1)
Differentiating, 2Jn = Jn 1 Jn + 1
or 4Jn = 2Jn 1 2Jn + 1 = (Jn 2 Jn) (Jn Jn + 2) | Using (1)
4Jn = Jn 2 2Jn + Jn + 2
3 1
= J0 J3
4 4
4J0 + 3J0 + J3 = 0.
J n (x) 1
Example 13. Prove that: lim n
n
; (n > 1).
x0 x 2 n1
Sol. We know that:
1 x
xn x 2 "
......#
4
Jn (x) =
2 n 1 !n .
2 . ( 2n 2) 2 4 . ( 2n 2) ( 2n 4 ) #$
lim
J (x)
n
lim
1 1 x x 2 "
– .....#
4
x0 x n
2 n 1 !
x0 n . . .
2 ( 2n 2) 2 4 ( 2n 2)( 2n 4 ) #$
1
= n .
2 n1
4 J 2
Example 14. Prove that: J2 (x) = 1
x 2 1 (x)
x
J0 (x) .
x
Example 18. Prove that: J n (n 1) J n 1 (n 3) J n 3 (n 5) J n 5 – .....
2
Sol. By Recurrence relation (4), we have
2nJn = x(Jn 1 + Jn + 1)
2(n + 1) Jn + 1 = x (Jn + Jn + 2) | Replacing n by (n + 1)
x x
J n (n 1) J n 1 J n 2 ...(1)
2 2
x x
J n 2 = (n 3) J n 3 J n 4 ...(2)
2 2
From (1) and (2),
x x
J n (n 1) J n 1 (n 3) J n 3 J n 4
2 2
Continuing this way,
x
J n ( n 1) J n 1 ( n 3) J n 3 ( n 5) J n 5 – .....
2
I0
x
x n J n 1 ( x) dx x n J n ( x)
x
0
= xn Jn (x) + Lt
1
x0
J (x) "#
! x $
n
n
= xn Jn (x) + n
.
2 n1
2 n
J n (n 2) J n 2 (n 4) J n 1 – ...... .
"#
Example 20. Prove that: Jn =
!
x 2 $
Sol. From Recurrence formula (2), we have
n
Jn = J n Jn 1
x
n 2
= J n [ nJ n ( n 2 ) J n 2 ... ] |Using
x x
example 18
2 n
J n (n 2) J n 2 ... .
"#
=
x 2 ! $
x 1
z
2 z
The function e is called generating function.
Prove that
x 1
z
z n
(i) e2
z
= J n ( x)
n
x 1
z
i.e., Jn (x) is the coefficient of zn in the expansion of e 2 z
.
x 1
z
(ii) e 2 z
=
n
( 1) n
J n ( x) z n
x 1
z
(1)n Jn (x) is the coefficient of zn in the expansion of e 2 z
i.e., .
Note. Above results are true if n is an integer.
Proof. We have,
x 1 xz x
z
2 z
e = e2 e 2z
xz x 2
z 2 ..... "#
x x 2
1 ..... "#
= 1
! 2
2 2!
1
#$ !
2z 2z 2!
#$
Self-Instructional Material 237
Ordinary Differential (i) Coeff. of zn in this product
Equations
x n
1 x n2
1 x n4
1 . 1 ......
=
2 n!
2 .
( n 1) !
2 .
(n 2 ) ! 2!
NOTES
( 1) r x n 2r
=
r0
r ! ( n r) ! 2 = Jn
x n
1 x n2
1 x n4
1 1
=
2 n!
–
2 .
(n 1) !
2 . .
(n 2) ! 2 !
......
( 1) r x n 2r
= (1) n
r0
r ! (n r ) ! 2 = (1)n Jn.
t
t n
We know that e 2 t
= J n ( x)
n
= J0 (x) + tJ1 (x) + t2J2 (x) + t3J3 (x) + ...... + t1J1 (x) + t2 J2 (x) + t3J3 (x) + ......
= J0 (x) + tJ1 (x) + t2J2 (x) + t3J3 (x) + ...... t1J1 (x) + t2 J2 (x) t3J3 (x) + ......
'
[ Jn (x) = (1)n Jn (x)]
1 1
1
2 3
= J0 (x) + t J1 (x) + t 2 J2 (x) + t 3 J3 (x) + ...... ...(1)
t t t
Put t = cos + i sin
1
tn = cos n + i sin n and = cos n i sin n
tn
| By De-Moivres theorem
n 1 n 1
so that t = 2 cos n and t n = 2i sin n
tn t
Substituting these values in (1), we have
eix sin = J0 (x) + 2i sin J1 (x) + 2 cos 2 J2 (x) + 3i sin 3 J3 (x) + ......
...(2)
Since eix sin = cos (x sin ) + i sin (x sin )
Equating the real and imaginary parts in (2), we get
cos (x sin q) = J0 (x) + 2[J2 (x) cos 2q + J4 (x) cos 4q + ......] ...(3)
sin (x sin q) = 2[J (x) sin q + J (x) sin 3q + ........]
1 3 ...(4)
These are known as Jacobi series.
Multiplying both sides of (3) by cos n and integrating w.r.t. between the
limits 0 and (when n is odd, all terms on the RHS vanish; when n is even, all terms
on the RHS except the one containing cos n vanish), we get
I
0
cos ( x sin ) cos n d
%&0,
' J n
when n is odd
( x), when n is even
I
0
sin ( x sin ) sin n d
%&J
' 0,
n ( x), when n is odd
when n is even
NOTES
Adding, we get I
[cos ( x sin ) cos n sin ( x sin ) sin n] d = Jn (x)
I
0
1
Jn (x) = cos ( n x sin ) d for all integral values of n.
0
SOLVED EXAMPLES
I0
cos 2 n d I0
sin 2 n d
2
and I0
cos m cos n d I0
sin m sin n d = 0, m n, we get
0
cos 2 ( x sin ) d
Adding, we have
2[J1 (x)]2 + 2[J3 (x)]2 + ...... =
=
I
0
[cos 2 ( x sin ) sin 2 ( x sin )] d
x.
1 1
z
1 1 1
J1 z 2 2 J 2 z 3 3 J 3 + ...
2 z
e J0 z ...(1)
z z z
1
Putting z = ei so that = ei
z
1 1
and z = 2 cos ; z = 2i sin ,
z z
I 0
cos ( x cos ) d J 0 I 0
d 2 J 2 I
0
cos 2 d 2J 4 I0
cos 4 d ... = J0
J0 =
1
I
0
cos ( x cos ) d .
EXERCISE A
1. Show that:
2
(iii) [J1/2 (x)]2 + [J1/2 (x)]2 =
x
2 15 6 x2 15 "
(iv) J7/2(x) = 3 sin x 2
1 cos x # .
x ! x x #
$
2. Show that:
1
(i) J0 = (J J0) (ii) J2 = J0 x1 J0
2 2
J 1 J 1
(iii) 2 = 0 (iv) J1(x) = J1(x) + J (x).
J1 x J0 x 2
3. Prove that:
d n d
(i) [ x J n (ax)] = ax n Jn 1 (ax) (ii) [J 2n ( x)] = x [ J 2 n 1 ( x) J 2n 1 ( x)] .
dx dx 2n
4. Prove that:
(i) I J0 ( x) J1 ( x) dx
1 2
J0 ( x) (ii) I x2 J0 ( x) J 1 ( x) dx
1 2 2
x J 1 ( x)
I I
2 2
J 4 ( x) 1 2 4 8
(iii) dx J 3 ( x) 2 J 2 ( x) (iv) J 5 ( x) dx J 4 ( x) J 3 ( x) 2 J 2 ( x)
I
x x x x x
(v) x 3 J 0 ( x) dx x3 J 1 ( x) 2 x 2 J 2 ( x)
5.
(vi) I
0
Prove that:
x J 0 (x) dx
J 1 ( ) .
(i) Jn (x) =
1 2
2 0 I
cos ( x sin n) d
(ii) cos x = J0 2J2 + 2J4 ...... ; sin x = 2J1 2J3 + 2J5 .......
[Hint: Put = /2 in Jacobi series]
[Hint: Replace by in Jacobi series]
2
NOTES
(iv) J0 (x) =
1
I 0
cos ( x sin ) d
2
I
0
/2
cos ( x sin ) d
2
I0
/2
cos ( x cos ) d
6.
(v) J0 + 2J2 + 2J4 + 2J6 + ..... = 1 (vi)
Show that Bessels function Jn (x) is an even function when n is even and is an odd
I 0
/2
x J 1/2 (2 x) dx = 1.
8 1 J (x) 4 J (x) .
(iii) Show that J3(x) =
x 2
x
1 0
8. Show that:
9. Evaluate:
I x 4 J 1( x) dx .
Answers
d2 y dy
x2
+ (1 2)x + [22 x2 + (2 n22)]y = 0 ...(1)
dx 2 dx
where , , and n are constants.
Putting X = x and Y = x y, equation (1) reduces to
d2 Y dY
X2 2
X (X 2 n 2 ) Y 0 ...(2)
dX dX
which is Bessels equation.
SOLVED EXAMPLES
2 dy d2 y
The differential equation x (x 2 n2 ) y 0
2
x ...(1)
dx dx
is called modified Bessels equation of order n.
dy d2 y
Equation (1) can be re-written as x 2 (i 2 x 2 – n 2 ) y 0
2
x
dx dx
When n is not an integer, its solution is given by y = c1Jn (ix) + c2Jn (ix)
( 1) k ix
n 2k
Now, Jn (ix) =
k0
k ! (n k 1) 2
k! (n k 1) 2
n 2k
1 x
=i
n
[' (1)k (i)2k = i2k . i2k = i4k = 1]
r0
It is denoted by In (x) and is called the modified Bessel function of the first kind
NOTES of order n.
Thus, In(x) = inJn (ix)
Since in is a constant, In (x) is also a solution of (1).
If n is not an integer, a second independent solution of (1) is In (x),
1 x n 2k
where In (x) =
r0
k ! ( n k 1) 2
Thus, if n is not an integer, the complete solution of (1) is given by
y = c1In (x) + c2In (x)
If n is a non-zero integer, a second independent solution of (1) is given by
/2
Kn (x) =
[I (x) Jn (x)]
sin n n
and is called modified Bessel function of second kind of order n.
In this case, the complete solution of (1) is given by
y = c1In(x) + c2Kn(x).
d 2 y dy
Consider the differential equation x ixy 0 ...(1)
dx 2 dx
Comparing it with equation (1) of Art. 4.7, we have
= 0, n = 0, = 1 and 2 = i or 2 = i3 so that = i3/2.
Hence a solution of (1) is given by J0 (i3/2 x)
Replacing x by i3/2 x in the series for J0 (x), we have
i3 x 2 i6x 4 i9x 6 i12 x 8
J0 (i3/2 x) = 1 ......
22 ( 2!) 2 . 2 4 (3!) 2 . 2 6 ( 4 !) 2 . 2 8
= 1
x4
x8
– .....
"#
! 22 . 4 2 22 . 4 4 . 62 . 82 #$
+i
x 2
x6
x 10 "#
.......
!2 2
2 . 4 2 . 62
2
2 2 . 4 2 . 6 2 . 82 . 10 2 #$
Thus J0 (i2/3 x) is a complex function for real values of x. The real and the
imaginary parts are denoted by ber (x) (Bessel-real) and bei (x) (Bessel-imaginary)
respectively. Thus
( 1) k x 4 k
ber(x) = 1 +
k 12
2 . 2 . 2 .......
4 6 ( 4k )2
( 1) k x 4 k 2
and bei(x) = 1 2
k 1
2 . 2 . 2 .......
4 6 ( 4k 2) 2
Hence a solution of (1) is y = J0(i3/2 x) = ber (x) + i bei (x).
5
x x x9
(b) Also, bei (x) = 2 2 2 2 2 2 ........
2 2 . 4 . 6 2 . 4 . 6 . 8 . 10
x2 x6 x 10
x bei (x) = 2 2 2 2 2 2 ........
2 2 . 4 . 6 2 . 4 . 6 . 8 . 10
d x5 x9
[x bei (x)] = x 2 2 2 2 2 2 ........
dx 2 .4 2 .4 .6 .8
x4 x8
= x 1
2 2 . 4 2 22 . 4 2 . 62 . 8 2
........ = x ber (x).
I0
1
x J n ( x) . J n ( x) dx
%& 0 ,
'J
1
2
2
when
n 1 () , when
()
*
Consider the Bessels equations
x2u + xu + (2x2 n2)u = 0 ...(1)
and x2v + xv + (2x2 n2) v=0 ...(2)
Their solutions are u = Jn (x) and v = Jn (x) respectively.
v u
Multiplying (1) by and (2) by and subtracting, we get
x x
x(uv uv) + (uv uv) + (2 2) xuv = 0
d
or [x(uv uv)] = (2 2) xuv
dx
I "# "#
Equations 1
1
= u v uv
(2 2)
0
xuv dx x (u v uv )
! $ 0 ! $ x1
...(3)
I
0
1
xJ n (x) J n (x) dx
2 2
J n () J n () J n () J n ()
I0
1
xJ n (x) J n (x) dx 0
0
However, if = , the value of the integral is , which is indeterminate.
0
To evaluate the integral, we assume that is a root of Jn(x) = 0 so that Jn() = 0
and is a variable approaching . Thus, from (4), we have
Lt
0 I 1
xJ n (x) J n (x) dx Lt
J n () J n ()
2 2
or I0
1
xJ 2n (x) dx Lt
J n () J n ()
2
[by L-Hospitals rule]
1
= [J ()]2 ...(5)
2 n
But xJn (x) = nJn (x) xJn + 1 (x)
xJn () = nJn () Jn + 1 () = Jn + 1 (), since Jn () = 0
Jn () = Jn + 1 ()
I0
a
xJ n2 (x) dx
a2 2
2
J n 1 (a) , where is a root of Jn (a) = 0.
From the orthogonal property of Bessel functions, we can expand a function f(x) in
Fourier-Bessel series in the range 0 to a.
Let f(x) = c1Jn (1x) + c2Jn (2x) + ....... + cnJn (nx) + ........ = cJ
i1
1 n ( i x ) ...(1)
where 1, 2, ........ are the roots of the equation Jn (a) = 0.
I a
xf ( x) J n ( i x) dx ci I a
xJ n 2 ( i x) dx ci .
a2 2
J n 1 ( i a) NOTES
I
0 0 2
2 a
ci = 2 2
xf ( x) J n ( i x) dx
a J n1 ( ia ) 0
Putting i = 1, 2, 3, ....... we can find c1, c2, c3 , ........ and hence the function f(x).
SOLVED EXAMPLES
Example 27. If 1, 2, ..........., n are the positive roots of J0 (x) = 0, prove that
1 J0 ( n x)
2 n 1 n J1 ( n )
.
then ci = 2 2
a J n1
2
( i a) I
0
a
xf ( x) J n ( i x) dx
I "# 1
2 1 2 xJ 1 ( i x) 2
ci = xJ 0 ( i x) dx 2
2
J 1 ( i ) 0 J 1 ( i ) i ! $ 0
i J 1 ( i )
2
From (1), we have 1 =
i1 i J 1 ( i )
J 0 ( i x)
1 J 0 ( n x)
or
2 n 1 n J 1 ( n )
.
Example 28. Show that the Fourier-Bessel series in J2(nx) for f(x) = x2 (0 < x < a),
where n a are positive roots of J2 (x) = 0, is
J 2 ( n x)
x2 = 2a
2
n1
a n J 3 ( n a)
.
Multiplying both sides by xJ2(nx) and integrating w.r.t. x between the limits 0
to a, we get
I0
a
x 3 J 2 ( n x ) dx cn I 0
a
xJ 22 ( n x ) dx
x J ( x) "#
3
3 n
a
a2 2
or
! #$ n 0
= cn . J ( a)
2 3 n
EXERCISE B
1 1
(vii) y + y + 8 2
y = 0 (viii) 4y + 9xy = 0
x x
1 y 1
(ix) xy + y + y = 0 (x) y + 1 y = 0
4 x 9x 2
2. Expand f(x) = 1 over the interval 0 < x < 3 in terms of the functions J0 (nx), where n are
determined by J0(3) = 0.
3. Expand f(x) = 4x x3 over the interval (0, 2) in terms of Bessel functions of first kind of
order one which satisfy the condition [J1 (x)]x = 2 = 0.
4. If 1, 2, ........, n, ........ are the positive roots of J1 (x) = 0, prove that
J 0 ( n x)
1 1 J0 ( n x)
(i) x2 =
2
4
n1 n
2
J 0 ( n )
(ii) (1 x2)2 =
3
64
n1 n
2
J0 ( n )
.
(i) I
0
1
J 1 ( ax) dx
1
a
(ii) I
0
a
J 1 ( x) dx 1 .
Answers